d120

¡Supera tus tareas y exámenes ahora con Quizwiz!

1. Discuss the incidence of early pregnancy loss and define the types of spontaneous abortion

. a. Spontaneous abortion occurs in 10-15% of clinically recognizable pregnancies. b. more than 50% of all conceptions are lost, the majority in the 14 days following conception. c. if a live, appropriately growing fetus is present at 8 weeks' gestation, the fetal loss rate over the next 20 weeks (up to 28 weeks) is in the order of 3%.

. Perinatal infections

1 i. Perinatal infections are sometimes listed using the acronym "TORCH," the letters standing for Toxoplasmosis, Other, Rubella, Cytomegalovirus, and Herpes simplex virus.

Discuss the risk factors, evaluation, and management of testosterone deficiency.

1. a. Low testosterone levels are associated with established cardiovascular disease (CVD) risk factors and an increased risk of death.1 Low testosterone can also significantly reduce a man's quality of life. Common symptoms include erectile dysfunction (ED), low libido, loss of night time erections, tiredness and depression. 1. Measuring serum luteinizing hormone (LH) helps differentiate between primary and secondary hypogonadism ii. Proven testosterone deficiency requires testosterone replacement, either with daily testosterone gel or long-acting testosterone undecanoate injections every 3 months. The therapeutic goal is to achieve and maintain serum testosterone levels in the low to middle normal range of young adult men iii. Men on TRT must be carefully followed up with a testosterone blood test 2 hours after the gel is rubbed in or just before the next injection is due, plus PSA and haematocrit every 6 to 12 months once established on treatment.

Discuss the manifestations, diagnostics and management of BPH.

1. a. bladder outlet obstruction (BOO), lower urinary tract symptoms (LUTS), i. Irritative symptoms include frequency, urgency, and nocturia and occur as a result of decreased functional bladder capacity and instability or infection b. Diagnostics i. Unialysis, post residual voids with bladder ultrasound ii. PSA if suggestive of PC hematuria 1. BPH not a risk for CA c. Management i. Limiting fluids before bed, limit caffeine alcohol and double voiding 1. Terazosin and doxazosin at bedtime 2. 5α-reductase inhibitor therapy (e.g., dutasteride and finasteride) 3. sildenafil or tadalafil for ED ii. Herbal no safety/efficacy 1. Serenoa repens (saw palmetto), 2. rye-grass pollen extract (Cernilton), 3. Pygeum africanum (African prune tree extract) iii. TURP 1. Prostates less the 100g increased mortality

b. homosexual men,

1. Any MSM who is at high risk for HIV infection should be offered HIV pre-exposure prophylaxis (PrEP).33 HIV PrEP is a comprehensive program of HIV prevention which includes once daily tenofovir/emtricitabine. HIV PrEP can prevent up to 92% of new cases of HIV and at the 22nd International AIDS conference in 2018, it was revealed that on-demand PrEP (day before and after high risk behavior) and daily PrEP were equivalent

Indications and Dosing for Rh Immune Globulin

1. Blood type and antibody screen are performed for all pregnant women at their first prenatal visit. 2. Women who are RhD-negative with a negative initial screen should have a repeat screen at 28 weeks. 3. Those women with a negative screen at 28 weeks should receive 300 µg of Rh immune globulin (prophylactically). 4. Those women with a positive screen should have their antibodies identified. If RhD-negative, they should also receive 300 µg of Rh immune globulin. 5. All pregnant women who are RhD-negative and who are not sensitized (anti-D-negative) and who experience (1) spontaneous or induced abortion, (2) ectopic pregnancy, (3) significant vaginal bleeding, (4) amniocentesis, (5) abdominal trauma, or (6) cephalic version should receive 50-100 µg of Rh immune globulin before 12 weeks' gestation and be administered 300 µg if later than 12 weeks. 6. Rh immune globulin is not necessary for complete molar pregnancies, but it is necessary for partial molar pregnancies, where fetal tissue may be present. Because this is not always clear at the time of evacuation, 300 µg of the immune globulin should be given. 7. The greatest risk of fetomaternal hemorrhage is at the time of delivery. Rh immune globulin (300 µg) should be given routinely within 72 hours of delivery to all Rh-negative, anti-D-negative women who deliver an Rh-positive child. 8. Additional Rh immune globulin is indicated if the delivery is complicated by excessive hemorrhage (>30 mL of fetal blood suspected or documented by Kleihauer-Betke testing).globulin.

TX-a. Gonorrhea

1. Ceftriaxone 250 mg* IM once plus 2. Azithromycin 1 g† once or doxycycline 100 mg† bid for 7 days 3. Alternative therapy (if ceftriaxone not available) 4. Gentamicin 240 mg IM plus azithromycin 2 g orally once 5. Cefoxitin 2 g IM once plus probenecid 1 g once plus azithromycin 1 g once 6. Doxycycline 100 mg bid for 7 days 7. Cefixime 400 mg once plus azithromycin 1 g once or doxycycline 100 mg bid for 7 days a. Avoid sexual intercourse until therapy is completed (or for 7 days after single-dose therapy) b. When treating, consider chlamydia; consider treating sexual partners exposed within the last 60 days c. Retest 3 mo after treatment or at earliest opportunity within 12 mo using NAAT testing d. If reinfection suspected, obtain culture and repeat with same antibiotic regimen0 e. If drug resistance suspected, consider increasing dose of ceftriaxone to 1 g and azithromycin 2 g once f. Report the infection to the health department 8. Test of cure should be done with NAAT, at least 28 days after completion of treatment; if standard treatment and no continued symptoms then no test of cure needed 9. With positive test of cure: 10. If reinfection suspected, obtain culture and repeat with same regimen 11. If drug resistance suspected, consider increasing dose of ceftriaxone to 1 g and azithromycin 2 g once 12. Refer to infectious disease specialist if continued positive test of cure

i. Recommend HPV immunizations

1. Gardasil 9 for males and females. 2. 9 to 14 yr of age: two dose series, 6 mo apart 3. 15 to 26 yr of age: three dose series at 0, 1 to 2, and 6 mo 4. If doses are <5 mo between two dose series and <5 mo between first and third dose, then a booster needs to be given at the appropriate interval 5. Not recommended during pregnancy but if becomes pregnant on series, no intervention is required. 6. Immunocompromised persons should receive three dose series. 7. If patient has completed a valid series of any HPV vaccine, no additional doses are required. 8. Refer to a dermatologist, plastic surgeon or GYN for possible surgical or laser therapy to excise or ablate multiple lesions

HERPES TX

1. Initial outbreak therapy for 7 to 10 days; should be started within 72 hr for best results, but do not withhold medications if presents later (can be extended if not healed in 10 days) 2. Acyclovir 400 mg tid or 200 mg 5 ×d 3. Famciclovir 250 mg tid 4. Valacyclovir 1 g bid 5. B. Episodic/recurrent therapy should be started at first signs of infection 6. Acyclovir 400 mg tid or 800 mg bid for 5 days or acyclovir 800 mg tid for 2 days 7. Famciclovir 125 mg bid for 5 days or 1000 mg bid for 1 day or 500 mg once, followed by 250 mg bid for 2 days 8. Valacyclovir 500 mg bid for 3 days or 1 g qd for 5 days 1. Suppressive therapy (e.g., for ≥10 outbreaks/yr) requires daily therapy a. Acyclovir 400 mg bid b. Famciclovir 250 mg bid c. Valacyclovir 500 mg or 1 g qd A. Reevaluate annually and consider providing the patient with refills depending on use to ensure immediate treatment of outbreaks

a. Genital warts S/S

1. May be small, singular, or multiple papules or plaques; can have a typical "cauliflower" appearance or be flat, pedunculated, or rough 2. Warts can be found on the vulva, in the vaginal or anal opening, under the foreskin of the penis, or on the scrotum; warts may be large enough to limit vaginal, urethral, or anal opening 3. Depending on the size and location, warts can be painful and pruritic 4. May regress spontaneously

a. Bacterial vaginosis tx

1. Metronidazole 500 mg PO bid for 7 days 2. Metronidazole gel 0.75% intravaginal applicator once qd for 5 days 3. Clindamycin 2% vaginal cream 1 applicator hs for 7 days 4. Clindamycin 300 mg bid for 7 days or 5. Clindamycin 100 mg vaginal supp hs for 3 days or 6. Tinidazole 1000 mg qd for 5 days or 2000 mg qd for 2 days 7. Secnidazole (Solosec) 2 g PO once; sprinkle in applesauce and finish within 30 min; do not chew granules 8. Intermittent treatment with vaginal metronidazole gel q1wk or 2 g PO q1mo 9. Intravaginal boric acid 600 mg at hs 1 to 2 times weekly may control symptoms (boric acid taken orally can cause death) 10. Treat for any concurrent Candida infection with diflucan 150 mg 11. Follow-up in 1 mo to repeat Amsel criteria, condom use, and COCs may ↓ recurrence and IUDs may ↑ infection risk to upper GU organs

a. Syphilis i. . Diagnostic testing

1. Need to use combination tests to avoid false-positive results: RPR and VDRL; if positive, order FTA-ABS 2. Test for HIV, GC/CT, hepatitis B and C

adls

1. Patients are scored as a yes/no for independence: a score of six indicates full function, four indicates moderate impairment, and a score of two or lower indicates significant impairment.

SyphilisSyphilis TX

1. Primary stage: benzathine penicillin G 2.4 million U IM in a single dose or amoxicillin 3 g + probenecid 500 mg bid for 14 days; if allergic to PCN, doxycycline 100 mg bid for 14 days 2. If allergic to penicillin, treat for 14 days with 3. Doxycycline 100 mg bid 4. Ceftriaxone 1 to 2 g IM qd 5. Tetracycline 500 mg qid 6. Refer to infectious disease specialist and health department

a. Thyroid storm i. Risks include infection, labor, cesarean delivery, or noncompliance with the medication regimen

1. S/S a. Hyperthermia, Marked tachycardia, Sweating, High output RF, Dehydration (severe) pos. fetal tachy 1. Treatment a. blocking β-adrenergic activity and controlling maternal heart rate with propranolol, b. blocking synthesis of thyroid hormone and conversion of T4 to T3 with PTU, c. administering potassium iodide 1 to 2 hours after starting PTU to block secretion of thyroid hormone, d. additional blocking of the deamination of T4 to T3 with glucocorticoids (dexamethasone), e. replacing fluid losses, and f. rapidly lowering the patient's body temperature with hypothermic techniques. i. Once the patient is stabilized and no longer acutely ill, methimazole should be substituted for PTU to avoid hepatotoxicity.

i. BV

1. Treatment is recommended for all symptomatic women to reduce the signs and symptoms. Standard treatment is with either 250 or 500 mg of oral metronidazole (Flagyl) twice a day for 7 days, which has been shown to be safe in pregnancy. However, 5 mg of 0.75% vaginal metronidazole gel intravaginally once a day for 5 days or 5 g of 2% clindamycin cream intravaginally at bedtime for 7 days are also both effective

. Herpes genitalis TEST

1. Urine hCG 2. Lesion: HSV PCR assay of lesion with differentiation between HSV-1 and HSV-2 (most sensitive) or viral culture for herpes 3. STI testing for GC/CT and BV

a. Genital wartsTESTING

1. Usually just visual inspection 2. If no resolution after treatment or worsening with treatment, consider biopsy 3. Urine hCG

HERPES i. General measures

1. Warm, wet soaks to the genital area using vinegar/water; gently pat dry perineal area (do not rub) 2. No feminine hygiene products or douching, no feminine pads; if concerned about discharge, have extra underwear available 3. Sleep without underwear or clothing in the genital area 4. Wear loose-fitting all-cotton clothing and underwear 5. No intercourse while the lesions are active or any symptoms are present 6. If symptoms are severe and urination is affected, may need to use topical xylocaine before urination or may consider intermittent catheterization for comfort 7. OTC pain medications such as NSAIDs or acetaminophen 8. Use condoms for all sexual encounters after symptoms have resolved; this may decrease the risk of transmission 9. Advise patients that the virus can spread even without the symptoms being present and that asymptomatic spread is more common in the first year of infection

a. GERD

1. eliminate dietary triggers; 2. refrain from eating large and late meals; 3. avoid the recumbent position, especially after meals; and use an extra pillow to elevate the head when sleeping 4. Antacids can be helpful and should be taken 1 to 3 hours after meals and at bedtime. ii. Those containing bismuth or bicarbonate should be avoided because of possible fetal toxicity. iii. Sucralfate (aluminum sucrose sulfate) is a surface-binding agent useful in pregnancy for symptom relief because it is poorly absorbed and has no apparent fetal toxicity. iv. An H2 receptor antagonist (cimetidine) or a proton pump inhibitor (omeprazole) is indicated if there is no response to the above-described measures. Both appear to be safe for the fetus, but proton pump inhibitors are not advised in breastfeeding women, because of lack of safety data.

a. lesbian women

1. lesbian women may have increased rates of ovarian and breast cancer

i. Yeast

1. the most appropriate first-line treatment for vaginal yeast infections in pregnancy is a 7-day topical vaginal azole cream,

. VTE in pregnancy.

1a. Dull ache, tingling, tightness, or pain in the calf or leg, especially when walking, may be present. Acute swelling and pain in the thigh area, as well as tenderness in the femoral triangle, are suggestive of iliofemoral thrombosis. DVT in pregnancy is usually in the left leg i. US ii. MRI iii. Dimer b. Both forms of heparin are safe for the fetus and do not cross the placenta. Unfractionated heparin is associated with a higher risk of maternal thrombocytopenia and osteoporosis than is low-molecular-weight heparin. c. Warfarin is a vitamin K antagonist that crosses the placenta, carries risks of fetal hemorrhage and teratogenesis, and, with few exceptions, should be used only in the postpartum period. Breastfeeding is not a contraindicationto the use of warfarin, low-molecular-weight heparin, or unfractionated heparin

A patient in her last trimester of pregnancy is being seen by an advanced practice registered nurse (APRN) for cold symptoms. The patient discusses the over-the-counter (OTC) medications she uses at home. The APRN notes that the patient has a history of mild hypertension that began two months earlier and decides to provide education on which OTC should be avoided during pregnancy. Which OTC medication should be of concern under these conditions? Pseudoephedrine Guaifenesin Loratadine Dextromethorphan

A

A patient speaks to an advanced practice registered nurse (APRN) about a sexual assault by a same-sex partner that took place one week earlier. During the exam, the APRN notes that the patient appears calm, but the patient mentions feeling guilt, shame, anxiety, and disbelief about the incident and having difficulty sleeping. Which phase of psychological sequelae of sexual assault should the APRN assign to this patient's response? Acute phase Resolution phase Acceptance phase Integration phase

A

The FNP is seeing a patient who just returned from a mission trip to Nicaragua. The patient is experiencing mild abdominal cramping and diarrhea. Which of the following antibiotics would the FNP prescribe to this patient? Cipro Augmentin Doxycycline Biaxin

A-Feedback: For patients with moderate to severe travelers diarrhea, empiric antibiotic treatment with a quinolone antibiotic may be given.

fetal aneuploidy.

Abnormal number of chromosomes.

21 An advanced practice registered nurse (APRN) examines a patient who is four-weeks postpartum. The patient is complaining of low-grade fever, chills, and a right breast that is red, indurated, and painful. Which action should the APRN take to assist this patient? Recommend aspirin with limited breastfeeding to the right side Prescribe dicloxacillin 500 mg orally every six hours for 7-10 days Stop lactation immediately and collect blood cultures Order penicillin G 5 million units IM and then an oral ampicillin for 10 days 22

B

3 A 67-year-old woman asks the advanced practice registered nurse (APRN) when her next Pap smear should be performed. The APRN notes that the client has had three consecutive negative cytology results. Which schedule should the APRN recommend for this patient? Repeat cervical cytology in five years Ongoing cervical cytology is not necessary Repeat cervical cytology in one year Repeat cervical cytology in three years

B

13 A 27-year-old female presents to the clinic. The advanced practice registered nurse (APRN) is told about the following symptoms:· intense vulvar and vaginal itching thick, creamy vaginal discharge vulvar erythema Which diagnosis should the APRN make for this patient? Bacterial vaginosis Pelvic inflammatory disease Vulvovaginal candidiasis Trichomoniasis vaginitis

C

20 A six-week postpartum, lactating mother presents to the clinic with complaints of headaches. The advanced practice registered nurse (APRN) notes that the patient has poor posture while nursing her infant and suggests using a support pillow and starting an analgesic. Which analgesic should the APRN prescribe for this patient? Oxycodone Aspirin Acetaminophen Tramadol

C

37 An advanced practice registered nurse (APRN) is examining a 70-year-old patient for complaints of fatigue, constipation, and dentures that are suddenly not fitting well. The APRN suspects dehydration. Which diagnostic tool will confirm this diagnosis? Lipid profile Abdominal X-ray Complete metabolic panel Urine culture and sensitivity

C

6 A 26-year-old female patient presents to the clinic with complaints of abdominal pain, vaginal discharge, and vaginal pruritis. The advanced practice registered nurse (APRN) performs a pelvic exam, including pH tape test and wet mount. The APRN notes a vaginal pH of 6.2 and visualizes ovoid, flagellated organisms under the microscopy. What should the APRN diagnose? Chlamydia Bacterial vaginosis Trichomoniasis Candidiasis

C

Identify the 3 purposes of the care provided to patients who have experienced sexual assault. Correct answers:

Correct answers: Provide acute medical care, Gather evidence, Transition to long-term care for psychological recovery

Quality measures for nurse practitioners are the same as quality measures for nurses. True False

Correct! Quality measures should demonstrate proficiency in skills required for the position, which is significantly different for nurse practitioners and nurses.

What are the normal cardiovascular physiologic responses to pregnancy? Increase HR, increased cardiac output, decreased blood volume, and systolic murmur Increased HR, decreased cardiac output, increased blood volume, and systolic murmur Increased HR, increased cardiac output, increased blood volume, and systolic murmur Decreased HR, increased cardiac output, increased blood volume, and diastolic murmur

C-Feedback: During pregnancy, a hyperdynamic state is caused by an increase in blood volume, which results in a slightly increased heart rate and increased cardiac output. Systolic ejection murmurs are common and are caused by increased flow across the pulmonic and aortic valves.

3.When using the "Five Wishes" approach to documenting patient preferences for end-of-life care, the provider will document which types of preferences? (Select all that apply.) A specific list of treatments the patient does not want The people designated to make care decisions for the patient The level of sedation versus alertness the patient desires How much information to give various family members OpA directive to avoid calling 911 at the time of deathtion 5

CDE-Feedback: The Five Wishes approach addresses the type of care a patient wants as a disease progresses and is less defensive than the traditional advance directive which indicates the type of care a patient does not want. Calling 911 may be done without requiring resuscitation if the patient has an appropriate advanced directive in place.

Because primary care offices operate independently from other practices, they do not need to participate in organizational quality improvement projects. True False

Correct! Although primary care offices operate independently, the quality scores of the organization as a whole will be affected by each individual office. It is essential that each practice in a health organization contributes toward system-wide improvement projects.

A woman at 10 weeks gestation presents to your office with a blood pressure of 154/88. True or False This patient meets criteria for the diagnosis of pregnancy induced hypertension. True False

Correct! Hypertension before 20 weeks gestation is considered chronic hypertension, not induced by pregnancy. Note that preeclampsia can also be superimposed on chronic hypertension.

Anal dysplasia screening is a consideration among men who have sex with men as well as women who have anal sex. Which of the following is true regarding this consideration?* 0/1 Anal dysplasia screening should be conducted on an annual basis. Universal anal dysplasia screening should be included in the care of all men who have sex with men. Practices who implement anal dysplasia screening should have a referral program. The CDC has a consensus guideline for the frequency of anal dysplasia screening.

Correct answer Practices who implement anal dysplasia screening should have a referral program.

Identify the 4 tools that the CDC recommends for screening for intimate partner violence.

Correct answers: HITS OVAT STaT WAST

Which of the following statements is true regarding gender dysphoria?* 0/1 Gender dysphoria is most common during adolescence. Gender dysphoria can be seen in children as young as age 4. The role of the primary care provider regarding gender dysphoria is to refer the child for counseling. gender identity disorder is the DSM-5 diagnosis for gender dysphoria

Correct answer Gender dysphoria can be seen in children as young as age 4.

An advanced practice registered nurse (APRN) is examining a patient who has been unresponsive to treatment for a second-degree burn on the forearm and hand that occurred three weeks ago. Which action should the APRN take first to assist this patient? Refer to a wound specialist Send for a surgical consult Refer to an orthopedic hand specialist Send to the emergency room

Correct! The patient needs to be referred to a wound specialist first. Burns that fail to heal within two to three weeks require further evaluation with a wound specialist.

15 A patient presents to an advanced practice registered nurse (APRN) for a prenatal check-up. The APRN completes an assessment and notes the following: B/P 132/78 pulse 84 respirations 22 glucose 105 mg/dL protein 9.5 g/dL creatinine 20 mg/kg/day creatinine clearance 100ml/min Which lab test is of concern? Creatinine Glucose Creatinine clearance Protein

D

27 A 66-year-old male patient comes to the clinic for the first time accompanied by a person who is introduced as his male life partner. The advanced practice registered nurse (APRN) wants to use inclusive language while obtaining the patient's history to help promote a welcoming healthcare environment. Which term should the APRN use during the interview to accomplish this goal? Spouse Roommate Friend Significant other

D

(Piskaçek sign)- PROBABLE

Early uterine enlargement tends to be in the anteroposterior diameter so that the uterus becomes globular. In addition, because of asymmetric implantation of the ovum, one cornu of the uterus may enlarge slightly

Compared to lower opioid dosages, higher opioid dosages have not been shown to decrease chronic pain more effectively and are associated with increased overdose risk. True False

False

What is an important contraindication to phosphodiesterase-5 inhibitors? SSRIs Beta blockers Nitrates . Thiazide diuretics

Feedback: Patients cannot concurrently take nitrates because of dangerous effects of synergistic vasodilation

16.Which immunization is contraindicated in pregnancy? Polio Hepatitis B MMR Tetanus

Feedback: The MMR is a live virus and is contraindicated during pregnancy. The others are inactivated bacterial or DNA-based and are safe when indicated.

3.A 21-year-old female patient presents for her first well-woman exam. She has never been sexually active. Her family history and past medical history are negative for any gynecologic diseases. Her menses occur every 28 days, lasting 5 days, with a relatively moderate flow and no significant dysmenorrhea. Her physical exam should include which tests? Pap smear Cultures for gonorrhea and chlamydia stool hemoccult baseline mammogram

Feedback: The recommended age for females to begin screening pap smears is at the age of 21, regardless of sexual activity history. STD screening is not necessary as the patient is not sexually active. Hemoccult and mammogram are not recommended for this age group of patient.

PID

I. Usually caused by gonorrhea, chlamydia, Escherichia coli, group A streptococci, Mycoplasma genitalium, Gardnerella vaginalis II. Commonly occurs between 15 and 30 yr of age

1. What is the recommended weight gain for pregnancy?

Normal pregnancy requires an increase in daily caloric intake of 300 kcal. a. Underweight <19 28-40 lbs.. b. Normal 19-25 25-35 c. Overweight 25-29.5 15-25 d. Obese ≥30.0 11-20

nabothian follicles

Plugged, distended cervical glands () may be seen on the exocervix (or ectocervix)

5.Identify the name of the screening checklist that can be used by the FNP to guide the encounter when sexual abuse is suspected. Include the Algorithm and what each letter stands for.

RADAR; Remember to ask, Ask directly and clearly, Document all information; Assess safety; Review options.

A 32-year-old woman's Pap smear test results show atypical squamous cells of undetermined significance (ASCUS). An advanced practice registered nurse (APRN) is performing a colposcopy and notes an area of sharply delineated acetowhite epithelium. Which action should this APRN take? Perform a repeat cervical cytology Perform a diagnostic conization Perform a radical trachelectomy Perform a punch biopsy

Repeat cervical cytology is not indicated. This patient has a grossly abnormal cervix as indicated by the observation of an area of sharply delineated acetowhite epithelium. Any patient with a grossly abnormal cervix should have a punch biopsy performed, regardless of the results of the Pap smear.

a. Chronic hypertension with superimposed preeclampsia

Superimposed preeclampsia is diagnosed by the detection ofnew-onset proteinuria or the development of signs of severe preeclampsia after 20 weeks' gestation in a pregnant woman with chronic hypertension.Management of superimposed preeclampsia is similar to that outlined for preeclampsia

Hegar sign-PROBABLE

Uterine consistency becomes softer, and it may be possible to palpate or to compress the connection between the cervix and fundus. This change is referred to as

. Herpes genitalis S/S

a Definition: chronic, life-long viral infection i. Many people are asymptomatic and unaware of transmission to others ii. Two types: HSV-1 (usually oral-labial) and HSV-2 (usually genital); however, either one can infect any mucous membrane; virus remains in the ganglia after infection iii. Recurrent episodes (common) are usually milder and of shorter duration a. Possible triggers A. Stress, illness, fever B. Menstrual changes, intercourse, trauma C. Fatigue, poor nutrition iv. Signs and symptoms 1. May have "flulike" illness with first outbreak along with lymphadenopathy, watery vaginal discharge, and dysuria 2. Vesicular lesions initially, followed by an ulcerated area on an erythematous base; vesicles appear "punched-out" on the skin; the surrounding skin may be spared of redness or swelling 3. Pain at site of the lesion with very little itching; prodromal symptoms may include pain or paresthesias of buttocks, thighs, legs ("boxer shorts" area) 4. Lesions usually last for about 7 to 10 days; recurrent episodes may be less severe

Eclampsia

a. i. Eclampsia is the presence of new-onset grand mal seizures in a woman with preeclampsia that cannot be attributed to other causes

HIV Screening

a. i. Testing at least once 13-64 ii. At higher risk 3 to 6 months

a. Ambiguous genitalia can present with

clitoromegaly, bifid clitoris, or midline fusion of the labioscrotal folds.

In the patient-centered medical home (PCMH) model, specialists are responsible for communicating their plan with primary care. True False

f

a. Androgen insensitivity syndrome (a form of male pseudohermaphroditism and formerly called testicular feminization) is a

genetic deficiency of androgen receptors that results in a 46,XY infant developing female external genitalia and, later in life, secondary sexual characteristics.

vulvar and vaginal cancer Prognosis

i. The overall survival rate for vulvar carcinoma is about 70%. ii. Patients with positive nodes have a 5-year survival rate of about 50%, whereas those with negative nodes have a 5-year survival rate of about 90%.

a. müllerian agenesis or Rokitansky-Küster-Hauser syndrome

i. the uterus is absent but the fallopian tubes are spared,

a. Male pseudohermaphroditism, which most commonly results from

mosaicism, may occur with varying degrees of virilization and müllerian development.

a. True hermaphroditism is rare. The affected child has

some degree of both female and male development externally and internally; dual gonadal development occurs with either a combined ovotestes or separate gonads.

a. Bacterial vaginosis s/s

· normal balance of bacteria in the vagina is disrupted, causing overgrowth of certain lactobacilli · Foul or fishy smelling vaginal discharge that increases before menses and after intercourse · Thin, gray-white ("spilled milk") vaginal discharge · Itching and burning are rare unless coinfection with another pathogen i. Presence of 3 out of 4 of the following i s diagnostic of bacterial vaginosis (BV; Amsel criteria) ii. Copious, thin, grayish-white discharge that coats the vagina iii. Vaginal pH >4.5 iv. Positive whiff test (fishy odor of vaginal discharge when 10% KOH is added to sample) v. Clue cells on wet prep (>20% more clue cells than epithelial cells) 1. If microscopy unavailable: examination findings plus positive whiff test and pH >4.5 are considered positive 2. Commercially available DNA probe for diagnosing BV is available 3. Consider STI, HIV, trichomoniasis, and Candida infection with repeated infections vi. Treatment

Which of the following statements is true regarding the cultural humility framework? Select all that apply.* 1/1 It invites respectful questions. The framework and terminology is interchangeable with cultural competence. It urges learners to be aware of the their own beliefs, values, and biases. It embraces the reality that no one person is a representative of any given culture..

ACD-Feedback Cultural competence provides a false connotation that there exists and endpoint where the learner is competent in the care of any given population, where cultural humility allows for lifelong learning

51 An advanced practice registered nurse (APRN) is discussing the treatment options available to a patient who is homeless and has been diagnosed with HIV. The patient admits that the $8,000 per month cost of the preferred treatment is not fiscally possible. Which action should the APRN take to assist this patient? Encourage the patient to cut the dose in half Refer the patient to the case management services Advise the patient to speak with the patient assistance program Delay prescribing the medication

B

A 17-year-old female is being seen in the clinic for a wellness visit. The patient tells the advanced practice registered nurse (APRN) that she is currently sexually active, has a history of three different partners, does not routinely use condoms, had a normal menstrual cycle the week before, and denies any complaints. Which testing should the APRN provide for this patient? Pelvic exam STI test Urinalysis Pregnancy test

B

A 23-year-old male is seen in the clinic with unilateral painful testicular swelling. The patient reports that he only has vaginal intercourse with women. After assessment, the advanced practice registered nurse (APRN) determines the patient has epididymitis. Which organism should the APRN suspect as the cause of this problem? Escherichia coli Chlamydia trachomatis Staphylococcus aureus Pseudomonas aeruginosa

B

A 55-year-old male is being seen by an advanced practice registered nurse (APRN) and is upset about being unable to complete the act of intercourse because an erection cannot be sustained. The patient does not take any medications and has a body mass index of 32. The APRN suspects the patient has erectile dysfunction. Which intervention should the APRN recommend at this time? Recommend bi-weekly sessions with a psychologist Advise to walk 30 minutes daily for five days a week Start an aggressive treatment of tadalafil Prescribe amitriptyline for depression

B

A large healthcare system wants to improve communication between patients, patients' families, and the healthcare system in order to reduce litigation costs by 20% by the end of the fiscal year. To reach this goal, the system has designated a single representative to communicate with patients and families immediately following each adverse event. An advanced practice registered nurse (APRN) in the quality department has been assigned to continuously monitor this project intervention. What data should the APRN monitor to determine if this goal is being met? Number of events during the fiscal year Number of spokespeople for each event Number of litigations for each event Number of stakeholders affected

B

A non-English speaking patient is seeing an advanced practice registered nurse (APRN) for chronic back pain. The patient speaks Spanish and arrived at the clinic with a bilingual spouse and child. The APRN would like to decrease the language barrier and ensure understanding. Which action should the APRN take to meet this goal? Ask the bilingual, adult spouse to assist with translation Request a professional interpreter who has healthcare experience Use a Spanish-English dictionary to ensure accurate word translations Have a clinic nurse who speaks some Spanish speak with the patient

B

A transgender patient presents to the primary clinic for an annual screening. During the examination, the advanced practice registered nurse (APRN) notes that the patient has transitioned from female to male. Which diagnostic screening tool should the APRN recommend for this patient? Human chorionic gonadotropin (hCG) levels Pap smear Digital rectal exam Prostate-specific antigen levels

B

An advanced practice registered nurse (APRN) is caring for an intrapartum patient with magnesium sulfate intravenously infusing. The APRN notes that the patient is more somnolent and has patellar reflexes +1 and a serum magnesium sulfate level of 10.8 mg/dL. Which plan of care should the APRN initiate? Increase the magnesium sulfate infusion rate Administer calcium gluconate 10 ml Administer hydralazine 5 mg IV Increase the oxytocin infusion rate

B

An advanced practice registered nurse (APRN) is seeing a 30-year-old patient who is having excessive and prolonged menses that last more than seven days. The interval spacing is typical. The examination and lab work identify no underlying pathology. Which diagnosis should guide the treatment plan for this patient? Metrorrhagia Menorrhagia Polymenorrhea Menometrorrhagia

B

10 A female patient presents to the primary care clinic complaining of lower abdominal pain, abnormal menses, and difficulty urinating. The advanced practice nurse (APRN) performed a pelvic examination that revealed cervical motion tenderness (CMT) with purulent discharge. A diagnosis of gonorrhea is confirmed. The APRN prescribes ceftriaxone 250 mg IM; however, the medication is unavailable. Which alternative treatment regimen should the APRN use for this patient? Gentamicin 240 mg IM plus azithromycin 2 g orally once Amoxicillin 3 g plus probenecid 500 mg orally Erythromycin 500 mg orally qid Metronidazole 2 g orally once

A

16 A patient who is eight weeks pregnant comes to the clinic because of vaginal bleeding. The advanced practice registered nurse (APRN) completes an assessment with the following findings: vital signs within normal limits dull ache in lower abdomen vaginal bleeding closed cervix Which procedure should the APRN add to this patient's plan of care? Ultrasonography Urine pregnancy test CT scan Serum triple screening test

A

28 An advanced practice registered nurse (APRN) is caring for a patient who is transgender and has been hospitalized for cancer treatment. The patient is using Medicaid coverage. The APRN asked the patient to complete paperwork to identify a medical decision-maker, and a best friend who is also transgender was selected. The patient mentions being concerned about the choice since the chosen person lives an alternative lifestyle and is not a family member. Which piece of information regarding this choice should the APRN share with this patient? Medicaid regulations allow patients to choose their own medical decision-makers regardless of their legal relationship to the patient. The court will need to appoint an advocate to work with any non-family member who is selected to be a medical decision-maker. Medicaid will not allow non-family members to be the medical decision-maker, so the patient should apply for Medicare instead. The patient will need to marry the friend, and then the new spouse will be able to be named the medical decision-maker.

A

29 A patient who is transgender presents to the primary care clinic with a laceration to the forearm. While suturing the laceration, the advanced practice registered nurse (APRN) discusses the most critical topic to address at that moment with the patient. Which topic should the APRN discuss with this patient? The patient's reason for the visit. The patient's level of depression. The patient's body image perception. The patient's history of drug usage.

A

39 An elderly patient with a recent history of several falls is discussing driving privileges with an advanced practice registered nurse (APRN). The patient mentions the desire to remain both mobile and independent. Which initial action should the APRN take to assist this patient? Refer to a rehabilitation facility for screening by occupational therapist Report to authorities for unsafe driving Call the family to discuss the patient's concerns Refer to registry of motor vehicles for a vision screening

A

49 A patient receiving antenatal care for a first pregnancy returns for another visit at 20-weeks gestation. The advanced practice registered nurse (APRN) notes that all diagnostic tests indicate the patient is experiencing intrauterine growth restriction and makes recommendations to improve the growth of the fetus. Which recommendation should the APRN make to this patient? Stop smoking Take aspirin 325 mg QD Increase physical activity Take heparin 10000 U bid

A

54 An advanced practice registered nurse (APRN) is evaluating a patient in the clinic. The patient complains about having diarrhea for the last five days, a fever that started the day before, and abdominal bloating. The patient mentions trying to keep hydrated but is finding that goal difficult to achieve. What is an appropriate question for the APRN to ask during the history and physical? "Have you traveled abroad recently?" "Have you exposed anyone to your infectious stool?" "Have you eaten any fried or overcooked food within the past week?" "Have you been exposed to new cats or dogs?"

A

55 An advanced practice registered nurse (APRN) is assessing a patient who has experienced mood instability for 10 years. The patient mentions noticing his feet and hands appear to be enlarged. Which specialty provider should the APRN recommend to assist this patient? Endocrinology Cardiology Psychiatry Podiatry

A

A 15-year-old female presents to the clinic with the complaint of amenorrhea. The advanced practice registered nurse (APRN) completes an assessment with the following results: breast development normal recent weight loss of 15 lb with BMI of 16.5 kg/m2 no drug use The patient reports being on the track team in high school and runs five miles every day as part of the sport. Which care plan should the APRN recommend for this patient? Review complications, refer to counseling, and refer to nutritionist Prescribe vitamin E 1000 IU qd, monitor progestin level every three months, and apply ice packs to lower abdomen Recommend frequent meals with simple carbohydrates and prescribe spironolactone 100 mg bid Prescribe medroxyprogesterone 20 mg daily and monitor FBG and GTT annually

A

A 72-year-old patient comes to the clinic for routine follow-up care with an advanced practice registered nurse (APRN). The APRN performs a comprehensive physical exam, which is negative for findings, and then completes advanced care planning services. Why did the APRN complete this planning for this patient? It enhances self-determination and quality of care. It is indicated for this patient's disease process. It is required by CMS for a patient of this age. It increases billing services for the practice.

A

An advanced practice registered nurse (APRN) is caring for a woman who is six-months pregnant. The patient is complaining of dysuria and frequency. A urine dip suggests a urinary tract infection. Which plan of care should the APRN use for this patient? Treat with an appropriate antibiotic Avoid antibiotics and wait for the final urine culture Have the patient see her ob-gyn for treatment Order a non-stress test to assess for preterm labor

A

An advanced practice registered nurse (APRN) is examining a 96-year-old patient who is complaining of an ear infection. The patient reports currently taking four pills every day. The APRN decides to respond to that information by taking a course of action recommended by the goals of geriatric care. Which action should the APRN take for this patient? Write the reason for the medication on the patient's prescription Identify the strongest antibiotic to take to prevent a super infection Add a medication to avoid a yeast infection Avoid any additional medications and wait to see if it resolves

A

An advanced practice registered nurse (APRN) is seeing a patient with preeclampsia who has recently developed grand mal seizures. Which diagnosis should the APRN make for this presenting condition? Eclampsia Gestational diabetes Preeclampsia with gestational diabetes Chronic hypertension with superimposed preeclampsia

A

An older adult has a history of taking multiple medications for gastroesophageal reflux disease. The advanced practice registered nurse (APRN) reviews the medications and decides to discontinue the patient's proton pump inhibitor. Which effect of this medication prompted the APRN to remove this prescription? Increased rate of bone fracture Increased risk for hypertension Increased rate of hypermagnesemia Increased risk for Helicobacter pylori

A

During an annual wellness exam, a patient presents with 2+ edema in the bilateral lower extremities and feet. The advanced practice registered nurse (APRN) rules out any acute causes. Which sensory change would the APRN suspect is causing this problem? Alternation in taste Decreased olfactory nerve fibers Increased intraocular pressures Reduced tactile sensation

A

During the annual wellness exam, an older adult patient reports to the advanced practice registered nurse (APRN) problems with sleep, incontinence, and anorexia. The APRN notes the patient has shin spots and recurrent yeast infections of the skin. A falls risk assessment reveals three falls in the past three months. Which condition should the APRN further evaluate for in this patient? Diabetes mellitus Hyperthyroidism Renal insufficiency Osteoarthritis

A

A 75-year-old patient asks to have all medications changed to generics because of financial concerns and a desire to not overburden the adult child who owns the home where the patient is now staying. An advanced practice registered nurse (APRN) examines the patient and notes a finding that could indicate abuse. Which of these findings would indicate abuse? Weight loss of 1 kg (2.2 lb) since exam three months before Positive vulnerable elder assessment Bruises on bilateral upper arms Diffuse alopecia

Correct! Because of increased vascular fragility in an older patient, bruising could be an indication of abuse, particularly since the bruises are on both upper arms.

To screen for intimate partner violence, a patient just needs to be asked whether it is occurring, which is better than using a screening tool. True False

Correct! Because patients may not recognize the severity of an abusive situation, screening tools have been shown to be more effective in identifying intimate partner violence.

Buprenorphine is an opioid agonist that can produce euphoria. Treating opioid use disorder with buprenorphine substitutes one addiction for another. True False

Correct! Buprenorphine is used in the long-term treatment of opioid use disorder. Because it is a partial opioid agonist, buprenorphine reduces withdrawal symptoms and does not produce euphoria in those with opioid use disorder.

Due to normal decline, drug metabolism becomes compromised with age. True or False In general, medication doses need to be higher to be effective in older patients. True False

Correct! Cardiovascular, neurocognitive, hepatic, and renal function are compromised in geriatric patients, although the extent can depend on an individual's overall health and comorbidities. Most medications need to be given in the same or smaller doses for older patients due to decreased drug metabolism and clearance.

. An advanced practice registered nurse (APRN) is assessing an older adult who is a Cambodian refugee. The APRN notes ecchymosis on the chest, upper arms, neck, and back and accurately identifies the cause of the condition. Which activity caused this condition in the patient? Coining Medicinal herb ingestion Elder abuse Cupping

Correct! Coining is meant to restore balance by rubbing a coin across the skin of the chest, upper arms, neck, and back, which can cause ecchymosis.

What is a health risk associated with prolonged sitting during long-distance travel? Yellow fever Sciatica Deep vein thrombosis Traveler's diarrhea

Correct! Deep vein thrombosis risk increases with prolonged sitting over six hours, such as on long flights.

Which of the following is part of a Medicare annual wellness visit? Education and counseling about preventive services Diabetic education and counseling Disease planning and treatment Medication refills

Correct! Developing a plan for screenings, wellness, and preventive services is the focus of the Medicare annual wellness visit, but it does not include treatments, medication refills, or education for specific diseases such as diabetes.

Patients may have secondary conditions such as pain, fatigue, obesity, and depression as a result of having a disabling condition. True False

Correct! Disabling conditions may cause multiple secondary conditions that affect differently abled persons' overall health and wellness.

There is a large body of research on protective factors that prevent substance misuse in young adults. True False

Correct! Due to limited research, little is known about the protective factors that prevent substance misuse in young adults.

Patients with intellectual impairment are not considered competent and cannot participate in their care decision-making. True False

Correct! Even if a person with an intellectual impairment is not considered competent, the person should be involved in shared decision-making.

What does the concept of rational polypharmacy refer to? Only prescribing the lowest cost medications For patients prescribed many pills, they should take only a small ration of them per day When patients have multiple chronic and comorbid conditions necessitating the prescribing of many medications The provider discussing the rationale for prescribing many medications

Correct! Rational polypharmacy refers to mindfully prescribing numerous medications needed to manage complex patients with multiple chronic diseases.

For a patient living in poverty, a nurse practitioner earns trust through his or her credentials, because credentials are tangible evidence of knowledge and achievement. True False

Correct! Relationships are key for those with limited resources, and providers must earn trust by listening to the patient and recognizing the patient as a person. For patients in poverty, credentials do not have much significance.

A 35-year-old multiparous woman presents to a clinic for an annual physical. An advanced practice registered nurse (APRN) obtains the patient's general health history and menstrual history. The patient's last pelvic exam with cytology and HPV co-testing was three years ago, which was found to be normal. Which action should the APRN include in this plan of care? Repeat cytology in one year with no need to repeat HPV co-testing Repeat cytology and HPV co-testing in five years Repeat HPV co-testing in one year with no need to repeat cytology Repeat cytology and HPV co-testing in two years

Correct! Repeat cytology and human papillomavirus (HPV) co-testing should be performed every five years. Since this was last done three years ago, it needs to be repeated in two years.

Reactions to sexual assault may include problems with sexual behavior and functioning. True False

Correct! Sexual assault may result in problems with sexual behavior and functioning.

Cognitive impairment is best detected using a routine history and physical examination. True False

Correct! Specialized cognitive assessment screening tools more accurately identify decline. Primary care clinicians may not recognize cognitive impairment when using a routine history and physical examination, and most of these patients are not diagnosed until they are at moderate to severe stages of the disease.

A patient is in a clinic for follow-up after a renal calculi extraction. The renal stones were found to be calcium-based. Which dietary modification should an advanced practice registered nurse (APRN) recommend? Increase nuts in the diet Limit coffee intake Increase fruit consumption Limit sugary drinks

Correct! Sugary drinks should be limited as they are known to increase the risk of calcium-based stones.

A holistic approach to prescribing for older patients includes an assessment of Choose...the ability to drive to the pharmacythe ability to swallow the size of the pill or tablet prescribedwhether the medications are locked up and kept in childproof containers.

Correct! Swallowing problems become more prevalent with age, and some medications should not be cut or crushed. Patients should be asked whether they are having difficulty swallowing medications.

What is the purpose of the American Geriatrics Society's Beers criteria? An alcohol intake screening assessment for geriatric patients A test for patients' knowledge of their medications Evidence-based recommendations for potentially inappropriate medication use in older adults A list of medications older adults should never be prescribed

Correct! The Beers criteria are evidence-based recommendations for safe prescribing in older adults. Though there are instances in which medications on the Beers list are appropriate for older adults, the risks and benefits should be carefully considered.

The CDC provides travel notices in three categories: Level 1 Watch, Level 2 Alert, and Level 3 Warning. True Falseisk.

Correct! The CDC travel health notices are provided in three categories based on the severity of the health r

A 65-year-old female patient is being seen by an advanced practice registered nurse (APRN) for a routine visit. The APRN focuses on the health maintenance tasks appropriate for the patient's age. Which task should the APRN complete for this patient? Order an annual Pap smear Administer a Prevnar 13 vaccine Administer haemophilus influenzae type B vaccine Order an initial screening colonoscopy

Correct! The Prevnar 13 vaccine is used to prevent infection caused by pneumococcal bacteria and is recommended at age 65.

Patients with Medicaid and private insurances always have reasonable medication co-pays. True False

Correct! The cost of medications may still be too much for Medicaid patients with high spend down amounts and patients with private insurance who have high co-pays.

When working in a specialty practice, it is acceptable to manage diagnoses that do not pertain to your specialty. True False

Correct! The expectation for nurse practitioners working in a specialty practice is that they will only treat the conditions for which they have been consulted.

You are working as a nurse practitioner in a dermatology office. A patient presents to you with a blood pressure of 172 /100. True or False It is permissible to prescribe an antihypertensive because you are licensed to do so. True False

Correct! The expectation for nurse practitioners working in a specialty practice is that they will only treat the conditions for which they have been consulted. Additionally, your credentialing with a specialty practice may not allow you to prescribe certain medications that you would be able to prescribe in other practice settings. Essential Vide

The nurse practitioner workforce continues to respond to changing needs in the healthcare market, including a need for more providers in specialty practices. True or False Family nurse practitioners can manage many chronic conditions in an outpatient setting. True False .

Correct! The family nurse practitioner's focus includes treatment of long-term chronic conditions, which can occur in a variety of settings

Minimally disruptive medicine is an approach to providing care for patients with complex needs. True or False The goal of minimally disruptive medicine is to focus on the patient's goals for his or her health and life while lessening treatment burden. True False .

Correct! The goals of minimally disruptive medicine are to improve a complex patient's quality of life, focus on individual health goals, and minimize treatment burden

Through motivational interviewing, you can directly persuade a patient to give up substance abuse. True False

Correct! The motivation to quit comes from the patient, not outside persuasion. With motivational interviewing, you guide the patient to recognizing and resolving ambivalence.

What are the attributes of the patient-centered medical home? Efficiency, time management, quality improvement, collaboration, and cost reduction Comprehensive care, cost reduction, patient safety, and quality improvement Comprehensive care; patient-centered, coordinated care; accessible services; quality; and safety Planned care, quality improvement, and team-based care

Correct! The patient-centered medical home model comprises the following five attributes: comprehensive care; patient-centered, coordinated care; accessible services; quality; and safety.

The patient-centered medical home requires primary care practices to have a systematic focus on QI and safety. True False

Correct! The patient-centered medical home requires primary care practices to have a systematic focus on QI and safety.

True or False The most common type of vulvar cancers are squamous cell carcinomas. True False

Correct! The predominant histology of lower genital tract carcinomas (including the cervix) is squamous cell. Human papillomavirus plays a role in the development of squamous cell cancer.

An advanced practice registered nurse (APRN) examines a patient with unexplained infertility. Lab results reveal a progesterone level of <5 ng/mL. Which treatment should the APRN prescribe for this cause of infertility? Human chorionic gonadotropin hormone 10,000U IM; single-dose once a day using prefilled syringe Danazol 800 mg tablets #30/5 refills; one tablet daily starting on the first day of menstrual cycle Doxycycline 100 mg tablets #20; one tablet twice daily for 10 days Clomiphene citrate 50 mg tablets #5; one tablet daily for five days starting on day three of the menstrual cycle

Correct! The progesterone level indicates a problem with luteal function. Clomiphene citrate is a fertility drug to correct any luteal insufficiency in women with unexplained infertility. Clomiphene is usually started on day 3, 4, or 5 of the menstrual cycle at a dose of 50 mg (one pill) once daily for five days. The first day of bleeding is called cycle day 1.

Routine physical exams and Medicare annual wellness visits are the same. True False

Correct! The purpose of a Medicare annual wellness visit is to provide a personalized plan of preventative services that does not include a physical exam.

What is the FDA-approved age range for females to receive the quadrivalent HPV vaccine? Ages 10 to 25 Ages 9 to 26 Ages 12 to 21 Ages 11 to 12

Correct! The quadrivalent HPV vaccine protects against HPV types 6, 11, 16, and 18 and is approved by the FDA for females aged 9 through 26 years.

What is the recommended interval for Pap smear screenings for a 26-year-old woman? Every five years with HPV co-testing Yearly Every three years Pap smears are not recommended for this age

Correct! The recommended interval for Pap smear screenings is every three years in women age 21-29. Women age 30-65 can continue Pap smears every three years, have HPV testing alone, or have HPV testing with a Pap every five years. Women under 21 and over 65 are no longer recommended to have a Pap smear.

An advanced practice registered nurse (APRN) is preparing to discharge a patient from the hospital who was originally admitted for tuberculosis and initiation of therapy. The patient is homeless and unable to pay for the treatment. Which action should the APRN take to assist this patient? Provide the patient with contact information for the drug company Arrange for the health department to provide the medication Order QuantiFERON-TB for the patient prior to discharge Provide the medication to the patient at no cost

Correct! There is an option to arrange for the health department to provide the medication. This will increase the likelihood of the patient continuing the treatment. The patient can get the medication free of charge, and the health department can deliver it to wherever the patient considers home, even if the patient is homeless.

According to the best practices in urgent care developed by The Joint Commission Journal on Quality and Patient Safety, a summary of clinical visit information should be sent to the primary provider upon visit completion. True False

Correct! To enhance safe care transitions, all patients should have a summary of clinical visit information sent to their primary provider.

Documented and undocumented immigrants receive healthcare with the same standards of care as U.S. citizens. True False

Correct! U.S. citizens receive healthcare at a much higher standard than that of documented and undocumented immigrants.

As part of routine women's care, you ask every eligible woman about mammogram screenings; however, your quality measure shows that you only complete mammogram counseling 10 percent of the time. True or False It is not necessary to focus on improving this percentage because you know that you actually do perform mammogram counseling. True False

Correct! You should look for a cause of the inaccuracy of the quality measures and why you are not getting credit for your work.

A diabetic patient presents to your office for preoperative clearance. The surgical clearance paperwork does not specifically ask for a glucose level or A1c result. True or False To clear the patient for surgery, you must have a recent lab result indicating good control of the patient's glucose. True False

Correct! You, as the provider, must determine which labs are pertinent and essential based on individual patients' needs.

A 28-year-old woman presents to clinic with complaints of painful vulvar ulcerations. After further evaluation, the advanced practice registered nurse (APRN) diagnoses the client with herpes simplex virus. Which education should the APRN provide to the patient for this condition? Episodic therapy should be started whenever the lesions appear. The virus is spread when symptoms are present. Vesicular lesions will last for two to three days. Daily suppressive therapy should be used for 10 or more occurrences per year.

D

A 32-year-old LGBTQ patient is speaking with an advanced practice registered nurse (APRN) and mentions a concern that is universal for all genders and sexual identities. Which concern was mentioned by this patient? Hormone therapy Breast cancer screening HPV Diet and exercise

D

A patient presents to an advanced practice registered nurse (APRN) for lower-back pain that has been returning intermittently for the past year. Corticosteroids have been tried and optimized with no relief. The APRN notes that previous lumbar cervical spine X-rays and an MRI suggest radiculopathy arthritis. Which action should the APRN take to assist this patient? Schedule for outpatient mental health counseling Schedule reassessment within six to eight weeks Prescribe a long-acting opioid using the maximum dose Prescribe a short-acting opioid using the lowest dose

D

A patient with a history of an acute adrenal insufficiency presents to the clinic. The advanced practice registered nurse (APRN) assesses that the patient has increased edema and weight gain from the corticosteroid therapy. Which action should the APRN take next for this patient? Order an ACTH stimulation test Discontinue the corticosteroid Schedule a CT of the adrenal glands Consult with an endocrinologist

D

An adult patient with a long history of alcohol abuse is being evaluated by the advanced practice registered nurse (APRN) for chronic lower-back pain. Which action the APRN take? Encourage use of acetaminophen Prescribe meperidine 100 mg q3h, as needed Recommend calcium intake of 2000 IU QD Encourage use of naproxen sodium

D

An advanced practice registered nurse (APRN) is conducting a follow-up appointment with an older patient who has recently been diagnosed with depression, constipation, and back pain. The APRN reviews the patient's medications and notes prescriptions for pain management on the list. Which recommendation should the APRN make to this patient? Drink at least five glasses of water per day and switch to lower fat foods Decrease fruit juices and soda intake Encourage the patient to follow a low residue diet and keep a food diary Modify diet to include more fiber and increase fluid intake

D

An advanced practice registered nurse (APRN) is conducting the physical portion of a Medicare wellness exam. The APRN determines that it is necessary to measure the patient's gait and observe the patient walking and performing maneuvers. Which assessment should the APRN perform to obtain this information? ARE BADLs FRAIL TUG

D

An advanced practice registered nurse (APRN) is seeing a 28-year-old male who is presenting with fever, chills, malaise, myalgia, and perineal pain. The patient reports urgency with urination. Upon examination, the prostate is tender, warm, boggy, and irregular on palpation. The urinalysis indicates WBCs > 10/hpf and positive bacteria. Which condition should be ruled out before the APRN determines a management plan for this patient? Viral infection of the prostate Testicular torsion Benign prostatic hypertrophy Sexually transmitted infection

D

An advanced practice registered nurse (APRN) is seeing a patient who states they are transgender and are having serious issues related to stress at work. The APRN decides to refer the patient to a psychologist and needs to determine the correct medical diagnostic terminology recommended by the Diagnostic and Statistical Manual of Mental Disorders (5th edition) to use during the process. Which medical diagnostic terminology should be used for this patient? Gender identity disorder Gender role conformity Gender expression Gender dysphoria

D

An advanced practice registered nurse (APRN) is treating a female patient with bacterial vaginitis. Which action should be included in the plan of care? Return in one week if symptoms persist Clotrimazole 1% cream 5 g intravaginally for 7 to 14 days Treatment of the sexual partner is recommended within seven days Metronidazole 500 mg orally twice a day for seven days

D

The advanced practice registered nurse (APRN) is performing a bimanual pelvic examination on a 30-year-old female patient. The APRN will be checking the patient's cervix for cervical motion tenderness and explains this to the patient. Which condition is the APRN evaluating in this patient? Enlargement of the uterus Adnexal cysts or masses Lesions of the vaginal walls Pelvic infection or inflammation

D

Health disparities that occur disproportionately in gay men include all of the following except:* 1/1 depression eating disorders intimate partner violence access to education

Feedback Gay men suffer a disproportionate burden of mental health, substance abuse, and intimate partner violence.

Which of the following statements is true regarding implicit bias?* 1/1 Implicit biases develop during adulthood. Implicit biases are not conflicting with one's own personal beliefs. Knowledge of one's own implicit biases allows for awareness of how biases can affect the care they provide. Implicit biases are conscious and intentional responses to people or situations.

Feedback Implicit biases form during childhood in response to prevailing family and societal values. It is common for implicit biases to be in conflict with stated values and beliefs. Implicit

Which of the following is a health disparity that is unique to transgender people?* 1/1 access to quality education housing instability higher burden of depression substance abuse

Feedback In a recent study of transgender health, 69% reported being homeless at least once.

A 21-year-old female patient presents for her annual wellness exam. Her sexual history reveals all female partners, and no history of penetrative intercourse. The FNP is aware that:* 1/1 This patient does not meet the USPSTF guidelines for cervical cancer screening because she has never had penetrative sex. This patient should have a pap smear for cervical screening, according to the USPSTF guidelines. Health promotion and screening should be based upon a person's sexual orientation. The incidence of STIs and HPV is lower in women who sleep with women.

Feedback It is recommended that all women begin cervical cancer screening at age 21, regardless of sexual activity. Health promotion and screening activities are related to behavior regardless of sexual orientation. There is clear evidence that HPV, HSV, and syphilis can be transmitted between female partners.

The FNP is providing care to a newborn who has intersex genitalia. What is the urgent clinical concern?* 1/1 Ensuring that the parents have adequate support. Referring the newborn for genital surgery. Ruling out congenital adrenal hyperplasia. Honoring the parent's wishes and beliefs related to decision making.

Feedback The urgent clinical concern for a provider caring for a newborn with intersex genitalia is to rule out congenital adrenal hyperplasia. This is the most common cause of atypical genitalia and can be life-threatening.

An example of work processes that impact sexual and gender minority is:* 1/1 clearly posted non-discrimination statements access to all-gender restrooms inclusive intake forms and interviewing tools use of normalizing statements

Feedback Work processes that impact sexual and gender minority people are typically intake forms. The other answers are related to other practice-level strategies to improve the care of sexual and gender minority people.

9.What finding is considered a normal surface characteristic of the cervix? Small, yellow, raised round area on the cervix. Red patches with occasional white spots. Friable, bleeding tissue at the opening of the cervical os. Irregular granular surface with red patches.

Feedback: A nabothian cyst is a small, white or yellow, raised round area on the cervix and is considered to be a normal variant.

The FNP is screening an 84-yer-old female patient for nutritional deficits using the MNA-SF tool. The FNP understands that a score of 12 on the screening tool indicates: Malnutrition Risk for malnutrition Normal nutritional status

Feedback: A score of 12-14 on the MNA-SF indicates normal nutritional status.

.Ten days after delivery, a patient is diagnosed with mastitis. Which of the following should the FNP expect to find on physical exam? Tender, hard, hot, reddened area on the breast. . Dimpled skin on breasts and firm nodules around the areola. Decreased milk production, inverted nipples, and firm, inflamed breast tissue. Soft, tender palpable masses with cracked bleeding nipples.

Feedback: A tender, hard, hot and reddened area on the breast over the affected area is typically found with mastitis. The patient with mastitis can also be febrile with flu-like symptoms

The patient comes in for her first prenatal visit. She is healthy and has no history that would contribute to complications to the pregnancy. She asks the FNP what she can take for occasional headaches caused by eyestrain and allergies. Which of the following would be appropriate for the FNP to recommend? Ibuprofen 200mg Q4-6 hours, not to exceed 600mg in 24 hours Naproxen 220mg Q 8-12 hours Aspirin 60mg Q6 hours, not to exceed 300mg in 24 hours Acetaminophen 650mg Q4-6 hours, not to exceed 4000mg in 24 hours

Feedback: Acetaminophen is a risk category drug B, problems have not been documented. Aspirin is a risk category D. NSAIDs are also a risk category B, but have been associated with prolonging pregnancy and prematurely closing the fetal ductus arteriosus because of antiprostaglandin effects.

20.A pregnant patient in the last trimester complains of a constant back ache aggravated by walking, moving, and bending. The pain does not radiate to either leg. In addition to rest, massage, and physiotherapy, which of the following medications is approved? Acetaminophen Codeine Naproxen Aspirin

Feedback: Acetaminophen is the best option for this patient. Naproxen and Aspirin are contraindicated during pregnancy. Codeine, although used at times during pregnancy, is not indicated for this patient.

Correct 4.A young adult patient presents with a history of vaginal itching and heavy white discharge. The patient denies a history of sexual activity. On exam, the FNP finds a red, edematous vulva and white patches on the vaginal walls. The discharge has no odor. What finding would the FNP suspect in the patient's history? vegetarian diet recent diarrhea early menopause recent antibiotic use

Feedback: Almost half of all vaginal infections are caused by candida. The majority of women who develop this infection have recently taken antibiotics.

1.A 45-year-old male presents with a several-week history of scrotal pain that worsens with coughing, lifting and straining. He states that his scrotum feels "full" at the end of the day. What would the FNP be suspicious of? Spermatocele Inguinal hernia Epididymitis Testicular torsion

Feedback: An inguinal hernia presents with the stated symptoms, along with a bulge in either the groin or scrotum that becomes more obvious when standing.

Which patient should have pulmonary function testing as part of the presurgical exam? A patient older than 60 years of age A patient with a history of pneumonia in the last 2 years A patient undergoing major intrathoracic surgery A patient with diabetes and morbid obesity

Feedback: Any patient undergoing major thoracic surgery should have pulmonary function testing. Age over 60 years, a history of pneumonia, and diabetes and obesity do not require pulmonary function testing unless there is comorbid COPD.

A 71-year-old male patient presents for his annual wellness exam, accompanied by his son. The FNP understands that which of the following factors is the least important when screening for cognitive decline: Family or caregiver concerns Patient history Clinician observations Age

Feedback: Evaluation of cognition is based upon an algorithm developed by the Alzheimer's association that is based upon patient history, concerns of family or caregivers, and clinical observations. The incidence of dementia increases with age. According to the AAFP, prevalence increases by 5% in persons aged 71-79. Although age is a factor, it is not the most important factor for this patient.

Strategies for achieving minimally disruptive medicine include all of the following, except: Goal setting Shared decision-making Increasing the number of provider visits Collaboration with community and therapy

Feedback: Examples of strategies for achieving MDM include goal setting, shared decision making, de-prescribing, streamlining of appointment schedules and prescription refills), and enhancement collaboration with community entities for material support; physical, psychological, and occupational therapy; palliative care

Finasteride is prescribed for a 50-year-old man who is experiencing a problem with urination secondary to an enlarged prostate. The FNP would teach the patient that while he is taking this medication it is important to: Increase fluid intake Restrain from sexual activity Take special precautions around women of childbearing age Increase intake of folic acid

Feedback: Finasteride has some risk to women of childbearing age. It is important that women of childbearing age are not exposed to the sperm of a patient taking finasteride. These women and pregnant women should avoid handling crushed tablets. Exposure can cause fetal anomalies.

A young woman presents with complaints of irritation in the vaginal area. This is the first time this has occurred. On exam, the cervix is inflamed and friable. Flagellated protozoa are seen on the wet mount. What is the most likely diagnosis? trichomoniasis cervicitis chlamydial infection bacterial vaginosis

Feedback: Flagellated protozoan confirms the diagnosis of trichomoniasis.

A 65-year-old male presents with a history of well-controlled hypertension and diabetes mellitus. He is a non-smoker. He has been married for 35 years and is monogamous with his partner. He reports that his relationship with his wife is good. He complains of new-onset erectile dysfunction (ED). What is the first line therapy for ED? Relationship counseling Oral phosphodiesterase-5 inhibitors Intraurethral injections of alprostadil Use of a vacuum device

Feedback: Oral phosphodiesterase-5 inhibitors are safe, effective, and a reasonable choice for first line therapy for ED. This class includes commonly known medications such as sildenafil, vardenafil, tadalafil, and avanafil. Patients should be educated on proper use and potential side effects prior to use.

7.A patient who is planning international travel to a developing country asks the provider about vaccinations. Which is true about pretravel vaccines? Requirements should be reviewed at least 4 to 6 weeks prior to travel. There are at least five required vaccines for entry into certain countries. Malaria vaccine is the most important vaccine for worldwide travel. Country-specific guidelines are provided by individual embassies.

Feedback: Patients seeking immunizations prior to international travel should have these reviewed at least 4 to 6 weeks prior so that antibody responses and completion of vaccine series may occur. Country-specific guidelines may be found on the CDC website. Malaria is not prevented by vaccine, but by prophylactic antimalarial drugs. There are only two vaccines that are required.

Which is the most appropriate research design for a Level III research study? Randomized clinical trials Epidemiological studies Qualitative studies Experimental design

Feedback: The experimental design is the most appropriate design for a Level III study. Epidemiological studies are appropriate for Level II studies. Qualitative designs are useful for Level I studies. Randomized clinical trials are used for Level IV studies.

he four-stage balance test is used to assess a patient's risk of falls. Which elements would patients perform as part of this screening tool? Stand from a seated position, walk 10 feet, turn, walk back, and sit down Hold each position for 10 seconds: standing with legs side by side, then place the instep of one foot touching the big toe of the other foot, next standing heel to toe, and finally standing on one foot Reach forward beyond arm's length while maintaining a fixed base of support in four positions without losing balance

Feedback: The four-stage balance test has the patient stand in four different positions that get progressively harder to maintain. Lower scores indicate balance issues and increased risk of falls. Cross their arms over their chest and repeatedly stand up as many times as they can in 30 seconds

13.The FNP is talking with a young woman who has been diagnosed with herpes simplex type 2. In discussing her care, it would be important for the FNP to include what information? The initial lesions are usually worse than lesions that occur in outbreaks at later time. Her sexual partner will not contract the virus if she does not have sex when the lesions are present. This condition can be treated and cured if she takes all of the antibiotics for two weeks. If she becomes pregnant in the future, she will need to have a cesarean section.

Feedback: The initial outbreak is usually the worst. HSV can be transmitted even when lesions are not present, there is no cure. Vaginal delivery is allowed if no lesions are present at the time of labor.

6.The recommended office visit interval for a low-risk patient at 28 weeks of pregnancy is every: 4 weeks until 36 weeks week for the remainder of the pregnancy 2 weeks until 36 weeks 6 weeks until 38 weeks

Feedback: The interval for prenatal visits for low-risk women is every 4 weeks until 28 weeks, every 2 weeks from 28-36 weeks, and weekly after 36 weeks.

1. Acute fatty liver of pregnancy

a. extremely serious complication that can occur in the third trimester of pregnancy. It is associated with diffuse microvesicular fatty infiltration of the liver, resulting in hepatic failure. b. may in some instances result from an inborn error of metabolism, possibly a deficiency of long-chain 3-hydroxyacyl-coenzyme A dehydrogenase (LCHAD). c. S/S i. with abdominal pain, nausea and vomiting, jaundice, and increased irritability. Hypertension and proteinuria are present in approximately 50% of patients, raising the issue of coexisting preeclampsia. Patients can develop coagulopathy with intraabdominal hemorrhage, hepatic coma, and renal failure. ii. Hematemesis and spontaneous bleeding become manifest as DIC develops. Liver failure is indicated by elevated blood ammonia levels. d. Treatment i. Prompt delivery and intensive supportive care are indicated after diagnosis. Treatment is directed mainly at providing supportive ii. intravenous fluids with 10% glucose to prevent dehydration and severe hypoglycemia. iii. For the coagulopathy of hepatic failure, vitamin K supplementation is not effective, and fresh-frozen plasma or cryoprecipitate should be given along with platelets and packed red blood cells if there is DIC.

1. Discuss the incidence and definition of gestational diabetes.

a. glucose intolerance with onset or first recognition during pregnancy. Rising levels of human placental lactogen, progesterone, prolactin, and cortisol in pregnancy are some of the primary factors associated with progressive insulin resistance during pregnancy. 10%

1. What is the APRN consensus model for?

a. provides guidance for states to adopt uniformity in the regulation of APRN roles, licensure, accreditation, certification and education. b. The COVID-19 pandemic has prompted some state governors to temporarily remove unnecessary restrictions on APRNs. Lifting these barriers which do not follow the Consensus Model allowed APRNs to practice across state lines to meet the need for health care providers in surge areas. As long as regulatory requirements differ from state to state, each border represents an obstacle to license portability - potentially preventing access to APRN professionals and the high-quality care they provide. c. Adoption of the Consensus Model moves states toward uniformity and license portability.

1. What are the mechanisms of transfer from mother to fetus across the placenta?

a. simple diffusion, facilitated diffusion, and active transport. b. Low molecular size and lipid solubility promote simple diffusion c. Glucose is the main energy substrate of the fetus, although amino acids and lactate may contribute up to 25% of fetal oxygen consumption.

Which assessments of care providers are performed as part of the value-based purchasing (VBP) initiative? (Select all that apply.) Monitoring mortality rates of all patients with pneumonia Appraising costs per case of care for Medicare patients Evaluating available evidence to guide clinical care guidelines Requiring advanced IT standards and minimum cash reserves Other

a/b-Feedback: Value-based purchasing looks at five domain areas of processes of care, including efficiency of care (cost per case), experience of care (patient satisfaction measures), and outcomes of care (mortality rates for certain conditions). Evaluation of evidence to guide clinical care is part of evidence-based practice. The requirements for IT standards and financial status are part of Accountable Care Organization standards.

breast disorders i. Pathologic causes

are usually unilateral, persistent, spontaneous, and localized to a single duct noted on nipple: 1. Cancer (e.g., unilateral discharge, usually clear to bloody color) 2. Pituitary lesion causing hyperprolactinemia 3. Severe head trauma, extensive thoracic surgery 4. Mammary duct ectasia (e.g., sticky green-black discharge)

To reduce adverse events associated with care transitions, the Centers for Medicare and Medicaid Service have implemented which policy? Requirements for written discharge instructions for patients and caregiver Reduction of payments for patients readmitted within 30 days after discharge Mandates for communication among primary caregivers and hospitalists Penalties for failure to perform medication reconciliations at time of discharge

b-Feedback: As a component of the Affordable Care Act, the Centers for Medicare and Medicaid Service developed the Readmissions Reduction Program reducing payments for certain patients readmitted within 30 days of discharge. The CMS did not mandate communication, institute penalties for failure to perform medication reconciliations, or require written discharge instructions.

What is the purpose of Level II research? To demonstrate the effectiveness of an intervention or treatment To describe relationships among characteristics or variables. To evaluate the nature of relationships between two variable To define characteristics of interest of groups of patients

b-Feedback: Level II research is concerned with describing the relationships among characteristics or variables. Level I research is conducted to define the characteristics of groups of patients. Level II research evaluates the nature of the relationships between variables. Level IV research is conducted to demonstrate the effectiveness of interventions or treatments

Which is the least important barrier to collaborative advanced nursing practice? Prescriptive authority Reimbursement privileges Legal scope of practice Political activism

b-Feedback: Three major issues are central to the expansion of the the FNP role: prescriptive authority, reimbursement privileges, and legal scope of practice.

Scrotal-inguinal hernia

a. i. A segment of the bowel slips through the internal inguinal ring, where it may remain in the inguinal canal or pass into the scrotal sac. An inguinal hernia may occur as a result of a defect in the anterior abdominal wall or because of a patent process vaginalis ii. A hernia may move freely between the abdomen and the scrotum or can be spontaneously reduced by digital manipulation. 1. When a hernia becomes strangulated or is unreducible, this compromises the blood supply and requires emergent surgical reduction. a. Strangulation should be suspected when a tender mass is palpated in the scrotum in addition to redness, nausea, and vomiting iii. Treatment 1. if the herniated bowel is reducible, surgical referral for possible future repair is indicated. Difficulty in reducing a hernia is cause for urgent surgical intervention

Hydrocele

a. i. Accumulation of fluid within the tunica vaginalis surrounding the testicle 1. result from a patent processus vaginalis at birth and sometimes closes spontaneously within the first 1 to 2 years of life. 2. Hydroceles are the most common cause of painless scrotal swelling; in adults they are often the result of trauma, a hernia, testicular tumor, or torsion or a complication of epididymitis ii. Treatment 1. Congenital resolves on its own. Watchful waiting asymptomatic or excision if pain or increasing in size.

Estrogens

a. i. Cholesterol is converted to pregnenolone in the placenta. This precursor is converted into DHEA-S largely in the fetal, and to a lesser extent the maternal, adrenals. ii. Estriol (E3), the most abundant estrogen in human pregnancy, is synthesized in the placenta from 16α-hydroxy-DHEA-S, which is produced in the fetal liver from adrenal DHEA-S iii. A sudden decline of estriol in the maternal circulation may indicate fetal compromise in a neurologically intact fetus.

. Chronic IBD

a i. Chrons or ulcerative colitis UC worse disease in pregnancy 1. Use of supplemental folic acid is advised, and if deficiencies are present, supplemental iron and vitamin B12 should be prescribed. 2. The best pregnancy outcomes occur in those patients who are in remission at the time of conception and whose disease activity can be controlled with medication that has minimal fetal toxicity. 3. Major teratogens such as methotrexate must be discontinued before conception. Overall, pregnancy in women with IBD is complicated by higher rates of preterm birth and IUGR, 4. Sulfasalazine does not appear to be associated with fetal toxicity. Prednisone is metabolized by placental enzymes and relatively little crosses the placenta. 5. If diarrhea is the main complaint, dietary restriction of lactose, fruits, and vegetables is necessary. 6. Surgery is indicated only for very severe complications, such as bowel perforation or abscess formation. In the absence of compelling obstetric indications, cesarean delivery is not recommended, unless there are perineal or rectal manifestations of Crohn disease

. Elephantiasis

a i. caused by a filariasis (parasitic disease) that affects the scrotum, causing massive scrotal lymphedema. 1. Filariasis is caused by threadlike roundworms, called filariae, that are transmitted by various mosquitoes, flies, and biting midges and is most often caused by Wuchereria bancrofti. ii. Treatment 1. The recommended treatment for W. bancrofti infection is diethylcarbamazine, but some research has shown response to doxycycline.34 Diethylcarbamazine is no longer sold in the United States or approved by the US Food and Drug Administration (FDA), but it can be obtained from the Centers for Disease Control and Prevention (CDC) with positive laboratory confirmation.

. Disorders of excitement

a i. neurovascular event initiated by cognitive or tactile stimulation that is processed in the brain. Chemical mediators cause the essential relaxation of tissue and perfusion of the corpora cavernosa and corpus spongiosum. Nitric oxide and cyclic guanosine monophosphate (cGMP) are the primary noncholinergic and cholinergic mediators responsible for the neurogenic aspect of erection. Engorgement of the corpora cavernosa and corpus spongiosum, in turn, compresses the veins to prevent the venous outflow of blood. This is how the erection is maintained and accounts for the vascular component of erection.

Disorders of orgasm

a. i. Caused by low testosterone levels, certain neurologic diseases, and some head and spinal cord injuries. Certain drugs, including hypertensive medications, antidepressants, anxiolytics, antipsychotics, and alcohol, can slow down the sympathetic nervous system and can also affect ejaculation. 1. The American Urological Association (AUA) and the World Health Organization (WHO) recommend limited diagnostic testing for men with ED. 2. Psychotherapy is the preferred treatment for psychogenic ED. Sexual counseling can enhance communication, ease some of the stress associated with ED, and dispel myths.

RHD risk factors

a. i. In RhD-negative patients whose anti-D antibody titers are positive (i.e., those who are RhD-sensitized), the RhD status of the father of the baby should be determined. ii. If the father is RhD-negative, the fetus will be RhD-negative and hemolytic disease will not occur, so further monitoring is unnecessary. 1. If the father is RhD-positive, his Rh genotype should be determined using quantitative polymerase chain reaction 2. first immunized pregnancy is not in serious jeopardy when the anti-D antibody titer remains below 1 : 16. I

a. A diagnostic cone biopsy of the cervix is indicated in the following circumstances:

a. i. Pap smear shows a high-grade lesion and the colposcopic examination is unsatisfactory. ii. Endocervical curettings show a high-grade lesion. iii. Pap smear shows a high-grade lesion that is not confirmed on punch biopsy. iv. Pap smear shows adenocarcinoma in situ. v. Microinvasion is present on the punch biopsy.

1. Discuss confirmation of pregnancy and determination of viability.

a. About 30-40% of all pregnant women will have some bleeding during early pregnancy (e.g., implantation bleeding), i. Test for pregnancy b. The pregnancy test detects human chorionic gonadotropin (hCG) in the serum or the urine c. The use of transvaginal ultrasonography has improved the accuracy of predicting viability in early pregnancies. i. 5 weeks' gestation or a mean hCG level of about 1500 IU/L d. The presence of a gestational sac of 8 mm (mean sac diameter) without a demonstrable yolk sac, 16 mm without a demonstrable embryo, or the absence of fetal cardiac motion in an embryo with a crown-rump length of greater than 5 mm indicates probable embryonic demise i. Doppler not recommended

1. Define the four stages of parturition.

a. Activation i. Normally, the signals for myometrial activation can come from uterine stretch as a result of fetal growth, or from activation of the fetal hypothalamic-pituitary-adrenal (HPA) axis as a result of fetal maturation ii. The concept of a role for the fetal lung in the initiation of parturition is particularly attractive because the fetal lung is the last major organ to mature. b. Stimulation i. Placental production of crh ii. involving uterine contractility, cervical ripening, and decidual/fetal membrane activation iii. For most of pregnancy, uterine quiescence is maintained by the action of progesterone. iv. PRA inhibits progesterone action. The ratio of PRA to PRB in the myometrium in labor is increased, which in effect results in a progesterone withdrawal. v. Functional progesterone withdrawal results in functional estrogen predominance, in part as a result of the increase in placental production of estrogen. vi. Cervical ripening is largely mediated by the actions of prostaglandins, uterine contractility by the actions of gap junctions and myosin light-chain kinase, and decidual/fetal membrane activation by the actions of enzymes such as metalloproteinases, which ultimately lead to rupture of the membranes. c. Involution i. During expulsion of the fetus, there is a dramatic increase in the release of maternal oxytocin which facilitates the initiation of the final phase of labor. ii. Phase 3 involves placental separation and continued uterine contractions. iii. Placental separation occurs by cleavage along the plane of the decidua basalis. iv. Uterine contraction is essential to prevent bleeding from large venous sinuses that are exposed after delivery of the placenta, and is primarily effected by oxytocin. v. This is further supported by oxytocin let down during early breast feeding.

1. Describe the physiologic changes associated with each body system during pregnancy.

a. Cardiovascular system i. The disproportionate increase in plasma volume compared with the red cell volume results in hemodilution with a decreased hematocrit reading, sometimes referred to as physiologic anemia of pregnancy. ii. diastolic pressure decreases more markedly; this reduction begins in the first trimester, iii. Pregnancy does not alter central venous pressures. iv. Pregnancy does not alter central venous pressures. v. Because of venous compression, the rate of blood flow in the lower veins is also markedly reduced, causing a predisposition to thrombosis. vi. During late pregnancy, the uterus can also partially compress the aorta and its branches. vii. Poseiro effect. femoral pulse is not palpable viii. Pregnancy requires about 1 g of elemental iron: 0.7 g for mother and 0.3 g for the placenta and fetus ix. Plasma volume expands proportionately more than red blood cell volume, leading to a fall in hematocrit. x. Hematocrit readings below 27%, or above 39%, are associated with less favorable outcomes. Despite the relatively low "optimal" hematocrit, the arteriovenous oxygen difference in pregnancy is below nonpregnant levels.

1. Discuss the management of UTI in men.

a. Causes of adult male UTIs include prostatitis, epididymitis, orchitis, pyelonephritis, cystitis, urethritis, and urinary catheters

a. Endocrine

a. Endocrine i. The thyroid gland undergoes moderate enlargement during pregnancy. ii. Placenta-derived human chorionic gonadotropin (hCG) has a TSH-effect on the thyroid gland, which can result in abnormally low levels of TSH in the first trimester, when hCG concentrations are highest. iii. TBG is increased during pregnancy because the high estrogen levels induce increased hepatic synthesis. iv. Only minimal amounts of thyroid hormone cross the placenta. it synthesizes thyroid hormone from its own thyroid gland to meet its requirements v. Adrenocorticotropic hormone (ACTH) and plasma cortisol levels are both elevated from 3 months' gestation to delivery vi. Unlike the level of thyroid hormones, the mean unbound level of cortisol is elevated in pregnancy

a. Innate and adaptive immunity

a. Innate and adaptive immunity i. The innate immune response is the first line of defense, and includes surface barriers (mucosal immunity), saliva, tears, nasal secretions, perspiration, blood and tissue monocytes/macrophages, natural killer (NK) cells, endothelial cells, polymorphonuclear neutrophils, the complement system, dendritic cells, and the normal microbial flora. ii. the adaptive immune system is composed of cell mediated (T lymphocytes) and humoral responses (B lymphocytes-antibodies). Activation of T and consequently B lymphocytes is critical for the development of lifelong immune responses. iii. B cells exposed to antigen for the first time produce IgM

1. Discuss the incidence and impact of cardiac disease during pregnancy.

a. Less than 5% of pregnancies in the United States are complicated by maternal cardiac disease b. new systolic flow murmur, an S3 gallop, an increase in resting heart rate, a decline in diastolic blood pressure, and dependent edema

1. Discuss the evaluation, manifestations, and treatment of mastitis.

a. Mastitis usually manifests with fever, generalized malaise, influenza-like symptoms, local erythema, and breast warmth and tenderness. application of warm packs to the breast and frequent breastfeeding or pumping. In addition to these nonpharmacologic interventions, the infection is treated with antibiotics such as amoxicillin-clavulanate, dicloxacillin, or a broad-spectrum cephalosporin to cover a probable staphylococcal or streptococcal infection

1. Review the medications used to treat opioid use disorder.

a. Methadone b. Buprenorphine c. Naltrexone

1. Discuss the general guidelines for surgery in pregnancy.

a. No electives b. the second trimester is the safest time. c. Precautions must be taken to avoid maternal hypotension and hypoxia that have adverse effects on uteroplacental blood flow. d. When possible, the patient should be in the left lateral decubitus position.

1. Discuss the difference between palliative care and hospice.

a. Palliative i. the part of hospice care that focuses on helping people who are terminally ill and their families maintain their quality of life. If you're terminally ill, palliative care can address your physical, intellectual, emotional, social, and spiritual needs. Palliative care supports your independence, access to information, and ability to make choices about your health care. b. Hospice i. Hospice is a program of care and support for people who are ii. terminally ill (with a life expectancy of 6 months or less, if the illness runs its normal course

1. Define the Barker hypothesis.

a. Poor placental development has been linked to preeclampsia, preterm birth, and intrauterine growth restriction (IUGR), all of which are associated with low birth weight (<2500 grams), and may play a role in fetal programming of chronic diseases later in life

1. What is the POLST program?

a. Portable medical order following in states b. POLST gives seriously ill or frail people more specific direction over their health care treatments compared to advance directives and more options than Do Not Resuscitate (DNR) orders. i. A process. Part of advance care planning, which helps you live the best life possible. ii. Conversation. A good talk with your provider about your medical condition, treatment options, and what you want. iii. A medical order form that travels with you (called a POLST form).

60 An advanced practice registered nurse (APRN) is leading a project team to ensure that during the discharge process patients are able to describe two ways home care instructions will be followed after release. The team is developing outcomes that will meet the criteria of the SMART tool. Which outcome should the APRN approve for the project? All patients will describe at least two ways they will follow the instructions prior to discharge. 80% of patients receiving home care instructions will describe at least two ways they will follow the instructions. 80% of patients receiving home care instructions will describe at least two ways they will follow the instructions prior to discharge. All patients receiving home care instructions will describe how they will follow the instructions prior to discharge.

c

.A small, rural hospital is part of an Accountable Care Organization (ACO) and is designated as a Level 1 ACO. What is part of this designation? Care coordination for chronic diseases Standards for minimum cash reserves Bonuses based on achievement of benchmarks Strict requirements for financial reporting

c-Feedback: A Level 1 ACO has the least amount of financial risk and requirements, but receives shared savings bonuses based on achievement of benchmarks for quality measures and expenditures. Care coordination and minimum cash reserves standards are part of Level 2 ACO requirements. Level 3 ACOs have strict requirements for financial reporting.

.What was an important finding of the Advisory Board survey of 2014 about primary care preferences of patients? Costs of ambulatory care The ratio of providers to patients Ease of access to care Associations with area hospitals

c-Feedback: As part of the 2014 survey, the Advisory Board learned that patients desired 24/7 access to care, walk-in settings and the ability to be seen within 30 minutes, and care that is close to home. Associations with hospitals, costs of care, and the ratio of providers to patients were not part of these results.

Which assessment question would the FNP ask when engaging in the new model for primary care? (Select all that apply.) "Are you experiencing any side effects from your newly prescribed medications?" "Do you have any questions about the lab tests that have been ordered for you?" "Are you ready to discuss some of the community resources that are available?" "Are you ready to discuss the results of your laboratory tests?" "Do you anticipate any problems with adhering to your treatment plan?"

c/d-Feedback: The nursing responsibilities in the previsit stage include assessing the patient's tolerance of prescribed medications, understanding of existing treatment plan, and education about required lab testing. The primary care provider is responsible for screening lab data and discussing community resources during the actual visit.

Manifestations

i. 1. Weight gain, edema, proteinuria, thrombocytopenia, liver vasospasm (HELP syndrome), fetal compromise from low placental perfusion IUGR, CNS effects 2. The elevation of blood pressure seen in preeclampsia (particularly the increase in diastolic pressure) is a result of generalized vasospasm and an increase in systemic vascular resistance. woman with severe preeclampsia or eclampsia whose disease presents at or beyond 34 weeks' gestation should usually be delivered after a brief period of stabilization. Severe preeclampsia presenting at less than 34 weeks' gestation may in certain situations be stabilized, and with careful monitoring of the mother and fetus, delivery may be delayed until the pregnancy reaches 34 weeks.

One of the earliest signs of fetal anemia caused by Rh alloimmunization is an elevated fetal middle cerebral artery (MCA) One of the earliest signs of fetal anemia caused by Rh alloimmunization is an elevated fetal middle cerebral artery (MCA)

i. 1. signs are an increase in the size and thickness of the placenta and fetal hepatomegaly. If the hemolysis is allowed to progress untreated, it will result in severe extramedullary hematopoiesis, portal hypertension, hypoalbuminemia, and the progressive development of in utero heart failure or hydrops fetalis

a. First-trimester screen

i. A combination of maternal age, fetal nuchal translucency (NT) thickness and maternal serum-free β-human chorionic gonadotropin (β-hCG) and pregnancy-associated plasma protein-A (PAPP-A) are included in the first-trimester screen ii. Elevated levels of free β-hCG and low levels of plasma protein-A are associated with an increased risk for Down syndrome. iii. Visualization of the nasal bone on first-trimester ultrasound has been shown to reduce the risk for Down syndrome, whereas nonvisualization (absence) has been associated with an increased risk.

a. Syphilis

i. A perinatal infection with Treponema pallidum may cause stillbirth, IUGR, nonimmune hydrops, rhinitis, hepatosplenomegaly, "mulberry molars" and "saber shins," "saddle nose deformity," and interstitial keratitis. Prenatal testing for syphilis is mandated in the United States. ii. The treatment for syphilis in pregnancy is benzathine penicillin. iii. Patients who have a positive penicillin skin test should be desensitized and treated with penicillin,

a. AGING GAMES

i. A: audiovisual ii. G: gait and mobility iii. I: insomnia iv. N: nutrition v. G: gastrointestinal vi. [GI]; G: genitourinary [GU] vii. A: assistance, ADLs/IADLs/advanced directives viii. M: mood and memory ix. E: environment + everyday activities x. S: sexuality

a. Mendelson Syndrome

i. Acute gastric aspiration is a cause of acute respiratory distress syndrome (ARDS) in adults. Treatment consists of supplemental oxygen, measures to maintain the airway, provision of ventilatory support if needed, and additional therapy for acute respiratory failure. Antibiotics are indicated if there is any suggestion of infectious pneumonitis. ii. Do not feed preg pt 6 hours before labor 2 hours drinking iii. If full stomach presumed before general anesthesia preop H2 and sodium citrate

a. nails

i. Aging nails often are yellowed, thickened, and can have longitudinal ridges ii. Brittle nails can be a sign of hypothyroidism. Any nail(s) that is/are misshapen or associated with clubbing (e.g., chronic obstructive pulmonary disease, malignancies and other illnesses), pitting (psoriasis), cuticle invasion (Lichen planus), spoonlike appearance (iron-deficiency anemia), splinter hemorrhages (bacterial endocarditis), longitudinal black or brown lines (melanoma or other cancer), and horizontal bands or ridges require further investigation and possible referral to dermatology for treatment

a. Inflammatory vaginitis

i. Desquamative inflammatory vaginitis (DIV) is a fairly rare clinical syndrome characterized by diffuse exudative vaginitis, epithelial cell exfoliation, and a profuse purulent vaginal discharge 1. ("strawberry cervix"). ii. By the time women are diagnosed, their symptoms have usually been present for years, and they have typically been treated repeatedly for a "vaginal infection" without any long-term improvement. iii. Initial therapy should be with 2% clindamycin cream, one applicator full (5 g) intravaginally daily for 7 days. If this is not effective, intravaginal 10% hydrocortisone daily for 14 days may be tried.

a. Describe the CDCs recommendation for calculating opioid prescriptions.

i. Determine opioid max a day convert to MME then add together

vulvar and vaginal cancer a. Diagnosis

i. Diagnosis requires biopsy of suspicious lesions, which can be done in the office with the patient under local anesthesia.

a. Fibrocystic breast changes TEST

i. Diagnostic testing 1. Breast examination should include location, size, smoothness, and mobility of mass 2. Diagnostic mammogram and/or U/S if indicated; consider 3D mammography (if available) 3. Refer for diagnostic needle biopsy/draining (if indicated)

TESTING-a. Gonorrhea

i. Diagnostic testing 1. Cervical or urethral culture/sensitivity or urine nucleic acid amplification test (NAAT) for GC/CT; urine hCG 2. Consider posterior oropharynx or rectal NAAT (will not delineate type or sensitivity of gonorrhea) 3. Culture and sensitivity (if NAAT unavailable) of any source; use swabs with plastic or wire shafts with rayon or Dacron tips (cotton tips are toxic to N. gonorrhoeae) 4. Consider testing for HIV and other STIs

TESTING-a. Chlamydia trachomatis

i. Diagnostic testing 1. Urine NAAT for N. gonorrhoeae/CT; urine hCG 2. Culture swab for C/S from cervix or urethra and if indicated, from anal or oral sites; culture swabs should be plastic or wire shafts with rayon or Dacron tips (cotton tips are toxic to N. gonorrhoeae and CT) 3. Consider testing for HIV and other STIs

TESTING-a. Trichomoniasis vaginitis

i. Diagnostic testing 1. Wet prep shows trichomonads and WBC >10/hpf (may see trichomonads in urine specimen); must keep slide moist and warm for better visualization 2. Whiff test negative for amine odor 3. pH >4.5 4. Commercial testing with Trichomonas Rapid Test of vaginal secretions

a. presurgical testing

i. Diagnostic testing for presurgical clearance is variable and depends on several factors: (1) the presenting diagnosis, (2) the patient's age, (3) the patient's comorbidities, (4) the type of anesthesia agent planned, and (5) the surgeon's preference.

a. Low milk production

i. Double pumping in addition to nursing produces the best results for increasing supply. If lactogenesis is delayed or inadequate, the infant should receive supplemental feedings while breast stimulation is increased with a breast pump

a. Androgens

i. During pregnancy, androgens originate mainly in the fetal zone of the fetal adrenal cortex. ii. Fetal androgens enter the umbilical and placental circulation and serve as precursors for estrone, estradiol, and estriol iii. The fetal testis also secretes androgens, particularly testosterone, which is converted within target cells to dihydrotestosterone (DHT), which is required for the development of male external genitalia. The main trophic stimulus appears to be hCG.

a. Prolactin

i. During pregnancy, maternal prolactin levels rise in response to increasing maternal estrogen output that stimulates the anterior pituitary lactotrophs. The main effect of prolactin is stimulation of postpartum milk production. ii. secreted by the fetal pituitary may be an important stimulus of fetal adrenal growth. Prolactin may also play a role in fluid and electrolyte shifts across the fetal membranes.

a. Cardiac arrhythmias

i. ECG ii. The management of supraventricular tachyarrhythmias (SVTs) with carotid massage or the Valsalva maneuver, antiarrhythmic therapy, or direct current cardioversion is based on the patient's hemodynamic status. Atrial fibrillation and atrial flutter, though much less common than SVTs, are more serious and are usually associated with underlying maternalcardiac disease.

a. Nausea and vomiting complicate up to 70% of pregnancies.

i. Eating small, frequent meals, and avoiding greasy or spicy foods may help. Also, having protein snacks at night, saltine crackers at the bedside, and room-temperature sodas are nonpharmacologic approaches that may provide some relief. ii. Vitamin B6 (pyridoxine) and acupressure ("sea sickness arm bands") may be effective

a. Red Lesions

i. Eczema 1. Allergic contact mc, itching swelling and crusting /discontinue use of allergen ii. Seborrheic dermatitis 1. Red glazed shiney appreace on skin folds, greasy scalp areas/topical steroids iii. Psoriasis 1. Auto immune, red plaques with clear borders, biopsy, itscing is common, pustular form confused with candidia/ topical steroids

a. Solid cysts or masses

i. Epidermal cysts 1. MC, obstructed hair follicle/ body hair care/ deflation ii. Vulvar vestibular papillomatosis 1. Soft elongated papules/ reassurance iii. Genital warts 1. Human papilloma virus iv. Fox-fordyce disease 1. Chronic inflammation of apocrine glands/systemic antipruritic v. Hidradenitis suppurativa 1. Red papules painful recurrent seen in obesity/ corticosteroids, abt for celluutis, oral contraceptive reduces reoccurrence vi. Vascular lesion 1. tortuous varicosities; cherry angiomas and hematomas/Evaluation, compression; possible evacuation of expanding hematoma vii. Urethral caruncle 1. Solitary red papule at the urethral meatus; usually <1 cm diameter; appears as a collar around urethral opening/ Observation and biopsy as needed; estrogen creams; surgery rarely indicated

Development of fetal immunity

i. Fetal B cells secrete IgG or IgA during the second trimester, ii. Fetal B cells are first detected in the liver by 8 weeks iii. Maternal IgG crosses the placenta as early as the late first trimester, but the efficiency of the transport is poor until 30 weeks. iv. for this reason, premature infants are not as well protected by maternal antibodies. IgM, because of its larger molecular size, is unable to cross the placenta. Physiologically, newborns have higher neutrophil and lymphocyte counts, and the proportion of lymphocytes and the absolute lymphocyte count are higher in neonates than in adults

a. Bleeding in pregnancy

i. For benign causes, reassurance, review of danger signs, and treatment of any underlying causes (e.g., BV) should be provided

a. Pigmented lesions

i. Genital melanosis 1. Dark pigment on muscus menbranes/ expectant management ii. Acanthosis nigricans 1. Pigmented areas on vulva, axilla and neck r/t insulin resistance/ weight loss and glucose control.

a. Epididymitis

i. acute or chronic inflammation of the epididymis and is the most common cause of acute scrotal pain in men 1. bacterial, viral, parasitic, chemically induced, or related to trauma, and it is further categorized as a nonspecific or specific infection or traumatic injury. 2. intact cremasteric reflex a positive Prehn sign (pain relief with the elevation of the affected testicle), and pain along the upper pole of the testicle 3. The two most common causes, especially in younger men, are Chlamydia trachomatis and Neisseria gonorrhoeae ii. Treatment 1. single-dose ceftriaxone given intramuscularly (IM), 250 to 500 mg, 2. doxycycline, 100 mg twice daily for 10 days for men younger than 35 3. < 35 years, a. levofloxacin (given intravenously [IV] or orally [PO]), 500 to 750 mg/day, b. ciprofloxacin, 500 mg (IV or PO), for 10 to 14 days

a. What are the typical assessment domains of the Comprehensive Geriatric Assessment (CGA)?

i. an evaluation of those areas that are key to detecting geriatric conditions and syndromes that are frequently unrecognized or not identified in this population 1. Functional ability: Activities of daily living (ADLs) and instrumental ADLs (IADLs) 2. Physical health: Problems common in older adults and found during a history and physical (e.g., gait, continence, balance, visual/hearing deficits). 3. Cognition and mental health: Screening for cognitive impairment and depression 4. Medications: A review of the patient's current prescription and over-the-counter (OTC) medication use 5. Socioeconomic: The patient's social support network, living situation, home safety 6. Other: Advanced directives, driving

a. Urethral diverticula

i. are small (0.3 to 3 cm), sac-like projections that can be found along the posterior urethra in the midline of the anterior vaginal wall

a. What is the Beers guide used for?

i. assist healthcare providers in improving medication safety in older adults. The role of this guide is to inform clinical decision making, research, training, quality measures and regulations concerning the prescribing of medications for older adults to improve safety and quality of care.

a. Spermatocele

i. benign, painless sperm-filled cyst small, nontender, freely movable mass above and behind the testis of the epididymis located between the head of the epididymis and the testes and arising from the tubules that connect the rete testis to the head of the epididymis 1. form from the obstruction of the efferent duct and contain a milky fluid that consist of spermatozoa, lymphocytes, and debris a. common after vasectomies ii. Treatment 1. None if asymptomatic or excision

a. Musculoskeletal

i. body weight increases but bone mass and lean muscle mass decrease in aging 1. Lyme disease should be a consideration in the differential diagnosis of muscle or joint pain. 2. Polymyalgia rheumatica (PMR) is an inflammatory disorder that most often affects people over 50 years of age.44 The pathophysiology of PMR and its association with giant cell arteritis (GCA) is not clear. These disorders frequently exist together and affect older adults 3. GCA is characterized as a vasculitis and affects large arteries (e.g., the aorta and temporal arteries), but cranial pain, headache, and visual disturbances are the symptoms that cause patients to seek care a. Sensation of stiffness in a patient's neck, shoulders, and pelvis.

a. Imperforate hymen

i. bulging, membrane-like structure may be noticed in the vestibule, usually blocking egress of mucus. If not detected until after menarche, an imperforate hymen

a. Preterm labor

i. cause cervical change and dilation before 37 weeks of gestation, ii. Women with a short cervix (between 10 and 20 mm) should receive vaginal progesterone 200 mg daily from 19 to 20 weeks until 36 weeks iii. If delivery appears possible in the next 7 days, injectable steroids (i.e., betamethasone, dexamethasone) should be used at 24 to 34 weeks' gestation to mature fetal lungs and to prevent neurologic and gastrointestinal complications associated with extreme prematurity

a. Testicular torsion

i. obstruction of blood flow to the testes because of a twisting of the arteries and veins in the spermatic cord 1. unilateral (commonly involving the left testis). ii. Treatment 1. Surgery, restore vascularity as soon as possible because salvage rates are greater than 90% within 6 hours, greater than 50% within 12 hours, and less than 10% in 24 hours

a. Abruptio placenta

i. or premature separation of the normally implanted placenta, bleeding can lead to fetal death ii. painful vaginal bleeding in association with uterine tenderness, hyperactivity, and increased tone

a. Vulvovaginal candidiasis s/s

i. overgrowth of normal yeast that occurs in vagina secondary to hormonal imbalances or changes in vaginal acidity ii. Intense vulvar and vaginal itching and burning; occasional pain with fissuring iii. Thick, creamy ("cottage cheese appearing") vaginal discharge, nonodorous iv. Vulvar erythema causing dysuria and dyspareunia v. Balanitis, urethritis, cutaneous lesions/rash on the penis i. wet prep with KOH slide shows budding yeast or hyphae

a. Placenta previa

i. painless vaginal bleeding in a previously normal pregnancy. ii. The mean gestational age at onset of bleeding is 30 weeks, with one-third presenting before 30 weeks. Placenta previa is almost exclusively diagnosed on the basis of ultrasonography iii. Accreta 1. (percreta). Two-thirds of patients with this complication require hysterectomy when an attempt to remove the placenta leads to severe hemorrhage intrapartum. Patients with a history of uterine surgery are at greatest risk of developing an accreta

a. Vaginal agenesis

i. represents the most extreme instance of a vaginal anomaly, with total absence of the vagina except for the most distal portion that is derived from the urogenital sinus,

a. PUD

i. symptomatic improvement in response to conservative treatment. ii. Endoscopy is reserved for those patients who do not respond to treatment, have more severe GI symptoms, or manifest significant GI hemorrhage. iii. Treatment involves avoiding caffeine, alcohol, tobacco, and spicy foods and administering antacids iv. H2 receptor antagonists, or proton pump inhibitors. Any nonsteroidal antiinflammatory drugs should be discontinued. 1. Antibiotic therapy should be considered for patients with Helicobacter pylori infection, but it is generally not given until the patient is postpartum.

You receive the lab results of a 20-week gestation patient and note the sodium level is 131 mEq/L. What is an appropriate action? Nothing, because this is a normal sodium value Immediately send the patient to the emergency department Prescribe Na+ supplementation tablets Nothing, because this is an expected change in pregnancy Correct! Sod

ium levels decrease by 2-4 mEq/L in pregnant women. Since this sodium level is at the very low end of normal, you may want to monitor closely with repeat testing.

Management cardiac

t i. Cardiac patients should be delivered vaginally unless obstetric indications for cesarean delivery are present ii. Pushing should be avoided during the second stage of labor because the associated increase in intraabdominal pressure can lead to cardiac decompensation. The second stage of labor can be assisted by performing an outlet forceps delivery or by the use of a vacuum extractor. iii. Cardiac output increases up to 80% above prelabor values in the first few hours after a vaginal delivery and up to 50% after cesarean delivery. To minimize the risks of volume overload or depletion, careful attention should be paid to fluid balance (avoiding overload) and prevention of uterine atony (avoiding depletion from blood loss) with oxytocin and uterine massage. When these measures are unsuccessful, prostaglandin F2α can be administered if pulmonary hypertension and reactive airway disease are not concerns avoid methergine iv. The use of digitalis is controversial. ACE inhibitors and similar drugs should be avoided

recurrent yeast infections should warrant further investigation for causes, such as

undiagnosed diabetes, possible environmental vaginal irritants, or a need to change oral contraceptives.

a. Progesterone !

i. In the luteal phase, it induces secretory changes in the endometrium and in pregnancy, higher levels induce decidual changes. ii. Up to the sixth or seventh week of pregnancy, the major source of progesterone (as 17-hydroxyprogesterone) is the ovarian corpus luteum. iii. If the corpus luteum of pregnancy is removed before 7 weeks and continuation of the pregnancy is desired, progesterone should be given to prevent spontaneous abortion. iv. Progesterone prevents uterine contractions and may also be involved in establishing an immune tolerance for the products of conception v. suppresses gap junction formation, placental CRH expression, and the actions of estrogen, cytokines, and prostaglandin. This steroid hormone therefore plays a central role in maintaining uterine quiescence throughout most of pregnancy.

S/S-a. Gonorrhea

i. Infection caused by N. gonorrhoeae, involving any mucus membrane (e.g., commonly genitalia, but also pharynx or eyes); may disseminate into joints, endocardium, and meninges ii. After exposure, approximately 60% to 90% of women become infected iii. If untreated, approximately 15% will have PID with increased risk of infertility

RDD MANGMT

i. Intrauterine Transfusion 1. The goal is to transfuse fresh group O, Rh-negative packed red blood cells. 2. Transfusions usually cannot be done until 18 to 20 weeks' gestation, because fetal size limits vascular access. 3. The overall survival rate following intrauterine transfusion is about 90%, but it is significantly lower for fetuses with hydrops before the transfusion. ii. Fetal Intraperitoneal Transfusion 1. In nonhydropic fetuses, the blood should be absorbed within 7 to 9 days 2. intraperitoneal transfusion is reserved for cases in which intravascular transfusion is not possible, such as gestational age less than 20 weeks. iii. Intravascular Transfusion 1. Under ultrasonic guidance, and using sterile technique, a 22-gauge spinal needle is inserted into the umbilical vein near the placental insertion

a. Anemia

i. Iron replacement therapy with 60 to 120 mg of elemental iron daily is indicated if hemoglobin is less than 11 g/dL, along with an increase in iron-rich food intake.

a. Engorgement

i. It can usually be minimized by feeding the infant 8 to 12 times each day in the days leading up to the milk coming in. ii. Deep asymmetric latch-on and sustained suckling will resolve most engorgement. iii. If engorgement persists, ice packs may be applied to the breasts between feedings to reduce edema, and hot packs may be applied right before feedings to soften the areola. Pumping for a minute or two before latch-on may also help.

1. vulvar and vaginal cancer.a. Clinical features

i. Itching is the most common symptom, although some patients present with palpable or visible abnormalities of the vulva. ii. Asymptomatic iii. Approximately 20% of the lesions have a "warty" appearance, and the lesions are multicentric in about two-thirds of cases.

a. Surgery ectopic pregnancy

i. Laparotomy is the preferred surgical approach for women who are hemodynamically unstable, ii. Salpingectomy (removal of the entire fallopian tube) is recommended when there has been significant damage to the tube, when a patient who previously has been sterilized verifies that she still does not desire future fertility, and when there is a high likelihood of retained products of conception. 1. salpingostomy results in better long-term tubal function compared with salpingotomy. The risk for incomplete trophoblastic tissue removal is greatest when the ectopic products of conception are "milked" through the tube to extrude through the fimbria. This technique should never be used, even when it appears that the pregnancy is spontaneously aborting through the fimbria

a. Rheumatic heart disease

i. MC lesion is mitral stenosis ii. nearly all women who develop atrial fibrillation during pregnancy experience congestive heart failure.

a. Leg cramps

i. Massage and stretching may afford some relief during an attack. Both calcium and sodium chloride supplementation ii. vitamin D 1000 to 2000 IU/day for pregnant women Daily requirements are 400 to 600 IU/day, which is the amount of vitamin D in prenatal vitamins, so prenatal vitamins will not correct the deficiency.

a. Cracked nipples

i. Medical-grade lanolin may improve breastfeeding comfort if the nipples are dry and cracked. For severely damaged nipples, hydrogel dressings are useful to maintain a moist wound healing environment. In addition, mothers must avoid overdrying their nipples and should apply warm, moist compresses after feedings to soothe and promote healing. ii. Nipple soreness typically improves after a rest from breastfeeding for 24 hours and institution of the correct attachment position. Mothers will need to pump every 3 hours while not breastfeeding. In general, it takes 10 days for sore nipples to heal completely

vulvar and vaginal cancer a. Management

i. Most cases of VIN can be treated adequately by local superficial surgical excision with primary closure. margins of about 5 mm are usually adequate. For extensive lesions involving most of the vulva, a "skinning" vulvectomy ii. Since the 1980s, a vulva-conserving approach has been used for most primary lesions, and the groin dissection has usually been performed through a separate groin incision

a. Poor latch

i. Mothers should be taught how to bring the baby to the breast to prevent the infant from sucking on the nipple tip. Shallow latch causes pain and poor let-down.

a. Depression

i. Neurotransmitter changes in aging also can possibly cause depression ii. constipation or urinary issues 1. Patient Health Questionnaire (PHQ-9) 2. Geriatric Depression Scale

a. What are the recommendations for treating chronic pain?

i. Nonpharmacologic therapy and nonopioid pharmacologic therapy are preferred for chronic pain. Clinicians should consider opioid therapy only if expected benefits for both pain and function are anticipated to outweigh risks to the patient. If opioids are used, they should be combined with nonpharmacologic therapy and nonopioid pharmacologic therapy, as appropriate. ii. Before starting opioid therapy for chronic pain, clinicians should establish treatment goals with all patients, including realistic goals for pain and function, and should consider how therapy will be discontinued if benefits do not outweigh risks. Clinicians should continue opioid therapy only if there is clinically meaningful improvement in pain and function that outweighs risks to patient safety. iii. Before starting and periodically during opioid therapy, clinicians should discuss with patients known risks and realistic benefits of opioid therapy and patient and clinician responsibilities for managing therapy.

a. Sleep disorders.

i. Nonrapid-eye-movement or slow-wave sleep decrease in aging (frequent waking) ii. Behavior or diet changes 1. Medications not recommended for sleep for older adults 2. Doxepin, a tricyclic antidepressant, in doses less than 6 milligrams po at bedtime is a possible therapeutic, although the indication is to use doxepin only if the patient has depression-related insomnia. Low-dose mirtazapine in older adults with depression

a. Epithelial Changes

i. Papillomatosis 1. Appear like papilloma virus/ reassurance ii. Contact dermatitis 1. irritation/ discontinue irritant

1. HIV.a. risk factors,

i. People who inject drugs and their sex partners. ii. People who exchange sex for money or drugs. iii. Sex partners of people with HIV. iv. Sexually active gay, bisexual, and other men who have sex with men (more frequent testing may be beneficial [e.g., every 3-6 months]). v. Heterosexuals who themselves or whose sex partners have had ≥1 sex partner since their most recent HIV test. vi. People receiving treatment for hepatitis, tuberculosis, or a sexually transmitted disease.

a. Vision

i. Peripheral vision is influenced by age ii. Presbyopia focusing up close 1. decrease in lens elasticity and the subsequent decrease in focus accommodation for near object iii. Burning or sting like eye

1. breast disorders. a. Nipple discharge

i. Physiologic causes: 1. Pregnancy and lactation or recent lactation (e.g., bilateral clear to straw colored with early pregnancy) 2. COC use 3. Overstimulation of breasts 4. Cysts (e.g., unilateral discharge, is usually serous to greenish color) 5. Nipple piercing 6. Cloudy/milky appearance may be common in premenopausal women and is usually bilateral and painless ii. Pharmacologic causes include the following: 1. Estrogen products, herbal products with estrogenic effects 2. Metoclopramide 3. Most common psychiatric medications a. Clomipramine b. Risperidone c. Fluphenazine d. Haloperidol

a. Hyperemesis gravidarum

i. Pregnancy outcome is usually good if the disorder is treated and there is catch-up weight gain. ii. intractable vomiting, beginning in the first trimester, is usually elicited. Physical findings of weight loss, dry and coated tongue, and decreased skin turgor are very suggestive. 1. Significant abdominal pain and tenderness are generally absent. iii. If outpatient management fails, patients must be admitted for intravenous administration of fluids, electrolytes, glucose, vitamins, and medical therapy. 1. may require nasogastric feeding or parenteral nutrition.

a. Peripartum cardiomyopathy

i. Pregnant women particularly at risk of developing cardiomyopathy are those with a history of preeclampsia or hypertension and those who are poorly nourished.

a. Preterm rupture of membranes

i. Preterm PROM (PPROM) should be used to define those patients who are preterm with ruptured membranes, whether or not they have contractions.

a. D/c Prep

i. Protection from HIV infection will wane over 7-10 days after ceasing daily PrEP use ii. Because some patients have acquired HIV infection soon after stopping PrEP use alternative methods to reduce risk for HIV acquisition should be discussed, including indications for PEP and how to access it quickly if needed

a. Acute and recurrent cystitis

i. Several factors increase the risk for cystitis, including sexual intercourse, the use of a diaphragm and a spermicide, delayed postcoital micturition, and a history of a recent urinary tract infection ii. Escherichia coli is present in the urine of 80% of young women with acute cystitis iii. There has been an increasing linear trend in the prevalence of resistance of E. coli (>10%) to the fluoroquinolones (e.g., ciprofloxacin) iv. Nitrofurantoin (macrocrystals, 100 mg orally twice daily for 5 days) or trimethoprim-sulfamethoxazole (160/800 mg orally twice daily for 3 days) are the optimal choices for empirical therapy for uncomplicated cystitis. v. In patients with typical symptoms, an abbreviated laboratory workup followed by empirical therapy is recommended.

a. preeclampsia/eclampsia

i. Severe hypertension (systolic BP ≥160 mm Hg or diastolic BP ≥110 mm Hg) at rest on two occasions at least 4 hr apart* ii. Renal insufficiency (serum Cr >1.1 mg/dL or doubling of baseline values) iii. Cerebral or visual disturbances iv. Pulmonary edema v. Epigastric or right upper quadrant pain vi. Elevated liver enzymes (AST or ALT at least two times normal level) vii. Thrombocytopenia (platelet count <100,000/µL) viii. ALT, Serum alanine aminotransferase; AST, serum aspartate aminotransferase; BP, blood pressure; Cr, creatinine. 1. Severe hypertension (systolic blood pressure ≥160 mm Hg and/or diastolic blood pressure ≥110 mm Hg) in the setting of preeclampsia requires rapid blood pressure control, usually with parenteral labetalol or hydralazine to prevent CNS hemorrhage or stroke. When preeclampsia is accompanied by new-onset grand mal seizures, it becomes eclampsia. Eclampsia can result in major maternal morbidity, including intracerebral hemorrhage, placental abruption, and preterm birth or fetal death. 2. A variant of severe preeclampsia with particularly high morbidity is the HELLP syndrome. This syndrome occurs in preeclamptic women with evidence of hemolysis, elevated liver enzymes, and low platelets (thrombocytopenia).

a. Fibrocystic breast changes i. Definition: premenstrual breast tenderness; may be either consistent or cyclic

i. Signs and symptoms 1. Lumpy breast(s) with nodules of varying sizes and shapes but usually smooth, well defined, and mobile 2. Tenderness on palpation to one or both breasts, ∼1 to 2 wk before menses and may resolve for 2 wk after menses 3. Tenderness may be aggravated by menopause and/or initiation of HRT 4. May have nipple discharge that is clear to dark, cloudy color

a. Ischemic heart disease

i. Stress testing and angiography should be done before pregnancy. Use of lipid-lowering agents or angiotensin-converting enzyme (ACE) inhibitors should be avoided during pregnancy because these drugs are currently labeled class X because of concerns regarding fetal toxicity.

a. Delayed let down

i. Stress, pain, and alcohol inhibit let-down. The let-down response is enhanced through thoughts about the infant, breast massage, and relaxation. ii. Although all mothers should be informed about inhibiting and enhancing factors for let-down, this information may be especially important for mothers who need to pump milk from their breasts, including mothers of premature infants or mothers returning to work

a. Taste

i. Taste disturbances can be described as diminished (hypogeusia), not pleasant (aliageusia), or simply absent (ageusia)

1. PrEP. a. Prescribing guidelines

i. Test for HIV if + not eligible ii. Syphilis? Gonorrhea chlamydia iii. Check renal function and hep B status iv. Vaccinate against HBV 1. Not contraindicated against Prep if HBV +

a. Endocervicitis

i. The diagnosis of cervicitis is based on the finding of a purulent endocervical discharge, generally yellow or green in color, and referred to as "mucopus ii. N. gonorrhoeae and C. trachomatis are associated with mucopurulent endocervicitis 50% of the time. Other etiologies include M. genitalium, bacterial vaginosis, and birth control pills. iii. dual therapy is recommended, and should include an intramuscular injection of ceftriaxone 250 mg, and a single oral dose of azithromycin (1 g). iv. It is imperative that all sexual partners be treated with a similar antibiotic regimen. Cervicitis is commonly associated with BV, which, if not treated concurrently, leads to significant persistence of the symptoms and signs of cervicitis.

a. Respiratory system

i. The diaphragm at rest rises to a level of 4 cm above its usual resting position. ii. As pregnancy progresses, the enlarging uterus elevates the resting position of the diaphragm. iii. the vital capacity (VC) remains unchanged. These characteristics—reduced FRC with unimpaired VC—are analogous to those seen in a pneumoperitoneum iv. Total body oxygen consumption increases by about 15-20% in pregnancy v. In pregnancy, the elevations in both cardiac output and alveolar ventilation are greater than those required to meet the increased oxygen consumption. vi. The rise in minute ventilation reflects an approximately 40% increase in tidal volume at term; the respiratory rate does not change during pregnancy vii. progesterone increases ventilation. viii. such increased respiratory sensitivity to CO2 is characteristic of pregnancy and probably accounts for the hyperventilation of pregnancy. ix. The hyperventilation of pregnancy results in a respiratory alkalosis. x. There is a slight downward trend in arterial PO2 as pregnancy progresses xi. Despite the absence of obstructive or restrictive effects, dyspnea is a common symptom in pregnancy. Some studies have suggested that dyspnea may be experienced by as many as 60-70% of women at some time during pregnancy

a. Toxoplasmosis

i. The infection is usually without symptoms in the mother, but when symptoms do occur they involve fever, rash, and fatigue. ii. Household cats litter box iii. Uncooked/undercooked food from infected beef, lamb, and other animals iv. Can result in fetal microcephaly fetal hepatomegaly enlarged placenta v. Treatment during pregnancy for infected women is available using spiramycin.

a. Epidemiology

i. The more common type occurs in older women and is frequently related to long-standing lichen sclerosis. ii. The less common type occurs in younger women and is related to infection with the human papillomavirus and smoking. iii. Most tumors are squamous cell carcinomas, and they occur mainly in postmenopausal women iv. vulvar cancer may also occur in association with the venereal diseases syphilis, lymphogranuloma venereum, and granuloma inguinale

The common gynecologic causes of acute lower abdominal pain a

re salpingo-oophoritis with peritoneal inflammation, torsion and infarction of an ovarian cyst, endometriosis, or rupture of an ectopic pregnancy.

i. Provider-applied therapies WARTS

1. Cryotherapy with liquid nitrogen q1-2wk 2. Podophyllin resin 10% to 25% in compounded tincture of benzoin use ≤ 0.5 mL/treatment 3. The area of application should be small and only on intact lesions 4. Apply to each wart and allow to dry before wearing clothes 5. Wash off after 1 to 4 hr to reduce irritation 6. Avoid with pregnancy 7. Trichloroacetic acid (TCA) or bichloroacetic acid (BCA) 80% to 90% 8. Before application, apply a small amount of Vaseline on the surrounding skin to prevent the acid from touching healthy tissue 9. For intense pain, neutralize with soap or sodium bicarb (i.e., baking soda) 10. Repeat treatment q1wk 11. Can be used with both vulvar and vaginal warts and in pregnant women

Fibrocystic breast changes TX

1. Decrease or stop caffeine, tea, chocolate, and nicotine 2. Wear a good supportive bra 3. Apply hot or cold compresses to breast when pain occurs 4. NSAID of choice 5. Vitamin E 1000 IU 1 tab qd, vitamin B6: 50 to 100 mg 1 tab qd (may help with pain) 6. Switch to COCs with lowest estrogenic activity or discontinue use a. Refer to breast surgeon if nodules are painful and need further evaluation

elements of prenatal care.

1. Women who test negative for hepatitis B surface antigen and are at high risk for hepatitis B infection (e.g., health care workers) are candidates for vaccination before and during pregnancy 2. Infants born to women who test positive for hepatitis B surface antigen should receive both hepatitis B immune globulins (HBIG) and hepatitis B vaccine within 12 hours of birth, followed by two more injections of hepatitis B vaccine in the first 6 months of life.

a. PREGNANCY i. Both the FDA labeling information6 and the perinatal antiretroviral treatment guidelines permit off-label use in pregnancy

1. mothers should be instructed not to breastfeed if they are receiving TRUVADA, whether they are taking TRUVADA for treatment or to reduce the risk of acquiring HIV-1

ecommendations for prostate cancer screening.

1. recommendations for prostate cancer screening. a. For men aged 55 to 69 years, the decision to undergo periodic PSA-based screening for prostate cancer should be an individual one and should include discussion of the potential benefits and harms of screening with their clinician. i. Patients with signs and symptoms of prostate cancer should be referred to a urologist ii. The decision should be made after getting information about the uncertainties, risks, and potential benefits of prostate cancer screening. Men should not be screened unless they have received this information. valuation, even in the presence of a normal PSA level. iii. Men who choose to be tested who have a PSA of less than 2.5 ng/mL may only need to be retested every 2 years. iv. Screening should be done yearly for men whose PSA level is 2.5 ng/mL or higher.

a. Genital warts TX

1. topical therapies; therapy may take as long as 3 mo to ablate lesions 2. Results improve if <6 warts 3. If no response to treatment in 3 to 6 wk, consider switching to another treatment 4. B. Patient-applied therapies (avoid with pregnancy) 5. Podoflox 0.5% solution or gel 6. Apply with a cotton swab to visible warts bid for 3 days, followed by no therapy for 4 days 7. Repeat as necessary up to 4 cycles 8. Imiquimod 5% cream 9. Apply at hs 3 times a week for up to 16 wk 10. Wash with soap and water 6 to 10 hr after application 11. Sinecatechins 15% ointment (extract from green tea) 12. Apply tid until resolution, but not longer than 16 wk 13. Do not wash off 14. No sexual contact while the ointment is on the skin

undescended testes.

1. undescended testes. a. have an approximately 17% higher incidence of developing testicular cancer than the general population 1. premature, low birth weight 2. painless on sac is empty 3. can be acquired ii. surgery treatment of choice 6mo to 1 year.

Which factors determine which diagnostic tests should be performed in a presurgical clearance evaluation? (Select all that apply.) Patient's age Type of anesthetic agent planned Previous surgeries Patient's comorbidities Surgeon's preference

ABDE-Feedback: The patient's age and comorbidities, surgeon preference, and the type of anesthetic planned all determine which presurgical diagnostic tests will be performed. The patient's previous surgeries do not determine presurgical testing.

4.A patient who is near death is exhibiting signs of agitation, anxiety, and intractable pain. When discussing palliative sedation with this patient's family, what will be discussed? (Select all that apply.) The need for informed consent from the patient and family The chance that refractory symptoms will be alleviated The fact that this is an intervention of last resort The chance of drug dependence is irrelevant in this situation. The possibility that this measure may hasten death The likelihood that the patient will develop dependence on the drugs

ACD-Feedback: Palliative sedation is used as a treatment of last resort for patients whose symptoms are intolerable or refractory. Patients, if possible, and family members must give informed consent. This treatment has the possibility of hastening death by inhibiting respirations. Symptoms will not be alleviated by using the measure.

44 An 82-year-old patient comes to a clinic for routine care. The patient lives alone and has no family living close by. Which topic should the advanced practice registered nurse (APRN) discuss as part of the plan of care for this patient? Completing the Beers criteria Reviewing advance directives Moving into a nursing home Invoking durable power of attorney for healthcare

B

41 An advanced practice registered nurse (APRN) is treating a 96-year-old patient who is taking six different medications for chronic diseases. The APRN wants to improve compliance with the medications while avoiding pill burden. Which action should the APRN take to meet these goals? Change the medications to liquid form Use combination medications to minimize prescriptions Encourage the patient to grind up all pills together Reduce the frequency of all medications to once a day 42

B

A 29-year-old male is being seen by an advanced practice registered nurse (APRN) for malaise, dysuria, and lower-back pain. After examination, the APRN detects a tender, warm, and boggy prostate. Penile secretions are positive for neisseria gonorrhoeae. Which treatment should the APRN prescribe for this patient? Ciprofloxacin 500 mg bid for 5 to 7 days Amoxicillin 875 mg bid for 10 to 14 days Doxycycline 100 mg bid for 7 days Levofloxacin 750 mg qd for 5 to 7 days

C

A 32-year-old female presents to the clinic as a new patient to establish care and complains of vague symptoms of abdominal fullness and bloating. On the rectovaginal exam, the advanced practice registered nurse (APRN) notes a solid, irregular, and fixed pelvic mass. Which cancer diagnosis should the APRN make for this patient? Vulvar Cervical Ovarian Anal

C

A female patient comes to the emergency department and states that she was raped on her way home from classes at the local college. The advanced practice registered nurse (APRN) notes that the patient appears disheveled, has bruising on her face and thighs, and has abrasions present on her knees, back, and elbows. Which action should the APRN take? Call the police to interview the patient and collect physical evidence Prescribe ulipristal acetate 30 mg once daily for three days Develop a relationship with the patient and get consent to collect evidence Give a one-time dose of alprazolam 0.1 mg to decrease post-traumatic stress disorder (PTSD) symptoms 47

C

An advanced practice registered nurse (APRN) is seeing a 35-year-old male patient who is complaining about a loss of libido and erectile dysfunction. The initial laboratory findings from the CBC, CMP, TSH, and PSA are within normal limits, but the patient has a low testosterone level. Which test should the APRN perform next? Alpha-fetoprotein Serum iron Serum prolactin Lactate dehydrogenase

C

A 40-year-old female patient seeks medical attention from an advanced practice registered nurse (APRN) for increasing anxiety that is only reduced by ingesting alcoholic beverages. The patient reports caring for two school-age children in addition to an adult parent who was recently diagnosed with a cognitive disorder. Which response should the APRN give to this patient? "Edible forms of marijuana might be better than alcohol." "A prescription for medical marijuana may be helpful." "Alcohol can cause rebound anxiety." "The time of day when alcohol is ingested may need to be changed."

Correct! Alcohol intake should be discouraged because it can cause rebound anxiety and lead to addiction problems.

A family nurse practitioner is credentialed to treat any patient in any care setting. True False

Correct! All nurse practitioners must practice within their education and certification as defined by the APRN consensus model.

What is an appropriate strategy to minimize the burden of a patient's treatment? Focus care on preventative services instead of difficult treatments. Evaluate functional status and ability to carry out the prescribed treatment. Stop all treatments. Prescribe continuous physical therapy for strengthening.

Correct! An evaluation of the patient's functional status should help the nurse practitioner determine which treatments are reasonable for the patient to perform.

What is a recommended medication course for chlamydia trachomatis? Ceftriaxone 250 mg IM once Erythromycin 150 mg PO QID x 3 days Doxycycline 1 g PO QD x 5 days Azithromycin 1 g PO once

Correct! A one-time dose of 1 gram of azithromycin is a medication course for chlamydia. Additionally, this one-time dose has a higher compliance rate than medication courses with multiple doses over several days.

The Katz independence in activities of daily living scale is a tool commonly used to assess basic function. True or False A score of four indicates severe impairment. True False

Correct! A score of four on the Katz independence in activities of daily living scale indicates moderate impairment. A score of two indicates severe impairment. A score of six suggests full function.

Question 1 This is not a form; we suggest that you use the browse mode and read all parts of the question carefully. During a neurological screening as part of a Medicare wellness exam, the daughter of a 76-year-old patient reports to an advanced practice registered nurse (APRN) that her mother has had a change in consciousness and an increase in lethargy over the last two days. The APRN suspects a problem and conducts further assessments to confirm the diagnosis. Which diagnosis should the APRN suspect? Diabetes Thiamine deficiency Pneumonia Vitamin B12 deficiency

Correct! A urinary tract infection or pneumonia are suspected in older adults with changes in consciousness, such as confusion and lethargy.

One attribute of the patient-centered medical home model of care is accessible services. True or False Accessible services mean offices must be equipped to manage patients who are differently abled. True False

Correct! Accessible services refer to the medical home practice that delivers shorter waiting times for urgent needs, alternative methods of communication, around-the-clock access to a member of the care team, and extended office hours.

A patient presents to your office for preoperative clearance for a total knee replacement. The patient is complaining of cough, fever, and shortness of breath. You diagnose bronchitis. True or False You can clear this patient for surgery now because the illness is acute and will resolve. True False

Correct! Patients must be healthy when they are cleared for surgery. This patient's condition could worsen, and patients should not undergo surgery while they are febrile.

An advanced practice registered nurse (APRN) is seeing a 96-year-old patient. The patient reports taking six and one-half pills every day. The APRN decides to respond to that information by taking a course of action recommended by the goals of geriatric care. Which action should the APRN take for this patient? Have the patient take the medications with an increased amount of water Ensure the patient's pills are in childproof bottles Determine how to delete or combine the half pill Create a six-bulleted chart to help the patient monitor medications

Correct! Avoiding half pills is one of the goals of geriatric care to simplify prescriptions.

You prescribed percocet 5-325 mg 1-2 tablets every 4-6 hours as needed. What is the maximum daily morphine milligram equivalent (MME) dose for this prescription? 20 MME 75 MME 35 MME 60 MME

Correct! Oxycodone dose is converted to MME by multiplying by 1.5 times. The prescription is for up to 10 mg oxycodone every 4 hours, or 60 mg a day. By multiplying 60 mg by 1.5, you have a 75 MME for this prescription.

The victims of sexual assault almost always have associated immediate and long-term effects, termed the rape trauma syndrome. What are the two phases of rape trauma syndrome? Treatment phase and chronic sequelae phase Acute/disorganization phase and the integration and resolution phase Shock phase and depression phase Denial and acceptance phases

Correct! Acute/disorganization phase starts immediately after the experience and can last from days to weeks. Victims may appear calm, although they commonly express shock, disbelief, fear, guilt, and shame. In the integration and resolution phase, victims begin to accept the assault, but psychological and emotional problems may persist.

True or False Gestational diabetes is a risk factor for developing preeclampsia. True False

Correct! Preeclampsia occurs more frequently in pregnant patients with gestational diabetes.

An advanced practice registered nurse (APRN) is part of an integrated multidisciplinary team that is working in a busy rural clinic. The team has been struggling with day-to-day operational communication, so the APRN suggests a strategy for improvement. Which strategy should the APRN suggest to manage this problem? Promoting timely feedback on concerns Requiring attendance at daily huddles Instituting a sign-off process for treatment plans Adopting an integrated electronic health record

Correct! Adoption of a health record that addresses both medical and behavioral documentation is key to addressing day-to-day operational needs.

Advance directive conversations must occur in the hospital, hospice, or long-term setting. True False

Correct! Advance care planning can occur in any setting, including primary care office visits.

In palliative care the patient may still receive life-prolonging and curative treatment. True False

Correct! Palliative care and hospice both focus on comfort; however, hospice patients must have a six-month or less life expectancy and have chosen to forgo curative treatment. Patients receiving palliative care can continue life-prolonging treatments.

Prescription opioids taken for longer periods of time or in higher dosages can increase the risk of opioid use disorder, overdose, and death. True False

Correct! Prescription opioids taken in higher doses or for longer periods have been associated with increased risk of opioid use disorder (abuse), overdose, and death.

A patient with a terminal illness explains to an advanced practice registered nurse (APRN) that family from out of town will be visiting, and they have questions about planning for the patient's care. During a previous visit, the APRN provided the patient with a medical orders for life-sustaining treatment (MOLST) form. Which information about this form should the APRN emphasize with this patient? One form needs to be completed for in-hospital use and a separate form for community care. The instructions on the form are activated after it is signed by both the patient and the patient's family. The form is used by the patient to select treatment choices and their health agent. The selections on the form cannot be changed once they are completed.

Correct! A MOLST form covers all aspects of advanced care planning to include life-sustaining interventions, the name of the identified health agent, the name of the physician, the patient's signature, and the clinician's signature, which is required to activate the form.

A 52-year-old woman presents to an advanced practice registered nurse (APRN) for complaints of hot flashes, irritability, and difficulty falling asleep. The APRN notes a T score of −2.0 on the woman's bone density scan. Which action should the APRN recommend for this patient? Start walking and weight-bearing exercises Supplement 2,000 to 3,000 mg of calcium twice daily Increase potassium and decrease vitamin D intake Begin a high-protein, low-carbohydrate diet

Correct! A bone densitometry T score of −2.0 indicates low bone density or osteopenia. Walking and weight-bearing exercises both help to increase bone mineral mass and reduce the risk of fracture-causing falls

The yellow fever vaccine requires a booster every five years. True False

Correct! A booster is needed every 10 years for the yellow fever vaccine.

The goal of a co-management agreement document is to define the roles of each provider and communicate this information to all clinicians. True False

Correct! A co-management agreement document enhances communication by defining the roles and expectations for each provider.

A family history of breast cancer is a risk factor for developing breast cancer. What is another established risk factor for developing breast cancer? Menarche at age 13 A history of benign breast biopsy First live birth at age 26 Smoking

Correct! A history of a breast biopsy with any histologic finding, even benign findings, is an established risk factor for developing breast cancer.

A heath system wants to decrease the number of employee injuries from lifting. A project team is formed to select lifting equipment that meets the needs of several departments across the health system. An advanced practice registered nurse (APRN) leads a team to evaluate equipment from three manufacturers. The team develops a chart to evaluate each piece of equipment and scores the equipment according to how it would operate in the following categories: • Severity: Identify all activities the equipment is expected to do. • Occurrence: For each activity, evaluate what could go wrong. • Detection: For each thing that could go wrong, evaluate what the effect would be. According to this scenario, which quality process did the APRN implement for this project? FMEA DMAIC SMART PDCA

Correct! FMEA (failure mode effects analysis) is a tool for proactively evaluating products and identifying ways to prevent defects. It can spot problems before a piece of equipment is put into practice.

At what point should families be involved in advance care planning? When a patient can no longer make his or her wishes known When a life-limiting illness is diagnosed During the terminal stages of a disease During any stage of health or disease course, and at any age

Correct! Families should always be encouraged to be involved and aware of patients' wishes because emergency situations that prohibit the patient from communicating can occur at any time.

How can poor nutrition and vitamin deficiencies affect the fetus? Breech presentation Metabolic changes and distended scalp veins Infantile torticollis Neural defects and endocrine changes

Correct! Folic acid deficiencies can cause neural defects, and poor nutrition can result in endocrine changes.

A 20-year-old woman meets with an advanced practice registered nurse (APRN) for a Pap smear, and there is blood in the specimen. This is the first time this result has occurred. Which recommendation should the APRN include in this patient's plan of care? Schedule a follow-up Pap smear in three months Repeat Pap smear as soon as possible Order a reanalysis of the results Obtain a CBC

Correct! If Pap smear results are unsatisfactory for any reason, such as insufficient cells, obstruction with solution particles, or blood in specimen, the Pap smear should be repeated in three months.

When caring for individuals in long-term care facilities, the nurse practitioner's goal of care is rehabilitation and improving functional status back to baseline. True False

Correct! In skilled care, the NP's goal of care is improving function back to baseline. For long-term care, the patient is stable, and no improvement is expected, so the NP's goal of care is maintaining function and preventing disease exacerbation.

A patient with a history of epilepsy tells an advanced practice registered nurse (APRN) about experiencing increased activity and lip-smacking during seizures. The patient is compliant with the current medication regimen. Which action should the APRN take next for this patient? Consult with a neurologist Change the anticonvulsant Schedule an EEG Order a STAT CT scan of the head

Correct! Increased activity and lip-smacking during seizures may indicate temporal lobe seizures. Because there is a change, the APRN needs to consult with a neurologist before determining other actions.

A patient presents to an advanced practice registered nurse (APRN) because of frequent headaches that started about two years before. After reviewing the patient's medical records, the APRN notes that the patient has already had several diagnostic tests performed. The patient has tried nonnarcotic medications with little effect. Which action should the APRN take to assist this patient? Prescribe a short-acting opioid at a low dose Schedule a urine drug screen in two weeks after prescribing opioid therapy Refer to a local substance abuse center for assistance with pain management Recommend long-acting medication for breakthrough pain

Correct! It is appropriate to prescribe a short-acting opioid using the lowest dose on product labeling and then reassess results in one to four weeks. This is especially important if there is consideration for long-term opioid therapy.

Dementia screenings are not necessary until patients exhibit signs of memory impairment. True False

Correct! It is necessary to perform a screening specifically for memory impairment because dementia symptoms are not always apparent with routine physical exams. Although there is no cure, prompt detection and treatment can add precious time to your patient's life.

As a prescriber, you must write for stronger doses of pain medication for patients who report high levels of pain on a pain scale. True False

Correct! Many factors that influence pain may be missed by focusing solely on pain intensity and rating. Pain therapy isn't necessarily about getting pain intensity to 0.

Various employee protection laws have agricultural exclusions, resulting in insufficient on-the-job protections for many migrant farmworkers. True False .

Correct! Many migrant farmworkers do not have the same worker protection standards and are exposed to more occupational health risks

Which type of visit will Medicare reimburse for advance care planning? Medicare does not reimburse for advance care planning discussions Sports physical Office visit for diabetic education Medicare wellness visits and a hospital follow-up after an acute severe illness

Correct! Medicare reimburses for advance care planning if the following are documented in the note: an account of the discussion, who was present, explanation of forms, and the time spent in the face-to-face encounter. In general, Medicare reimburses for advance care planning during visits for routine physicals and with significant changes in health status.

When conducting a gynecologic history, what are the upper and lower ages considered normal for menstruation? Ages 9 and 75 Ages 12 and 60 Ages 16 and 52 Ages 9 and 52

Correct! Menarche before age 9 and continued menstruation past age 52 are causes for concern and require further workup. Though some women continue normal menstruation up to age 58, vaginal bleeding should be evaluated for uterine cancer or other sources of exogenous estrogen after the age of 52.

In geriatric patients, a new symptom should be considered a drug side effect or drug-drug interaction until proven otherwise. True False

Correct! Older adults can present with atypical symptoms, and new symptoms should be considered to be caused by medications until proven otherwise.

One of the main reasons for a post-hospitalization follow-up appointment is to prevent hospital readmission. True False

Correct! Preventing readmission is one of the primary goals of a post-hospitalization follow-up visit.

40 An advanced practice registered nurse (APRN) reviews a patient's history and notes that prednisolone is being taken for severe allergies even though the patient no longer has allergy symptoms. The APRN decides to stop this medication and reviews the history again to determine if the medication must be discontinued gradually instead of abruptly. Under which circumstance should the APRN discontinue this medication gradually? If the patient has been receiving 20 mg daily for the past four days If the patient has been on the medication for two weeks If the patient has been receiving 10 mg daily for the past six days If the patient has been on the medication for four weeks

D

50 An advanced practice registered nurse (APRN) is evaluating a 96-year-old male for possible pneumonia. The APRN identifies that the patient has pneumonia and requires antibiotics. The APRN is planning multimodal treatments for this patient. Which system should the APRN carefully evaluate first before recommending a wide-spectrum antibiotic? Dermatological Endocrine Neurologic Renal

D

53 An advanced practice registered nurse (APRN) is caring for a newly admitted Filipino patient and needs to collect a history prior to starting a care plan. Which fact should the APRN understand about the Filipino culture prior to collecting this data? Family members will provide opinions about care without prompting. Handshakes are discouraged as a greeting. Patients prefer to be called by their first name. Providers should sit, attempt to maintain eye contact, and allow periods of silence.

D

57 A patient with chronic kidney disease is scheduled for a permanent A/V fistula placement. The patient is experiencing intra-dialysis hypotension. During the physical exam, the advanced practice registered nurse (APRN) notes that the patient has a systolic ejection murmur. Which order should the APRN make prior to this patient's surgery? Stress test Potassium level Fistulogram Echocardiogram

D

58 An advanced practice registered nurse (APRN) is working with an integrated multidisciplinary team and recognizes that team processes need to be strengthened. The APRN suggests a strategy to meet that goal. Which strategy should the APRN use to meet this goal? Introducing value-based interviewing Initiating cross-training Identifying clear patient outcomes Instituting clinical case review

D

5.What are the common symptoms of benign prostatic hyperplasia (BPH)? Dribbling, hesitancy, loss of stream volume and force, and recurrent urinary tract infections Dysuria, urgency, frequency, nocturia, and suprapubic heaviness or discomfort Obstructive symptoms such as a weak urine stream, abdominal straining to void, hesitancy, incomplete bladder emptying, and terminal dribbling

Feedback: Obstructive symptoms are common in BPH. The other symptoms are associated with different diagnoses. Acute onset of fever, chills, flank pain, head ache, malaise, CVA tenderness, and possibly hematuria

1.A patient diagnosed with chronic pain calls to request an oxycodone (Oxycontin) refill. Which action should the prescriber take initially? Fax the renewal order to the pharmacy. Arrange to schedule an appointment with the patient. Verify the patient's adherence to the prescribed drug regimen. Determine the patient's current medication dosage and pain level.

Feedback: Ongoing evaluation of the benefits and risks of opioid use is an element of the CDC pain guidelines.

11.The FNP is reviewing the lab results of a 61-year-old patient recently seen for a pap smear. Results are: atrophic changes, scent endocervical cells, and adequate smear. She has been treated for breast cancer with mastectomy and tamoxifen. She has never received hormone replacement therapy. What is appropriate for the FNP to tell the patient? Your pap smear is slightly abnormal. I would recommend the use of some estrogen vaginal cream nightly for 3 weeks, then return to the office to have the pap smear repeated. Your pap smear is normal but shows some mild thinning of the tissue. This is to be expected in someone who is postmenopausal and not on hormones. It does not pose a threat to your health. Please return to the office in 1 year for your annual exam or sooner if needed. Your pap smear shows that you don't have enough endocervical cells. Please make an appointment for endocervical curettage. Your pap smear is abnormal. This could signify a disease state of the cervix. Please schedule a colposcopy at your earliest convenience.

Feedback: Atrophic changes on the cervix of a postmenopausal woman are to be expected, as is the paucity of endocervical cells. Because of her past medical history, she is not a candidate for HRT, and the pap smear results are not abnormal.

A patient is diagnosed with BPH. Which medication should be recognized by the FNP as likely to aggravate this condition? glyburide oral buspirone inhaled ipratropium ophthalmic timolol

Feedback: BPH is a common cause of urinary retention in older men. Inhaled ipratropium is an atropine-like bronchodilator used to treat chronic bronchitis, its anticholinergic properties may aggravate urinary retention.

8.Which two patients should have a Pap smear test performed by the FNP? An 18-year-old female who reports sexual activity with multiple partners. A 45-year-old female patient who denies sexual activity but has two children. A 21-year-old female who denies sexual activity. Feedback: Based on USPSTF guidelines, pap smear tests should be performed on female patients aged 21 and older, regardless of sexual activity. A 16-year-old patient who denies sexual activity.

Feedback: Based on USPSTF guidelines, pap smear tests should be performed on female patients aged 21 and older, regardless of sexual activity.

The FNP would note which finding as a possible sign of preeclampsia? Urinary urgency at night Edema in all extremities and a puffy face Stomach cramps Clear fluid discharge from the nipples

Feedback: Classic signs of pre-eclampsia are hypertension and proteinuria. Generalized edema of the extremities and around the face occurs due to increased permeability and capillary leakage.

What is the purpose of clinical research trials in the spectrum of translational research? Determination of the basis of disease and various treatment options Adoption of interventions and clinical practices into routine clinical care Exploration of fundamental mechanisms of biology, disease, or behavior Examination of safety and effectiveness of various interventions

Feedback: Clinical research trials are concerned with determining the safety and effectiveness of interventions. Adoption of interventions and practices is part of clinical implementation. Determination of the basis of disease and treatment options is part of the preclinical research phase. Exploration of the fundamental mechanisms of biology, disease, or behavior is part of the basic research stage.

7.A 41-year-old patient is seen for her 6-week postpartum exam by the FNP. She is breastfeeding without difficulty and plans to continue for a year. She wants to begin using contraception and plans no further pregnancies. Which of the following is not an appropriate choice for this patient? Depo-Provera 150mg IM Q 3 months IUD Progestin only oral contraceptive Combination OC

Feedback: Combination OCs are not recommended for breastfeeding mothers because of the effect of estrogen on milk supply. Progestin only OCs, IUDs, and Depo-Provera are acceptable methods of contraception for breastfeeding mothers.

What are the common side-effects of phosphodiesterase-5 inhibitors? Erection lasting longer than 4 hours. Headaches, flushing, and dyspepsia. Nausea and vomiting. Rash, itching, and loss of appetite.

Feedback: Common side effects include headache and flushing caused by vasodilation.

Which is an abnormal complaint in the second trimester of pregnancy? Frequent uterine contractions Frequent fetal movement Calf cramps Heartburn

Feedback: Contractions can represent preterm labor and should always be evaluated to rule out early cervical change.

The largest category of psychiatric disorders in elderly patients is: Panic disorder Generalized anxiety disorder Depression Eating disorders

Feedback: Depression remains the largest category of psychiatric disorders in older adults and is associated with significant morbidity and mortality.

A patient has been taking doxazosin 2 mg PO daily for 3 weeks for treatment of BPH. He returns to the clinic and is complaining of feeling dizzy when he stands up. Which action would the FNP take? Determine BP when lying, sitting, and standing. . Order urinalysis to determine hematuria and presence of bacteria. Review with patient his symptoms over the last 3 weeks. Perform digital rectal exam to determine whether prostate is smaller than previously noted.

Feedback: Doxazosin is also used as an antihypertensive, the patient may be experiencing orthostatic hypertension

A 74-year-old male patient has BPH and stage one hypertension. Which of the following medications would be the most appropriate selection to treat both disorders? tamsulosin finasteride doxazosin tadalafil

Feedback: Doxazosin is indicated as a treatment in both BPH and hypertension because of its favorable side effect.

2.During pregnancy, what is estrogen responsible for? Hyperpigmentation Facilitating implantation Reducing smooth muscle tone Decreased uterine contractility

Feedback: During pregnancy, estrogen is responsible for stimulation of the melanin-stimulating hormone, resulting in hyperpigmentation. The other factors are related to progesterone.

Which of the following is a common cause of erectile dysfunction? The use of antihypertensives. Dietary supplements Masturbation It is a natural part of aging.

Feedback: Erectile dysfunction can be commonly caused by the use of antihypertensives, particularly thiazide diuretics and beta blockers.

19.For the patient who wants to breastfeed and take oral contraceptives, what is the pill of choice? 1/35 preparation Triphasic preparation Progestin-only preparation 1/50 preparation

Feedback: Estrogen inhibits milk production. Progestin only pills are ideal for breastfeeding because they do not contain estrogen and will not impact milk production.

A 46-year-old female patient is being seen in the clinic by the FNP. She was last seen 2 weeks ago for an upper respiratory tract infection and was treated with amoxicillin 250mg PO TID x 10days. She completed her medication last week, but now complains of vaginal itching and thick white discharge. She states that she has never experienced such intense itching. She is in a mutually monogamous relationship. Her LMP was 2 weeks ago. Her partner had a vasectomy. Wet mount shows negative whiff test, rare clue cells, positive lactobacilli, positive hyphae, positive spores, few WBCs, and no trichomonads. She is leaving tomorrow for a week long cruise. She is not taking any medications and has no known drug allergies. The FNP should prescribe which of the following? metronidazole 500mg PO BID x 7days clindamycin vaginal cream one applicator full vaginally at HS x 7days fluconazole 150mg 1 tab PO x1 dose terconazole vaginal cream 1 applicator @ HS x 7 days.

Feedback: Fluconazole is approved for a single-dose oral treatment of uncomplicated vulvovaginal candidiasis. It is the most convenient treatment for this patient who will is unlikely to be compliant with vaginal creams given the upcoming travel.

10.The FNP understands that all of the following factors contribute to the process of aging, except: A decrease in hormone production A decline in cell-mediated immunity A decrease in free radicals of oxygen A decrease in the production of B-cells and T-cells

Feedback: Free radicals of oxygen do not have a paired electron. The increase of "free" molecules are thought to be the consequence of oxidative stress and result in cellular damage, as well as influencing the aging process of the body.

.During the regular prenatal visits, what assessment data other than vital signs and weight are determined with each visit? Fundal height, fetal heart rate, urine dip for protein and glucose, and presence of edema Urinalysis, glucose screen, fundal height, and fetal heart rate Presence of/changes in Chadwick's sign, CBC, and blood glucose screening Pelvic measurements, fundal height, urinalysis, and CBC

Feedback: Fundal height, fetal heart rate, urine dip and assessment of edema are performed with each prenatal visit. The other assessments are not done at every visit.

11.Which is an abnormal complaint in the third trimester of pregnancy? leukorrhea headache with blurred vision urinary frequency uterine contractions

Feedback: Headache associated with visual changes can represent early symptoms of pre-eclampsia and warrants further evaluation.

5.The FNP is seeing a 68-year-old patient for their annual wellness exam. The FNP knows that screening for hearing deficits includes which of the following: Referral to an audiologist Whisper test Otoacoustic emissions Behavioral Audiometry Evaluation

Feedback: Hearing impairments affect up to 1/3 of persons over age 65. Hearing deficits can be screened for by the whisper voice test.

What is the provider's ideal goal of drug rehabilitation? Abstinence from the drug Decreasing episodes of relapse Minimizing drug cravings Reduction of drug use

Feedback: Ideally the goal of treatment is complete cessation of drug use.

4.A male patient presents with a complaint of sexual dysfunction. The FNP understands that sexual dysfunction is an impairment of: Erection only Emission only Ejaculation only Erection, emission, or ejaculation

Feedback: In sexual dysfunction, erection, emission, or ejaculation may not be functioning because of multifactorial causes such as medications, vascular disorders, neuropathy, and trauma.

4.The FNP understands that which of the following patient populations should be screened for intimate partner violence: Unmarried women Adolescents Patients with a history of multiple partners All patients

Feedback: It should be routine practice to screen all patients as a part of the social history. Ideally this should be done when the patient is alone.

8.Which medication would be considered safe to use in all trimesters of pregnancy? metronidazole tetracycline isotretinoin ACE inhibitors

Feedback: Metronidazole is safe to use in all trimesters of pregnancy. The other medications are known teratogens and contraindicated in pregnancy.

6.A 25-year-old patient presents with complaints of malodorous vaginal discharge, which is white and watery. She douches with vinegar and water every 2 weeks. She uses a diaphragm for contraception and has been sexually active with her boyfriend for two years, using condoms for STD prevention. Her LMP was 1 week ago, and there are no noted changes in her normal menstrual pattern. Her wet mount shows a positive whiff test, clue cells too numerous to count, no lactobacilli, no hyphae, no spores. What is the diagnosis and treatment for this patient? chlamydia: doxycycline 100mg PO BID x 10days candida albicans: terconazole vaginal cream 1 applicator HS x 7days HSV type 2: acyclovir 200mg PO q4h X 5days bacterial vaginosis: metronidazole vaginal gel 1 applicator HS x 5 days.

Feedback: Metronidazole vaginal gel is the treatment of choice for bacterial vaginosis in the non-pregnant female. The presence of clue cells, and the associated malodorous discharge and absence of lactobacilli are markers for the diagnosis of bacterial vaginosis.

A young adult female, G2P1A0L1, 10 weeks gestation with intrauterine pregnancy is seen for her first intake and history visit. She knows when she conceived and denies any vaginal bleeding or abdominal pain. The patient has a soft, non-tender fundus that measures 14cm; adnexal exam negative for masses or tenderness, no FHR audible with doppler. What is the most likely diagnosis seen on ultrasound? Multiple gestation Fibroid uterus Ectopic pregnancy 14-week viable IUP

Feedback: Multiple gestation pregnancies will cause the uterus to enlarge faster than normal. FHR may be inaudible with the doppler due to the 10 week gestation.

15.A new first-time mother is being evaluated for a complaint of breast pain. Her infant is 3-weeks-old and she is breastfeeding. The infant is gaining weight and seems satisfied after feeding. On exam, the FNP finds red, irritated nipples on both breasts but no masses or tenderness to the breasts themselves. What is the most important part of the FNPs continued evaluation? Mammogram of the breast. Exam of the infant's mouth. STAT CBC Analysis of the milk.

Feedback: Nipple irritation in breastfeeding mothers is often caused by candida albicans. The source is usually the infant's mouth (thrush).

Which is NOT a criterion for the diagnosis of bacterial vaginosis? Positive amine (whiff) test. Presence of clue cells. Vaginal pH greater than 4.5. Presence of pseudohyphae.

Feedback: The criteria for diagnosis of bacterial vaginosis are the characteristic milky homogenous discharge, pH greater than 4.5, amine odor with addition of potassium hydroxide, and presence of epithelial cells studded with coccobacilli that obscure borders (clue cells). Pseudohyphae are present in candidiasis.

An 80-year-old woman who lives alone is noted to have a recent weight loss of 5 pounds. She appears somewhat confused, according to her daughter, who is concerned that she is developing dementia. The provider learns that the woman still drives, volunteers at the local hospital, and attends a book club with several friends once a month. What is the initial step in evaluating this patient? Ordering a CBC, serum ferritin, and TIBC Referring the patient to a dietician for nutritional evaluation Referring the patient to a neurologist for evaluation for AD Obtaining a CBC, serum electrolytes, BUN, and glucose

Feedback: Patients with weight loss, confusion, and lethargy are often dehydrated and this should be evaluated by looking at Hgb and Hct, electrolytes, and BUN. This patient is currently leading an active life, so the likelihood that recent symptoms are related to AD, although this may be evaluated if dehydration is ruled out. Anemia would be a consideration when dehydration is ruled out. Referrals are not necessary unless initial evaluations suggest that malnutrition or AD is present.

2.A patient has taken a narcotic analgesic for chronic pain for several months. At a follow-up appointment, the provider notes that the patient has been taking more than the prescribed dosage. The patient has normal vital signs, is awake and alert, and reports mild pain. What does the provider suspect is responsible for the patient's response? This patient exhibits a negative placebo effect with a reduced response to the drug. This patient has developed tachyphylaxis because of repeated exposure to the drug. This patient has developed pharmacodynamic tolerance, which has increased the minimal effective concentration (MEC) needed for analgesic effect. This patient has increased hepatic enzyme production as a result of prolonged exposure to the drug.

Feedback: Pharmacodynamic drug tolerance is the drug tolerance that develops from the drug being in your body for so long that it gets use to the drug and becomes less sensitive to its presence in your body. This usually happens after extended use of drugs.

Which of the following is correct regarding short-acting phosphodiesterase-5 inhibitors, such as sildenafil or vardenafil? They work best in combination with a large fatty meal. They should be taken 30 minutes to 1 hour before intercourse. They work in all men.

Feedback: Phosphodiesterase-5 inhibitors should be taken 30-60 minutes prior to intercourse. Fatty foods interfere with absorption. They are the only class of medications that is effective for treating ED.

What is a positive sign of pregnancy? Softening of the cervix Fetal heartbeat Enlargement of the uterus and abdomen Mother's perception of fetal movement

Feedback: Positive evidence of pregnancy includes fetal heartbeat, palpation of fetal movement by the provider, and visualization of the fetus by ultrasonography.

An adult patients LMP was 2 months ago. She has had and IUD in place for the last 4 months. She is complaining of nausea, fatigue, breast tenderness, and abdominal bloating. Physical exam reveals the following: Abdomen- no abnormalities noted; Pelvic- cervix with positive Chadwick's sign, IUD strings protruding from cervix; Uterus- enlarged and non-tender; Adnexa- non-tender, without masses and no CMT. What would be the likely diagnosis? Uterine fibroid Ovarian cancer Dislodged IUD Pregnancy

Feedback: Pregnancy is the most likely diagnosis, given the symptoms and exam findings.

An older adult patient with a history of forgetfulness will need to take multiple drugs after discharge from the hospital. What provider action will most successfully promote medication adherence in a forgetful patient? Ask the patient to share the medication teaching with a neighbor or friend soon after discharge. Give the patient detailed written information about each drug. Schedule medications to be taken at the same times as much as possible. Make sure the patient understands the actions and side effects of each drug.

Feedback: Prescription regimen adherence can be promoted through simplifying the schedule of medications.

1.How does progesterone affect the GI system during pregnancy? Causes nausea and vomiting early in pregnancy Causes hypertrophy and bleeding of the gums Delays gastric emptying time and decreases intestinal peristalsis Causes diarrhea caused by increased intestinal peristalsis

Feedback: Progesterone affects the GI system by decreasing smooth muscle tone, delaying gastric emptying, and decreasing intestinal peristalsis.

A young woman is complaining of tenderness and burning of her vulva. On exam, the vulva is edematous and excoriated. The FNP performs a wet mount preparation of vaginal secretions. It reveals pseudohyphae and spores. What is the diagnosis for this patient? vulvovaginal candidiasis

Feedback: Pseudohyphae and spores on the wet mount with potassium hydroxide are diagnostic for candida infection. chlamydial infection bacterial vaginosis gonorrhea

At an initial prenatal visit occurring in the first trimester, which blood test is NOT recommended? Antibody screen Rubella Maternal serum alpha-fetoprotein Hepatitis B surface antigen

Feedback: Routine laboratory tests include CBC, blood type and Rh, antibody screen, HBSAg, syphilis screen, and rubella immune titer. The MSAFP is done between 15 and 20 weeks and is time-sensitive.

A 69-year-old female patient is being seen for her annual wellness exam. She complains of an increasing number of purple spots on the dorsal surface of her hands. The patient denies any recent falls or injuries. The FNP understands that the patient is likely experiencing: xerosis lentigos senile purpura bullous pemphigoid

Feedback: Senile purpura is a common, benign condition characterized by the recurrent formation of purple ecchymoses (bruises) on the extensor surfaces of forearms following minor trauma.

What intervention will the provider implement when prescribing medications to an 80-year-old patient? Beginning with higher doses and decrease according to the patient's response Reviewing all patient medications at the annual health maintenance visit Consulting the Beers list to help identify potentially problematic drugs Ensuring that the patient does not take more than five concurrent medications

Feedback: The Beers list provides a list of potentially inappropriate medications in all patients aged 65 and older and helps minimize drug-related problems in this age group. Older patients should be started on lower doses with gradual increase of doses depending on response and side effects. Patients who take five or more drugs are at increased risk for problems of polypharmacy, but many will need to take more than five drugs; providers must monitor their response more closely. Medications should be reviewed at all visits, not just annually.

10.The FNP is reviewing the lab results of a 28-year-old patient recently seen for a pap smear. Classification is high-grade squamous intraepithelial lesion, endocervical cells seen, and adequate smear. The FNP phones the patient and tells her which of the following? Your pap smear was normal. Follow up in one year or sooner if problems arise. Your pap smear shows invasive cancer. I would like you to see a gynecologic oncologist for treatment. Your pap smear shows abnormal tissue that needs to be evaluated. Please schedule an appointment for a colposcopy. Your pap smear shows a minor abnormality. Sometimes this can signify a disease process that is just beginning. Please schedule a follow up pap smear in 4 months.

Feedback: The pap smear is a screening test for cervical cancer and precancerous states. The diagnostic test needed to confirm the diagnosis of a high-grade lesion is a colposcopy with guided biopsies.

A 28-year-old man presents with complaints of fever, low back pain, perineal pain, and intense pain on voiding. Rectal exam reveals a tender, swollen, firm, warm prostate. Based on the patient's symptoms, what is the treatment of choice? ceftriaxone 250mg IM x1 followed by doxycycline 100mg PO BID x 10 days tetracycline 250mg PO qid x 10 days amoxicillin 500mg PO tid x 14 days erythromycin 250mg PO q6h x 24 days

Feedback: The patient is presenting with classic symptoms of acute bacterial prostatitis. The treatment of choice is IM ceftriaxone plus oral doxycycline.

A 70-year-old man complains of scrotal pain with dysuria and frequency that has been increasing over the last 2 weeks. Physical exam reveals extreme scrotal tenderness and swelling, urethral discharge, and testes normal in size and position. Urinalysis reveals pyuria. What is the treatment of choice for this patient? Nitrofurantoin 100mg PO QID x 14 days Levofloxacin 750mg PO QD x 10 days Doxazosin 1mg PO qd x 10 days Oxybutynin 5mg PO TID x 10 days

Feedback: The patient is presenting with classic symptoms of epididymitis. The treatment of choice is a fluoroquinolone, such as levofloxacin or ciprofloxacin.

12.The FNP is assessing the skin and nails of a 76-year-old patient. The FNP notes that the patient's nails have a spoon like appearance. Which of the following laboratory tests would the FNP order? Arterial blood gases CBC Liver function tests Serum electrolytes

Feedback: The spoon like appearance of nails is associated with iron deficiency anemia.

The FNP is assessing a patient who has a positive pregnancy test. Laboratory data indicate that the mother's blood type is O positive and the father's blood type is AB negative. What risk is associated with this pregnancy? The mother may build up antibodies to the infants blood if the infant is blood type B, which will be significant in future pregnancies. The mother is Rh positive, if the infant is Rh negative, there is an increased incidence of the infant building up antibodies. Because the mother is O and the father is AB, there is an increased risk of ABO incompatibility leading to jaundice in the newborn. Type O blood is the dominant characteristic. The baby's blood type will be O and there are no associated risks.

Feedback: There is an increased risk of ABO incompatibility if the mother is type O and the father is AB, A, or B.

A 74-year-old female patient is brought in by her daughter for evaluation. The patient is a widow and has lived on her own since her husband died 4 years ago. The daughter states that her mother has been increasingly forgetful, sleeping more than normal, agitated, and recently suffered a fall in the bathroom. The daughter also states that her mother has not eaten the meals that she left for her 3 days prior. The FNP should consider which of the following as a priority? CT scan or MRI Screening for cognitive impairment. Urinalysis with culture Nutritional Assessment

Feedback: UTIs may not cause classic symptoms in older adults. Instead, older adults, may experience behavioral symptoms such as confusion and agitation, in addition to lethargy and a change in oral intake.

The FNP teaches a prenatal patient that a significant source of toxoplasmosis is: Rare red meat Fresh fruits Raw oysters Raw vegetables

Feedback: Undercooked red meat is a major source of toxoplasmosis. Pregnant women should be cautioned against eating undercooked meats. Cats are also known to be hosts. Toxoplasmosis is spread through cat feces. Pregnant women should be educated about not cleaning the litter box and about avoiding contaminated soil

An advanced practice registered nurse (APRN) is examining a 92-year-old patient for new complaints associated with a urinary tract infection. The APRN is creating a plan of action to treat this patient while not overly adding to the pill burden. Which action should the APRN take for this patient? Change the medication to liquid form Recommend splitting pills Spread the dosage over q8h Recommend a once-a-day medication treatment Correct!

It is best to prescribe medications that are taken once a day when possible, so the pill burden is not increased.

The patient is asked to bear down (Valsalva maneuver) or to cough to demonstrate

The patient is asked to bear down (Valsalva maneuver) or to cough to demonstrate any stress incontinence.

i. Trichinosis

The pregnant woman receives a single 2-g oral dose, and her partner can be treated with either a single 2-g oral dose or 500 mg of oral metronidazole twice daily for 7 days

Cytomegalovirus

a. i. Cytomegalovirus (CMV) is the most common intrauterine viral infection. ii. The transmission is via contaminated urine, blood, saliva, semen, or cervical secretions. iii. A vaccine is available but should not be administered during pregnancy. iv. The highest rate of transmission is in the third trimester but the severity of fetal effects is highest in the first trimester. The infection may also be "reactivated" during pregnancy v. Perinatal fetal effects include jaundice, thrombocy vi. The "blueberry muffin baby" has been described with the appearance caused by numerous petechiae on the skin. Amniotic fluid is positive (by PCR) for CMV with active infection, and for 4 to 8 months maternal IgM antibodies topenia, intrauterine growth restriction (IUGR), and microcephaly. vii. . Ganciclovir and valacyclovir have been used in nonpregnant women and in neonates after birth

Smell

a. i. Decreased olfactory neurons 1. Olfactory loss is a common cause of taste disturbances, but in addition to affecting smell, medications can also cause dry mouth (xerostomia) or persistent taste dysfunction (dysgeusia). Dental caries, poorly fitting dentures, gingivitis, infections, and diseases also cause oral dryness or decrease taste.

SLE

a. i. Systemic lupus erythematosus (SLE) is a chronic autoimmune disease with multiple organ system involvement in which patients are subject to acute exacerbations or flares. Associated antibodies include antinuclear, anti-ribonucleoprotein (anti-RNP), and anti-Smith (anti-Sm) antibodies. 1. Anti-double-stranded DNA (anti-dsDNA) antibody is associated with nephritis and lupus activity. 2. Anti-Ro/SSA and anti-La/SSB are present in Sjögren syndrome and neonatal lupus with heart block. 3. Antihistone antibody is common in drug-induced SLE. ii. best pregnancy outcomes occur if the disease has been under good control for at least 6 months before conception and there is no evidence of active lupus nephritis. iii. increased rate of miscarriage and intrauterine fetal death, especially when associated with antiphospholipid antibodies, as well as preterm delivery and fetal growth restriction iv. Neonatal lupus is associated with passive transmission of anti-Ro/SSA or anti-La/SSB antibodies and is one of the main causes of congenital heart block.

GD Diagnosis

a. i. The first step involves universal screening for gestational diabetes between 24 and 28 weeks' gestation with a 50-g, 1-hour oral glucose challenge test (OGCT), given without regard to most recent oral intake. ii. If a first-trimester screen is done and is found to be negative, it should be repeated at 24 to 28 weeks. Glucose values above 130 to 140 mg/dL on an OGCT are considered abnormal and have an 80-90% sensitivity in detecting GDM. iii. The second step involves performing a diagnostic 3-hour, 100-g oral glucose tolerance test (OGTT) if the screening test is abnormal 1. Fasted 100 of glucose check BS hourly Fasting 95 1 hr 180 2 hr 155 3 hr 140mal. b. If there are two or more abnormal values on the 3-hour OGTT, the patient is diagnosed with GDM (Table 16-3). If the 1-hour screening (50 g of oral glucose) plasma glucose exceeds 200 mg/dL, an OGTT is not required and may dangerously elevate blood glucose values.

Jaundice

a. i. assess the baby during the most vulnerable period, 3 to 5 days of age. The AAP has developed hyperbilirubinemia phototherapy treatment guidelines based on the rate of rise of the bilirubin levels plotted against the baby's age in hours.

IUGR

a. i. is defined as poor fetal growth during pregnancy and reduced size at birth. With IUGR, the infant birth weight is less than the 10th percentile and about 25% of preterm infants are in this category. The causes of IUGR are maternal, placental, and fetal. The main maternal risk factors are poor nutrition, smoking, and metabolic diseases such as hypertension and diabetes. Placental factors are mostly related to failure of proper implantation during early pregnancy. More recently, metabolic dysregulation in women with diabetes has been shown to adversely affect placental function and lead to poor fetal growth. Fetal factors are mostly limited to chronic fetal infections and abnormal development.

Anxiety

a. i. physiologic neurobiological disconnect between the amygdala and frontal areas of the brain ii. Thoughts of aging iii. Loss of support Finances, care Substance use 1. Effective and safer medication categories include antidepressants (e.g., serotonin reuptake inhibitors [SSRIs] and serotonin-norepinephrine reuptake inhibitors [SRNIs])

1. S/S maternal hythrd

a. .A resting pulse rate greater than 100, a wide pulse pressure, tremor, eye changes (exophthalmos), failure to gain weight despite normal or increased food intake, and heat intolerance, 2. Labs a. Elevated T4 and low/suppressed TSH 3. Treatment a. Atenolol of propranolol short term for high BP b. Thioamides (mainstay treatment) i. PTU should be used to treat overt hyperthyroidism in the first trimester ii. Methimazole should be used in the second and third trimesters. iii. Methimazole is preferred over PTU in breastfeeding mothers because of the lower risk of liver toxicity.

1. Differentiate the types of assessment of fetal well-being.

a. A combination of maternal self-assessment, nonstress testing (NST), and real-time ultrasonic assessment is used to evaluate fetal well-being. b. A simple technique (kick counting) i. used to assess fetal well-being. The mother assesses fetal movement (kick counts) each evening while lying on her left side. She should recognize 10 movements in 1 hour and if she does not, she should retest in 1 hour. If she still does not have 10 fetal movements in 1 hour, c. Nonstress test i. the mother resting in the left lateral supine position, a continuous fetal heart rate tracing is obtained using external Doppler equipment 1. A normal fetus responds to fetal movement with acceleration in fetal heart rate of 15 beats or more per minute above the baseline for at least 15 seconds (Figure 7-3). If at least two such accelerations occur in a 20-minute interval, the fetus is regarded as being healthy, and the test is said to be reactive. d. Ultrasonic assessment i. Reduced fluid (oligohydramnios) suggests fetal compromise. Oligohydramnios can be defined as an amniotic fluid index (AFI) of less than 5 cm. 1. When amniotic fluid is reduced, the fetus is more likely to become compromised as a result of umbilical cord compression. ii. Excessive amniotic fluid (polyhydramnios; AFI > 23 cm) can be a sign of poor control in a diabetic pregnancy iii. Fetal breathing (chest wall movements) and fetal movements (stretching and rotational movements) are also used to assess the fetus. A fetus that has at least 30 breathing movements in 10 minutes or 3 body movements in 10 minutes is considered healthy.

1. Discuss breast cancer

a. Etiology i. The incidence and mortality rates for breast cancer are approximately five times higher in North America and northern Europe than they are in many Asian and African countries. ii. About 5-10% of breast cancer cases are hereditary and result from mutations in the BRCA1 or BRCA2 gene. These genetic mutations also increase the risk of ovarian cancer. Hereditary breast cancer is particularly common in premenopausal women. b. Risk factors c. Types of tumors d. Staging e. Clinical features i. Extension to the skin may cause retraction and dimpling, whereas ductal involvement may cause nipple retraction. Blockage of skin lymphatics may cause lymphedema and thickening of the skin, a change referred to as peau d'orange f. Treatment i. surgery g. Prognosis i. the status of the axillary lymph nodes has been considered to be the single most important prognosticator. ii. In the National Surgical Adjuvant Breast Project, patients with negative lymph nodes had an actuarial 5-year survival of 83%

1. Discuss the process of pre-surgical clearance.

a. Functional status and frailty b. Cognition c. Sensory impairment d. Mental health e. Medication management (including indication, dose, adverse events, and adherence) f. Pain management g. Cardiovascular risk stratification h. Pulmonary risk assessment i. Obstructive sleep apnea i. 30 days prior to surgery ii. Medication reconciliation j. Review of systems 1. simple screening of cardiac functional status may be done by simply asking if the patient can walk four blocks or climb two flights of stairs 2. exercise intolerance, cough, or unexplained dyspnea as these symptoms may suggest undiagnosed heart failure or chronic lung disease k. physical exam i. height, weight, and overall general appearance and functional status of the patient. Evaluation of mental status, airway, dentition, and range of motion of the head and neck is also necessary. Any abnormalities of the appearance of neck veins or the presence of bruits, as well as any abnormalities on auscultation of the heart, lungs, and abdomen should be noted. Further evaluation for any abdominal masses, genitourinary or rectal problems, of peripheral pulses, cranial nerves, and for any neurologic changes may also be warranted depending on the reason for presentation, the medical history, and the anesthetic plan

a. Genital warts

a. Genital warts i. Definition: viral lesion caused by HPV commonly found in anogenital area 1. Increases risk of vulvar and cervical cancers 2. Most common STI; highly contagious 3. Lesions may occur up to 3 to 6 mo after exposure 4. Condoms may decrease the risk but will not be protective because the virus infects areas not covered by condoms 5. Can be transmitted vertically during birth

1. How is gestational age estimated and the EDC calculated?

a. Gestational age should be determined during the first prenatal visit. b. Estimated date of confinement (EDC) or "due date" may be determined by adding 9 months and 7 days to the first day of the last menstrual period. Ultrasonography may also be used to estimate gestational age. Measurement of fetal crown-rump length between 6 and 11 weeks' gestation can define gestational age to within 7 days. At 12 to 20 weeks, gestational age can be determined within 10 days by the average of multiple measurements

1. Outline the elements of the Well Woman Physical (refer to your women's health SOAP note from D117) e cervix, uterus, and adnexa.

a. History b. Physical Exam i. (1) inspection of the external genitalia, vagina, and cervix; (2) collection of cytologic specimens from the exocervix (or ectocervix) and superficial endocervical canal; and (3) palpation of th

1. Outline the elements of the Well Woman Physical (refer to your women's health SOAP note from D117)

a. History b. Physical Exam i. (1) inspection of the external genitalia, vagina, and cervix; (2) collection of cytologic specimens from the exocervix (or ectocervix) and superficial endocervical canal; and (3) palpation of the cervix, uterus, and adnexa.

1. Who is eligible for hospice care?

a. If you have Medicare Part A (Hospital Insurance) AND meet all of i. Your hospice doctor and your regular doctor AGREE that you're terminally ill (you're expected to live 6 months or less ii. You accept palliative care for comfort instead of care to cure your illness. iii. You sign a statement choosing hospice care instead of other Medicare-covered treatments for your terminal illness and related conditions.

1. hormone therapy and gender-affirming surgeries.

a. In order to avoid the development of undesired secondary sex characteristics and delay the onset of puberty, a model protocol was developed in the Netherlands that includes gonadotropin-releasing hormone (GnRH) analogues b. Among transgender women who use estrogen, there is the possibility of excess CV risk due to estrogen therapy. But this risk may be mitigated by using transdermal rather than oral estrogen treatment.64 When stratifying CV risk for a transgender patient, the primary care provider may wish to consider using the risk estimator associated with the sex hormones to which the individual has had the longest exposure. For example, a 50-year-old transgender woman who began estrogen supplementation in her 40s would use the male risk assessment. A 50-year-old transgender woman who began estrogen in her 20s would use the female risk assessment due to longer exposure to estrogen. i. Two important principles to understand when caring for transgender people are that surgery is not an endpoint and inquiries regarding gender-affirming surgeries or the status of one's genitalia can be traumatic. Discussions around an individual's genitourinary anatomy should be limited to times when it is relevant to care being rendered

1. Discuss the treatment of seizure disorder during pregnancy.

a. In patients who have had no seizure activity for at least 2 years, AED therapy can be discontinued before conception. b. Valproic acid should probably not be used in a woman planning a pregnancy, c. women of reproductive age who are taking AEDs are advised to take at least 0.4 to 0.8 mg of folic acid daily to protect against neural tube defects should they become pregnant. d. Status epilepticus i. intravenous lorazepam should be given slowly, followed by a loading dose of phenytoin with continuous cardiac monitoring. ii. The management of labor and delivery follows obstetric indications. During labor and in the immediate postpartum period, anticonvulsant drugs must be continued. The dose of the anticonvulsant drug may be lowered postpartum, provided that a therapeutic level is maintained. iii. breastfeeding is not contraindicated.

1. What is the recommendation for the interval of follow up visits during pregnancy? Identify the assessments performed during these visits.

a. Prenatal visits should be scheduled every 4 weeks until 28 weeks' gestation, every 2 to 3 weeks until 36 weeks, and then weekly until delivery b. blood pressure, weight, urine protein and glucose, uterine size for progressive growth, and fetal heart rate. After the woman reports quickening (first sensation of fetal movement, on average at 20 weeks' gestation) and at each subsequent visit, she should be asked about fetal movement. Between 24 and 34 weeks, women should be taught the warning symptoms of preterm labor (uterine contractions, leakage of fluid, vaginal bleeding, low pelvic pressure, or low back pain). c. Beginning at 28 weeks, systematic examination of the abdomen should be carried out at each prenatal visit to identify the lie (e.g., longitudinal, transverse, oblique), presentation (e.g., vertex, breech, shoulder), and position (e.g., flexion, extension, or rotation of the occiput) of the fetus. d. screening for gestational diabetes should be performed between 24 and 28 weeks' gestation. i. Risk factors for selective screening include family history of diabetes; previous birth of a macrosomic, malformed, or stillborn baby; hypertension; glycosuria; maternal age of 30 years or older; or previous gestational diabetes. e. universal screening for maternal colonization of group B streptococcus at 35 to 37 weeks' gestation.

1. What is the RADAR checklist?

a. R: Remember to always ask about partner violence or sexual abuse in your practice. b. A: Ask directly and clearly (in a private setting) about violence and abuse with questions such as, "At any time, has someone you live with hit, kicked, or otherwise physically abused or frightened you?" c. D: Document all information about "suspected violence and abuse" in the patient's chart and file reports in accordance with local laws. d. A: Assess the patient's safety, whether there are weapons in the house, and if any children are in danger. e. R: Review possible options that your patient may have to increase safety, such as shelters and support groups.

Rubella

a. Rubella i. Vaccination has reduced the rate of infection in pregnant women with a reported incidence of <0.1% of pregnancies. ii. The vaccine should not be given during pregnancy. iii. spreads by respiratory droplets, and has an incubation period of 2 to 3 weeks. The symptoms are malaise and myalgia in the presence of a nonpruritic, maculopapular, reddish rash. iv. Deafness, retinopathies, and central nervous system and cardiac malformations are the most common teratogenic manifestations. No treatment is available for rubella, v. but prevention using the measles, mumps and rubella (MMR) vaccine is strongly recommended.

1. Discuss the 4 Principles of Practice

a. Safety is paramount b. Adhere to ethical practices c. Value base system health care delivery d. Prevention and early migration of disease

1. Review the key elements of SOAP notes.

a. Subjective i. What they tell you b. Objective i. Labs, assessment, c. Assessment i. d. Plan i. Very specific 20 to 10 a week 1. Write a story, thought through assessment make it make sense, specific plan. 2. Legal document, collaboration contract between you in the patient

testicular tumor

a. T i. Germ cell tumors (GCTs) are the most common type of testicular tumor and account for 90% to 95% of all primary malignant neoplasms. ii. Stromal tumors are rare and usually consist of Leydig and Sertoli cell tumor types. 1. GCTs are associated with serum tumor marker products alpha fetoprotein (AFP), human chorionic gonadotropin (hCG), and lactate dehydrogenase (LDH) and are critical to diagnosing, prognosing, staging, and monitoring treatment response of testicular cancer. 2. Any solid, firm mass within the body of the testicle should be considered a tumor unless proven otherwise iii. Treatment 1. Prompt evaluation is essential. The mainstays of testicular cancer treatment are surgery, chemotherapy, and radiation 2. Additional management is monitoring of tumor markers hCG, AFP, and LDH. Follow-up visits should include a thorough physical examination, chest imaging, and measurement of serum tumor markers monthly for the first year, every 2 months for the second year, and every 3 to 6 months for up to 5 years. Regardless of the disease stage, more than 90% of all newly diagnosed cases of testicular cancer will be cured

1. Discuss the management of each type of early pregnancy loss.

a. Threatened abortion i. Of those in whom a live fetus is present, 94% will produce a live baby ii. Once a live fetus has been demonstrated to the couple on ultrasonography, management consists essentially of reassurance b. Incomplete abortion i. Once the patient's condition is stable, the remaining products of conception should be evacuated from the uterus using appropriate pain control. ii. Pathologic eval and prompt treatment c. Missed abortion i. US to confirm ii. Spontaneous d. Recurrent abortion i. Systemic disorder ii. Lupus anticoagulant iii. Hysteroscopy iv. Chromosome assessment v. over half of couples with recurrent losses will have normal findings during the standard evaluation. vi. congenital abnormalities of the uterus can now be diagnosed using pelvic ultrasonography vii. cervical incompetence 1. suture

1. What are the etiologies of recurrent pregnancy loss?

a. Three successive spontaneous abortions usually occur before a patient is considered to be a recurrent aborter. b. Infection i. Mycoplasma ii. Listeria toxoplasma c. Smoking and alcohol d. Psychological stress e. Medical disorders i. Dm ii. Hypothyroid iii. SLE iv. Age 30> 40>

Torsion of the appendix testis

a. Torsion of the appendix testis i. An appendage (appendix testis) on the testicles that is vestigial tissue may twist, making it difficult to distinguish from a testicular torsion. 1. The appendix testis is located at the superior pole of the testicle and is the most common cause of acute scrotal pain in children 2. absent cremasteric reflex. Blue dot sign ii. Treatment 1. Self limiting, rest ice scrotal support, increase in pain surgical referral

1. Discuss the transition from research to best practice.

a. Translational research simply means moving empirically based understanding into clinical practice b. "Application of the latest research into clinical practice is a mandatory requirement for improving healthcare delivery. c. Find practice issue d. Evidence for practice change e. Grade evidence f. PDSA g. Translational research is the movement of research from the bench to practice with the understanding of how the research supports the practice change to improve outcomes. Advancements in technology and health care research are changing the way nurses practice. Although large amounts of research evidence are continually being produced, it sometimes takes more than a decade to implement research into clinical practice

HTN

a. Treatment i. labetalol and hydralazine for HTN 1. Systolic blood pressure is ≥160 mm Hg or the diastolic blood pressure is ≥105 mm Hg. There is little evidence that lowering blood pressure below the 140/90 mm Hg range benefits the pregnancy a. Angiotensin-converting enzyme inhibitors, angiotensin II receptor blockers, renin inhibitors, and mineralocorticoid blockers should be avoided at all stages of pregnancy because of potential fetal toxicity. ii. eclampsia 1. As with any seizure condition, the initial requirement is to protect the patient from injury, clear the airway, and give oxygen by face mask to relieve hypoxia. Blood pressure and pulse oximetry should be recorded every 10 minutes with the patient in the lateral position. A 16- to 18-gauge IV line should be placed for drawing blood and administering drugs and fluids. An indwelling catheter should be placed in the bladder, and laboratory tests should be performed as outlined in 2. Magnesium sulfate at 6g loading 2 gm maintenance 3. Once hypoxia has been corrected, convulsions controlled, and the diastolic blood pressures brought down to the 90 to 100 mm Hg range, delivery should be expedited, preferably by the vaginal route. 4. Currently, there is no scientifically proven method for the prevention of preeclampsia

1. Intrahepatic cholestasis of pregnancy

a. Unknown patho b. S/S i. cholestasis and pruritus in the second half of pregnancy without other major liver dysfunction, ii. a tendency for recurrence with each pregnancy, (3) iii. an association with oral contraceptives and multiple gestations, iv. a benign course in that there are usually no maternal hepatic sequelae, and v. an increased rate of meconium-stained amniotic fluid and fetal demise. vi. The main symptom of ICP is itching (most intense on the palms and soles), without abdominal pain or a rash, which may occur as early as 20 weeks' gestation. 1. Jaundice is rarely observed. Laboratory tests show elevated levels of serum bile acids. c. Treatment i. Symptomatic treatment with cold baths, emollients, and antihistamines such as hydroxyzine may be of some help. The best results have been obtained with ursodeoxycholic acid.

1. Identify the preventative recommendations and common healthcare concerns for international travelers.

a. Vaccination b. Hep a c. Heb B d. Typhoid e. Rabies f. Yellow fever g. Cholera h. Meningococcal i. Malaria

1. Discuss iatrogenic anatomical changes of the vulva.

a. WHo i. Type I—Partial or total removal of the clitoris and/or prepuce (clitoridectomy) ii. Type II—Partial or total removal of the clitoris and labia minora, with or without the excision of the labia majora (excision) iii. Type III—Narrowing of the vaginal orifice with creation of a covering seal by cutting and appositioning the labia minora and/or the labia majora, with or without excision of the clitoris (infibulation) iv. Type IV—All other harmful procedures to the female genitalia for nonmedical purposes; for example, pricking, piercing, incising, scraping, and cauterizing

1. Falls

a. What three screening questions should providers ask their older patients on regular intervals? i. 1. Have you fallen in the past month, months, or year? ii. 2. Do you feel unsteady when standing or walking? iii. 3. Do you worry about falling? b. Which elements are performed as part of the TUG test? i. measures the patient's ability to stand from a seated position, walk 10 feet, turn, walk back, and sit down. Those who take longer than 12 seconds should receive further evaluation

1. Identify the benign conditions of the vulva.

a. White lesions i. Vitiligo 1. Loss of pigment / observation ii. Lichen sclerosis 1. Intense itching and dyspareunia/topical steroids, carcinoma iii. Lichen planus 1. Auto immune Wickham striae, classic fernlike or lacy patterns./ topical steroids iv. Lichen simples chronicus 1. Mimics psoriasa biosy needed/ immediate potency topical steroids v. Vulvovaginal atrophy 1. Postmenamaual decreased fat / estrogen creams/ moisturizers/ lubricants

1. Identify the appropriate age and interval of recommended health screening for women.

a. Woman 19-49 MMR, hep b, and varicella for those non immune. b. Girls 11-26 HPV vaccine c. Tdap 19-64 replace Td d. Women 50> influenza annually i. 19-49 annually for health care workers ii. Chronic diseases, HD, DM iii. Pregnant women e. Pneumococcal 65 >or chronic disease, alcohol, immunocompromised f. Contraindicated in pregnant women i. MMR ii. Varicella iii. HPV

1. of ectopic pregnancy.

a. about 1 in 80 pregnancies in the United States will be located outside the uterine cavity. The fallopian tubes are the site of over 95% of ectopic pregnancies. b. S/S 1. subtle lower abdominal discomfort and light uterine spotting to symptoms consistent with hypovolemic shock due to massive internal hemorrhage from tubal rupture 2. hemodynamic instability is indicated by tachycardia, diaphoresis, and hypotension. 3. The abdomen may be distended, and both abdominal guarding and rebound tenderness may be present. 4. There may be only minor bleeding from the cervix found by speculum examination, but noticeable cervical motion tenderness and a slightly enlarged, globular uterus may be detected by bimanual examination. A palpable adnexal mass may or may not be present

1. Discuss the management of asthma in pregnancy, labor and delivery.

a. mild intermittent asthma, i. a short-acting inhaled β2-agonist (albuterol) can be used as needed. b. mild persistent asthma i. should be treated with a daily low-dose inhaled glucocorticoid (budesonide). c. moderate persistent asthma i. is either a daily medium-dose inhaled glucocorticoid ii. combination of a daily low-dose inhaled glucocorticoid and a long-acting β2-agonist (salmeterol). d. severe persistent asthma i. should be treated with a daily high-dose inhaled glucocorticoid combined with a long-acting β2-agonist. They may require the addition of a systemic glucocorticoid. e. Labor i. Vaginal delivery and Prostaglandins should not be used, because they are likely to trigger acute bronchoconstriction.

The FNP understands the following about quality improvement (QI): (select two responses) QI needs to be realistic and achiveable Feedback: QI needs to be realistic and achievable. Large QI studies are not necessary and can be intimidating or overwhelming to those engaging in the project; small projects with available resources and tools are best. QI is not research, the intent is not to generate new ideas and knowledge for publication and dissemination, but to use existing tools and evidence to change and improve nursing practice and patient care. QI involves large-scale projects for best outcomes QI is action research QI generates new ideas and knowledge QI avoids replicating strategies used by others QI uses existing tools and evidence to change and improve nursing practice

d-Feedback: QI needs to be realistic and achievable. Large QI studies are not necessary and can be intimidating or overwhelming to those engaging in the project; small projects with available resources and tools are best. QI is not research, the intent is not to generate new ideas and knowledge for publication and dissemination, but to use existing tools and evidence to change and improve nursing practice and patient care.

According to multiple research studies, which intervention has resulted in lower costs and fewer rehospitalizations in high-risk older patients? Inclusion of extended family members in the outpatient plan of care Frequent posthospital clinic visits with a primary care provide Telephone follow-up by the pharmacist to assess medication compliance Coordination of posthospital care by advanced practice health care providers

d-Feedback: Research studies provided evidence that high-risk older patients who had posthospital care coordinated by an APN had reduced rehospitalization rates. It did not include clinic visits with a primary care provider, inclusion of extended family members in the plan of care, or telephone follow-up by a pharmacist.

a. Metabolism

i. Aldosterone is a mineralocorticoid synthesized in the zona glomerulosa of the adrenal cortex. The main source in pregnancy is the maternal adrenal. 1. Aldosterone secretion is regulated by the renin-angiotensin system. 2. Aldosterone stimulates the absorption of sodium and the secretion of potassium in the distal tubule of the kidney, thereby maintaining sodium and potassium balance. 3. Aldosterone secretion rates decline in pregnancy-induced hypertension and, in some cases, may fall below nonpregnant levels. ii. Although calcium absorption is increased in pregnancy, total maternal serum calcium declines. The fall in total calcium parallels that of serum albumin, 1. Ionic calcium, the physiologically important calcium fraction, remains essentially constant throughout pregnancy because of increased maternal production of parathyroid hormone. The latter facilitates the transfer of calcium across the placenta to the fetus for adequate bone development, and at the same time the mobilization of calcium from the mother's skeleton to maintain adequate calcium homeostasis 2. Calcium ions are actively transported across the placenta, and fetal serum levels of total as well as ionized calcium are higher than maternal levels in late pregnancy

i. the overall risk of immunization for the second full-term, RhD-positive, ABO-compatible pregnancy is about one in six pregnancies.

i. All pregnant RhD-negative women who are not sensitized to the D antigen should routinely receive prophylactic Rh immune globulin at 28 weeks' gestation, within 72 hours of delivery of an RhD-positive fetus, and at the time of recognition of any of the problems cited above that are associated with fetomaternal hemorrhage.

a. Mild cognitive impairment

i. Amnestic MCI is strongly associated with progression to Alzheimer's dementia ii. Single domain amnestic MCI: + memory loss, but no other deficits iii. 2. Multiple domain amnestic MCI: + memory loss and other deficits iv. 3. Single domain nonamnestic MCI: + impairment in a single, but not memory-related, cognitive domain v. 4. Multiple domain nonamnestic MCI: + impairment in multiple, but not memory-related, cognitive domain.

a. Ulcerations a fissures

i. Aphthous ulcers 1. Painful like canker sores 2cm wide to 1cm deep / symptomatic ii. Bechet disease 1. Genital and oral ulcers with uveitis in combo with antivirals or IBS/depends iii. Chrons disease 1. Granulomatous intestinal tract inflammatory process/ treat bowels disease iv. Traumatic ulcerations 1. Scratching itching neglect /counseling suspect psych disorder

a. Cardiac

i. Arteriosclerosis ii. decrease in "nitrous oxide mediated vasodilated responses" throughout the vasculature iii. combination of oxidative stress (related to reactive oxygen species) and increased arginase (an enzyme), which limits nitrous oxide production. iv. These processes combined with endothelial senescence, cause endothelial cell injury and dysfunction, as well as a decrease in vascular homeostasis 1. Autonomic nervous system changes ] not as responsive to beta adrenergic or parasympathetic stimulation. 2. cardiac decompensation that occurs when elders are stressed by illness or dehydration, as well orthostatic hypotension and syncope

a. Low infant weight gain

i. Assess feeding session ii. A supplemental nursing system can deliver supplemental formula or pooled pasteurized human milk while the infant is breastfeeding. Supplemental nursing systems offer an important alternative to bottles; breastfed babies who are offered bottles sometimes have difficulty returning to the breast, especially when the mother's milk supply is low. The supplemental nursing system offers increased flow at the breast to keep the baby interested and engaged while increasing stimulation and extraction at the breast.

a. Cystic Masses

i. Bartholin cyst 1. Cystic mass below hymenal ring at 4 or 8 o'clock; must rule out underlying malignancy in older women/ no treatment; if symptomatic, consider drainage with Word catheter placement or marsupialization ii. Gartner duct cyst 1. soft cystic structures formed from remnants of the Wolffian duct;/ asymptomatic

1. Differentiate the signs of pregnancy a. Presumptive Skin and mucus membrane changes

i. Chadwick sign dark discoloration of vulva vaginal walls ii. Abdominal stria iii. Linea nigra midline of lower abdomen iv. Chloasma pigmentation under eyes (hormonal contraceptives) b. Probable physical changes i. Uterine enlargement Piskacek Sign ii. Hagar Sign softer cervix iii. Pregnancy test c. Positive i. Fetal heartbeat endo ultrasound (6 weeks) ii. Fetal movements (7-8 weeks)

a. Hearing

i. Conductive Less common ii. Sensorineural hearing loss (i.e., presbycusis) is an expected aging change, although the cause is variable: genetic, illness, medications (e.g., furosemide), loud environmental noise, trauma, or just aging.

a. Congenital heart disease

i. Congenital heart disease includes atrial or ventricular septal defects, valvular defects, primary pulmonary hypertension (Eisenmenger syndrome), and cyanotic heart diseases such as tetralogy of Fallot and transposition of the great arteries if surgically treated progress through pregnancy well. ii. patients with primary pulmonary hypertension or cyanotic heart disease with residual pulmonary hypertension are in danger of experiencing decompensation during pregnancy. iii. significant pulmonary hypertension with Eisenmenger syndrome is a contraindication to pregnancy due to the high maternal mortality that accompanies this condition.

a. Glucocorticoids

i. Cortisol is derived from circulating cholesterol ii. Both the fetal adrenal and the placenta participate in cortisol metabolism. iii. Toward the end of pregnancy cortisol promotes differentiation of type II alveolar cells and the biosynthesis and release of surfactant into the alveoli. iv. Cortisol also plays an important role in the activation of labor, increasing the release of placental CRH and prostaglandins.

S/Sa. Trichomoniasis vaginitis

i. Definition: infection caused by Trichomonas vaginalis, most common anaerobic, flagellated protozoan found in the vagina is considered an STI ii. Signs and symptoms 1. Yellow-green frothy discharge and foul odor with appearance of "strawberry cervix" 2. Vulvar itching; redness and burning of genitalia 3. Dysuria 4. Dyspareunia and symptoms may increase with intercourse or after menses

a. UTI

i. Women should undergo a test of cure after treatment and have a repeat urine culture every 6 to 12 weeks for the remainder of the pregnancy, and a urine dipstick should be performed at every visit to evaluate for blood, nitrites, and leukocytes.10 After two positive cultures, suppressive therapy should be given ii. painful vaginal bleeding in association with uterine tenderness, hyperactivity, and increased tone iii. Sulfonamides and nitrofurantoin are contraindicates in glucose-6phosphate dehydratase deficiency. 1. Not fist line use in 1st trimester studies

a. Immune thrombocytopenia

i. Gestational thrombocytopenia is unlikely to present with a platelet count less than 70,000/µL, is not associated with bleeding complications, does not require therapy, occurs late in pregnancy, and resolves after delivery. ii. Treatment 1. Oral prednisone at a dose of 1 mg/kg per day is given initially, and once platelet counts improve, it is tapered off over several weeks. 2. Severe ITP can be more rapidly treated with intravenous immunoglobulin (IVIG). 3. In patients with life-threatening hemorrhage, platelet transfusions combined with high-dose steroids and IVIG may be required. 4. Splenectomy is a last resort for patients who do not respond to medical therapy 5. . Maternal hemorrhage is unlikely if the platelet count is greater than 40,000/µL at the time of delivery, but concerns about the possibility of an epidural hematoma may preclude the use of epidural anesthesia. 6. Advanced consultation with anesthesia staff is advised. The neonate should be monitored for thrombocytopenia, because 7. placental transfer of maternal antiplatelet antibodies can occur. Rarely, neonatal intracranial hemorrhage may occur once the neonatal platelet count has reached its nadir after the first 2 to 3 days of life. 8. There is no correlation between fetal platelet counts and neonatal outcome; thus, monitoring fetal platelet counts is not done in pregnancy. 9. Vaginal delivery is generally preferred, because there is little evidence that the fetal outcome is improved by cesarean delivery and surgery carries additional maternal risks.

a. Zika exposure

i. Health care providers should contact their local or state health department to facilitate testing.37 Women with confirmed or suspected Zika exposure should have serial ultrasounds every 3 to 4 weeks that include detailed fetal anatomy, particularly fetal neuroanatomy.3

a. Human Placental Lactogen

i. Human placental lactogen antagonizes the cellular action of insulin and decreases maternal glucose utilization, which increases glucose availability to the fetus. Gestational diabetes

a. Atrophic vaginitis

i. Hypoestrogenic women having undergone natural or surgical menopause may have dyspareunia and postcoital bleeding resulting from atrophy of the vaginal and vulvar epithelium. ii. Treatment Atrophic vaginitis is treated with vaginal estrogen cream.

a. PID s/s

i. II. Signs and symptoms 1. Minimum criteria for diagnosis: treat as PID if the woman is sexually active, no other cause is identified, and she meets these three criteria 2. Lower abdominal tenderness; worsens with movement and intercourse 3. Adnexal tenderness and/or mass 4. Cervical motion tenderness (CMT [highest sensitivity]) with purulent endocervical exudate (highest specificity) 5. Oral temperature >101°F 6. AUB and vaginal discharge 7. N/V 8. Dysuria, urinary frequency

a. PID test

i. III. Diagnostic testing 1. Elevated WBC, ESR, C-reactive protein 2. STI test for Neisseria gonorrhoeae or Chlamydia trachomatis (CT) 3. Transvaginal or pelvic U/S may show thickened, fluid-filled tubes or free pelvic fluid 4. Urine hCG (do not miss pregnancy)

ectopic pregnancy.a. Medical management with methotrexate

i. In general, medical management is preferred for an early ectopic pregnancy, and surgery is reserved for unstable patients, those whose diagnosis is uncertain, and those whose medical therapy has failed. ii. Patients being treated with MTX should be instructed to avoid folate supplements, nonsteroidal antiinflammatory agents, and alcohol. Pelvic rest (no sexual activity) is required, and women should also avoid sunlight exposure and vigorous physical exercise. Women should be warned about potential side effects of MTX. Gastrointestinal side effects, stomatitis, and hair loss are possible, especially with more prolonged therapy. Women may experience abdominal pain 2 to 3 days after injection, which is potentially caused by continued expansion of the pregnancy mass.

a. What are some Geriatric specific pharmacology and prescribing concerns?

i. In older adults, a new symptom should be considered a potential drug-drug interaction until proven otherwise ii. Medication reconciliation is required at every visit and needs to include the current dose of each medication. Encourage the "brown bag review" iii. All over-the-counter medications (e.g., vitamins, acetaminophen, NSAIDs, supplements) and herbals should be queried and documented. iv. Ask the patient/family member if any "borrowed" medications were obtained from friends or relatives for pain or other reasons. v. If necessary, call the pharmacy to determine whether the patient has stopped refilling. vi. Allergy review at each encounter with the documented specific adverse reaction (e.g., nausea versus angioedema versus previous ADR) vii. Review patient comorbidities at each visit to be certain patient is stable viii. Check routine renal status every 3 months and adjust patient medication doses appropriately. 1. For example, if a patient with type 2 diabetes is taking metformin 1000 milligrams po twice a day, but the patient's estimated GFR has been slowing deteriorating over time and is now 40 mL/min/1.73 m2, a dose reduction to metformin 1000 milligrams po once a day would be indicated. Then it would be necessary to carefully follow renal status every 3 months and discontinue the metformin if the estimated GFR falls below 30 mL/min/1.73 m2.31 ix. Check liver function periodically, especially in patients with fatty liverdisease or cirrhosis. x. Check for drug-drug interactions (e.g., narcotic and benzodiazepine or histamine H1 antagonist), drug-alcohol or food interactions (e.g., bleeding, dizziness, fainting, falls, depression), and drug-disease interactions (e.g., aspirin or NSAID use if peptic ulcer disease, NSAID use if hypertension, heart failure). xi. Assess drug effectiveness. xii. Avoid the prescribing cascade.

a. Homeostasis

i. In pregnancy, the insulin response to glucose stimulation is augmented. By the 10th week of normal pregnancy and continuing to term, fasting concentrations of insulin are elevated and those of glucose reduced. ii. Glycogen synthesis and storage by the liver increases, and gluconeogenesis is inhibited. iii. After early pregnancy, insulin resistance emerges, so glucose tolerance is impaired. The fall in serum glucose for a given dose of insulin is reduced compared with the response to earlier pregnancy. iv. A variety of humoral factors derived from the placenta have been suggested to account for the antiinsulin environment of the latter part of pregnancy. Perhaps the most important are cytokines and human placental lactogen (hPL) v. During the second half of pregnancy, probably as a result of rising hPL levels, lipolysis is augmented, and fasting plasma concentrations of free fatty acids are elevated. vi. Pregnancy is thus associated with an increased risk of ketoacidosis, especially after prolonged fasting. vii. rise in fasting triglyceride concentration.

a. Renal system

i. The urinary collecting system, including the calyces, renal pelves, and ureters, undergoes marked dilation in pregnancy, ii. As the uterus enlarges, partial obstruction of the ureter occurs at the pelvic brim in both the supine and the upright positions. iii. Renal plasma flow and the glomerular filtration rate (GFR) increase early in pregnancy. Maximum plateau elevations of at least 40-50% above nonpregnant levels are reached by mid-gestation, and they remain unchanged to term iv. The elevated GFR is reflected in lower serum levels of creatinine and urea nitrogen, v. sodium balance is maintained with exquisite precision. vi. Potassium metabolism during pregnancy remains unchanged vii. The hyperventilation (low partial pressure of CO2 in arterial blood [PaCO2]) of pregnancy results in respiratory alkalosis, which is compensated by renal excretion of bicarbonate. viii. The maternal extracellular volume, which consists of intravascular and interstitial components, increases throughout pregnancy, leading to a state of physiologic extracellular hypervolemia. ix. Renin levels remain elevated throughout pregnancy, x. The uterus and placenta, like the kidney, can produce renin, and extremely high concentrations of renin occur in the amniotic fluid.

a. Immunologic responses during pregnancy The mother's immunologic defense system remains intact during pregnancy

i. There are three reasons to suggest that vitamin D may be an important regulator of the immune system during pregnancy. ii. Pregnant women are at higher risk of severe infection and death from certain pathogens such as viruses (hepatitis, influenza, varicella, cytomegalovirus, polio), bacteria (listeria, streptococcus, gonorrhea, salmonella, leprosy), and parasites (malaria, coccidioidomycosis) compared with nonpregnant women. iii. SUPPLEMENT VITAMIN D

a. Congenital hypothyroidism

i. Thyroid hormone deficiency during the fetal and early neonatal periods leads to generalized cognitive impairment ii. Newborn screening programs can identify many cases of congenital hypothyroidism, and with early administration of thyroid hormonereplacement, the impairment can be minimized.

GD MGMT

i. To achieve an optimal outcome, the patient's fasting blood glucose level should be less than 95 mg/dL, with the 1-hour postprandial glucose level less than 140 mg/dL and the 2-hour postprandial glucose level less than 120 mg/dL. ii. Optimal less than 95 for fasting iii. Less than 140 after 1 hour iv. Less than 120 after 2 hours v. 45/20/20 Carb/Protein/fat

a. Second-trimester screen

i. Traditionally, a woman was offered the serum triple screening test that measures α-fetoprotein (AFP), hCG, and unconjugated estriol (UE3) at 16 to 20 weeks' gestation. ii. If the MSAFP level is elevated, an ultrasound should be done to rule out multiple gestation, fetal demise, or inaccurate gestational age (all of which can give false-positive results). iii. If none of these factors are present, amniocentesis is recommended to determine the amniotic fluid AFP level and to measure acetylcholinesterase (AChE). Acetylcholinesterase is a protein that is present only if there is an open neural tube defect. iv. The combination of low MSAFP, elevated hCG, and low UE3 levels (triple screen) has a detection rate for Down syndrome of approximately 70%, with a positive screen result in approximately 5% of all pregnancies.

TX--a. Chlamydia trachomatis

i. Treatment 1. Recommended therapy: azithromycin 1 g once or doxycycline 100 mg bid for 7 days (if treating empirically and suspect N. gonorrhoeae, also treat with ceftriaxone 250 mg IM once) 2. Alternative therapy for 7 days (caution with pregnancy or lactation) 3. Erythromycin base 500 mg qid (may cause GI upset) 4. Levofloxacin 500 mg qd (caution if <18 yr of age) 5. Ofloxacin 300 mg bid (caution if <18 yr of age) a. When treating, consider gonorrhea; consider treating all sexual partners exposed within the last 60 days b. Avoid sexual intercourse until therapy is completed (or for 7 days after single-dose therapy) c. Return for repeat "test of cure" in 3 mo (if suspect poor compliance, can retest in 4 wk after treatment) d. Report the infection to the health department

a. Vulvovaginal candidiasis tx

i. Treatment 1. Topical OTC 2. Clotrimazole (Gyne-Lotrimin) 1% crm: 1 applicator hs for 7 to 14 days 3. Miconazole (Monistat) 2% crm: 1 applicator hs for 7 days 4. Tioconazole (Vagistat) 6.5% ointment: 1 applicator hs once 5. Topical prescription (if recurring, increase number of days used) 6. Butoconazole (Gynazole-1) 2% crm: 100 mg/applicator once. 7. Terconazole (Terozol-3) 80 mg vaginal supp or 40 mg/applicator: 1 supp or applicator qd ×3d 8. Diflucan 150 mg tab PO once; may repeat q3d for three doses if not improved or recurring 9. Treat causes to prevent recurrence 10. Not recommended in first or second trimester of pregnancy (causes miscarriage)

1. What services are NOT included in hospice care?

i. Treatment for cure ii. Care that was not ordered from hospice provider/team iii. Room and BOARD iv. EMS or outpatient care unless arranged by hospice

TX-a. Trichomoniasis vaginitis

i. Treatment should include partner i. Initial treatment 1. Metronidazole 2 g orally once 2. Tinidazole 2 g orally once 3. Metronidazole 500 mg bid for 7 days (more effective for men) ii. Recurrent treatment 1. Metronidazole 500 mg bid ×7d 2. Metronidazole 2 g qd ×7d 3. Tinidazole 2 g qd ×7d iii. Consider betadine or vinegar douche qd0 for 1 mo iv. Follow-up women in 3 mo for recheck of vaginal secretions v. No sexual intercourse for 7 days after treatment vi. Refer to GYN if symptoms persist or if pregnant

a. Tubo-ovarian abscess

i. Tubo-ovarian abscess (TOA), an endstage process of acute PID, is diagnosed when a patient with PID has a pelvic mass that is palpable during bimanual examination. ii. About 75% of women with a tubo-ovarian abscess respond to antimicrobial therapy alone. Failure of medical therapy suggests the need for drainage of the abscess. iii. Although drainage may require surgical exploration, percutaneous drainage, guided by imaging studies (ultrasonography or computed tomography) should be used as an initial option if possible.

a. Ulcerations and fistulas

i. Ulcers 1. most ulceration of the vagina is associated with acute infection due to herpes simplex or cytomegalovirus;/ steroids ii. Fistulas 1. bladder or rectum into the vagina may occur due to surgical complication, infection, or malignancy/ surgical

a. Group B streptococcus

i. Universal screening of pregnant women at 35 to 37 weeks' gestation by sterile rectovaginal swab is used to identify GBS in pregnant women.34 ii. If in urine, consider GBS positive iii. If previous GBS disease prophylactic abt for labor no cultures needed

a. Acute pyelonephritis

i. cystitis-like illness with mild flank pain. E. coli accounts for more than 80% of these cases. Microscopy of unspun urine reveals pyuria and gram-negative bacteria. A urine culture ii. . Pyelonephritis in a pregnant patient can cause premature labor and preterm delivery if not treated promptly. iii. trimethoprim-sulfamethoxazole (160/800 mg every 12 hours for 14 days) or a quinolone (e.g., levofloxacin 750 mg daily for 7 days). Inpatient treatment regimens include the use of parenteral levofloxacin (750 mg daily), ceftriaxone (1 to 2 g daily), ampicillin (1 g every 6 hours), and gentamicin (especially if Enterococcus species are suspected) or aztreonam (1 g every 8 to 12 hours). Symptoms should resolve after 48 to 72 hours. iv. If fever and flank pain persist after 72 hours of therapy, ultrasonography or computed tomography should be considered to rule out a perinephric or intrarenal abscess or ureteral obstruction

S/Sa. Chlamydia trachomatis

i. ectopic pregnancy, and PID ii. Signs and symptoms 1. May be asymptomatic with mild dyspareunia 2. Mucopurulent cervicitis or vaginal discharge with spotting usually after intercourse 3. Abdominal or pelvic pain or "heaviness" 4. Pelvic examination: positive CMT with purulent discharge from cervix; cervix is friable with erosions and irritation 5. Dysuria

a. Post-term pregnancy

i. failure of the timely onset of labor and the fetus is not delivered at or before 42 weeks' gestation. For the last 30 years, practitioners have recognized that beginning at 41 weeks' gestational age, the risk of fetal distress increases and the fetus fails to continue to grow, increasing the risk of morbidity and mortality. Consequently, fetal assessment techniques have been developed to assess fetal well-being when labor and delivery are delayed. When signs of fetal distress are identified, labor should be induced to rescue the fetus from a potentially hostile environment

a. IUFD

i. fetal death between 20 weeks' gestation and the onset of labor. With improved management of IUGR and postterm pregnancies, the incidence of IUFD has decreased dramatically. 1. About 80% of patients experience the spontaneous onset of labor within 2 to 3 weeks of fetal demise. 2. Vaginal suppositories of prostaglandin E2 (dinoprostone [Prostin E2]) can be used from the 12th to the 28th week of gestation. 3. Misoprostol (Cytotec, a synthetic prostaglandin E1 analogue) vaginal tablets have been found to be quite effective with little or no gastrointestinal side effects, and they are less expensive than dinoprostone. 4. After 28 weeks' gestation, if the condition of the cervix is favorable for induction and there are no contraindications, Cytotec followed by oxytocin are the drugs of choice. ii. weekly fibrinogen levels should be monitored during the period of expectant management, along with a hematocrit iii. all women should be tested for the presence of anticardiolipin antibodies. thrombophilias 1. If congenital abnormalities are detected, fetal chromosomal studies and total body radiographs should be done, in addition to a complete autopsy. 2. In a stillborn fetus, the best tissue for a chromosomal analysis is the fascia lata, obtained from the lateral aspect of the thigh. The tissue can be stored in saline or Hanks solution 3. A significant number of cases of IUFD are the result of fetomaternal hemorrhage, which can be detected by identifying fetal erythrocytes in maternal blood (Kleihauer-Betke test). a. Referral to a bereavement support group for counseling is advisable.and platelet count.

a. Constipation and hemorrhoids

i. increased water and fiber intake, intake of foods with laxative properties (e.g., prunes, decaffeinated coffee), and moderate physical activity. Bulking agents (psyllium, methylcellulose) have been shown to be safe in pregnancy, even for long-term use, and are first line treatments, usually 1 tablespoon in 8 ounces of fluid 1 to 3 times a day. ii. The stool softener docusate sodium is also considered safe in pregnancy. Mineral oil, castor oil, and saline should be avoided during pregnancy.

a. Dementia

i. intellectual dysfunction and behavioral changes. ii. Vascular dementia is associated with cardiovascular disease, and there are infarctions that occur throughout the brain. iii. Lewy body dementia have "Lewy bodies" (e.g., α-synuclein inclusions) that cause anxiety, confusion, disorientation, memory loss, poor judgment, and the other symptoms associated with dementia. iv. Frontotemporal dementia is caused by neurodegeneration (cerebral tissue death) in the frontal temporal lobes v. frontotemporal dementia 1. recommended to try cholinesterase inhibitors in patients with Alzheimer's dementia, as well as those with vascular dementia, Parkinson's disease dementia, and Lewy body dem

a. transverse vaginal septum,

i. is most commonly found at the junction of the upper and middle thirds of the vagina a. midline longitudinal septum i. double cervix

a. Chronic hypertension

i. known hypertension before pregnancy or the development of hypertension before 20 weeks' gestation. ii. BP not treated in pregnancy unless the blood pressure is ≥160/105 iii. . It is not uncommon for the physiologic stress of pregnancy to cause subclinical vascular or renal disease to become manifest.

a. Antiphospholipid syndrome

i. lupus anticoagulant, anticardiolipin immunoglobulin G (IgG) or IgM antibodies, and β2-glycoprotein-1 antibodies. ii. The antiphospholipid syndrome is defined as the presence of at least one of these antibodies in association with arterial or venous thrombosis and/or one or more obstetric complications. iii. These complications include an unexplained fetal demise after 10 weeks' gestation, a history of preterm delivery before 34 weeks' gestation due to severe preeclampsia or placental insufficiency, or three or more unexplained miscarriages before 10 weeks' gestation. iv. Lupus anticoagulant can be screened for with an activated partial prothrombin time (aPTT) or with the dilute Russell viper venom test, a sensitive and specific radioimmunoassay that is available for the detection of anticardiolipin. v. a history of antiphospholipid syndrome is treated with prophylactic low-molecular-weight heparin and low-dose aspirin (81 mg), unless there is a history of thrombosis, in which case a full dosage of anticoagulants is indicated.

a. Corticotropin-releasing hormone

i. measured as early as 12 weeks' gestation ii. Fetal cortisol stimulates placental CRH release, which then stimulates fetal ACTH secretion, completing a positive feedback loop that plays an important role in the activation and amplification of labor, both preterm and term.

a. Hypothyroidism

i. most important laboratory finding of hypothyroidism is an elevated TSH level. 1. untreated maternal hypothyroidism has been associated with an increased risk of spontaneous abortion, preeclampsia, abruption, low-birth-weight or stillborn infants, and lower cognitive function in offspring.

a. Syphilis S/S

i. systemic disease caused by Treponema pallidum, Incubation period is 10 to 90 days, Recent outbreaks of syphilis have been associated with increases in HIV infection, drug use, and poverty condition ii. II. Signs and symptoms (may occur at any stage up to 30 yr after exposure) 1. Early syphilis has three periods of infectivity a. Primary stage consists of single painless firm, round ulcer (chancre) at site of infection; heals in 3 to 6 wk b. Secondary stage consists of A. Nonpruritic rough, reddish brown macular rash on torso, hands, and feet or large, raised white lesions in warm, moist areas of mouth or perineum (condyloma lata) B. Lymphadenopathy particularly in epitrochlear area; nodes are nontender and rubbery C. Anorexia, weight loss, arthralgia 2. Early latent stage may have no obvious symptoms, but disease can be transmitted a. Has positive serology b. Occurs within first year of infection 3. Latent syphilis and neurosyphilis are usually asymptomatic for years before symptoms occur and include progressive cardiovascular, skin, and neurologic symptoms a. Dementia b. Gradual blindness and deafness c. Loss of coordination, paralysis, numbness d. Aortic dilation and aortic valve regurgitation, ascending aortic arch calcification e. Skin manifestation with ulcers, elevated round, irregular, serpiginous-shaped lesions (gummas) usually seen in HIV+ persons

a. Orchitis

i. systemic, blood-borne infection that results in an acute inflammation of one or both testicles. 1. It may coexist with infections of the prostate and epididymis; be a consequence of systemic viral infections, such as mumps; 2. complication of syphilis, mycobacterial infections, or fungal infections.9 3. Orchitis is commonly caused by C. trachomatis and N. gonorrhoeae in adolescents and urinary tract pathogens such as E. coli in men older than 35.12,14 4. When orchitis is a complication of mumps, it is seen in 25% of postpubertal males and may be accompanied by a hydrocele and scrotal wall thickening ii. Treatment Same as epididymis

1. Review the general principles of caring for sexual and gender minority people.

i. the ability to identify personal biases that may affect the quality of care. ii. (e.g., rainbow flag or pink triangle). A clearly posted nondiscrimination statement that includes references to SO/GI can serve as another cue that the practice welcomes all patients. Inclusion of images of same sex couples and families as well as transgender people in online and print materials (e.g., brochures, patient education) are other cues to SGM people that the practice is a welcoming on iii. Pronouns iv. Gender neutral bathrooms v. Avoidance of gender role

a. Gestational hypertension

i. the development of elevated blood pressure without proteinuria or other signs of preeclampsia after 20 weeks' gestation or within 72 hours of delivery thatresolves by 12 weeks postpartum. ii. diagnosis of gestational hypertension can only be made in retrospect, if the pregnancy has been completed without the development of proteinuria or other evidence of preeclampsia, and if the blood pressure has returned to normal before the 12th week postpartum

a. Varicocele

i. varicocele is an abnormal dilation of the pampiniform plexus and spermatic veins in the spermatic cord 1. incompetent valves within the pampiniform venous plexus, which results in a backflow of blood and venous pooling. 2. Varicoceles usually develop slowly, are often symptomatic, and can lead to testicular damage or dysfunction and male infertility a. A right-sided varicocele is a a secondary cause of the varicocele, specifically an abdominal, pelvic, or retroperitoneal mass. 3. soft mass reveals a "bag of worms" on the proximal spermatic cord, more frequently encountered on the left side ii. Treatment 1. Surgical treatment by ligation of the spermatic vein (varicocelectomy) is often the treatment of choice, and there are several different surgical approaches

a. Adenosis of the vaginal

i. wall consists of islands of columnar epithelium in the normal squamous epithelium.

. Female pseudo hermaphroditism is caused by

in utero masculinization due to androgens from maternal or fetal congenital adrenal hyperplasia, androgen-producing tumors of the mother's ovary or adrenal glands, or the mother's use of exogenous androgens. Often the infant will present with ambiguous genitalia. The enlarged clitoris is the most conspicuous abnormality.

Clitoral agenesis may result from the failure

of the genital tubercle to develop. Incomplete development of the genitalia can result in a cloaca with no separation of the bladder and the vagina.

a. The most significant of the vulvar anomalies are those that

pose challenges to the assignment of gender at birth.


Conjuntos de estudio relacionados

Econ Ch. 20- Aggregate Demand and Aggregate Supply

View Set